You are on page 1of 311

United States Physics Team

Contest Papers with Solutions

(2007-2014)

United States Physics Team


F= ma Contest
2007

2007 F=ma Contest


1. An object moves in two dimensions according to
r (t ) = (4.0t 2 9.0)i + (2.0t 5.0) j ,
where r is in meters and t in seconds. When does the object cross the x-axis?

(a)
(b)
(c)
(d)
(e)

0.0 s
0.4 s
0.6 s
1.5 s
2.5 s

2. The graph shows velocity as a


function of time for a car. What
was the acceleration at time = 90
seconds?
(a)
(b)
(c)
(d)
(e)

0.22 m/s2
0.33 m/s2
1.0 m/s2
9.8 m/s2
30 m/s2

3. The coordinate of an object is given as a function of time by x = 8t - 3t2, where x is in


meters and t is in seconds. Its average velocity over the interval from t = 1 to t = 2s is
(a)
(b)
(c)
(d)
(e)

-2 m/s
-1 m/s
-0.5 m/s
0.5 m/s
1 m/s

4. An object is released from rest and falls a distance h during the first second of time.
How far will it fall during the next second of time?
(a)
(b)
(c)
(d)
(e)

h
2h
3h
4h
h2

5. A crate of toys remains at rest on a sleigh as the sleigh is pulled up a hill with an
increasing speed. The crate is not fastened down to the sleigh. What force is responsible
for the crates increase in speed up the hill?
(a)
(b)
(c)
(d)
(e)

the force of static friction of the sleigh on the crate


the contact force (normal force) of the ground on the sleigh
the contact force (normal force) of the sleigh on the crate
the gravitational force acting on the sleigh
no force is needed

6. At time t = 0 a drag racer starts from rest at the origin and moves along a straight line
with velocity given by v = 5t2, where v is in m/s and t in s. The expression for the
displacement of the car from t = 0 to time t is
(a)
(b)
(c)
(d)
(e)

5t3
5t3/3
10t
15t2
5t/2

7. The chemical potential energy stored in a battery is converted into kinetic energy in a toy
car that increases its speed first from 0 mph to 2 mph and then from 2 mph up to 4 mph.
Ignore the energy transferred to thermal energy due to friction and air resistance.
Compared to the energy required to go from 0 to 2 mph, the energy required to go from 2
to 4 mph is
(a)
(b)
(c)
(d)
(e)

half the amount.


the same amount.
twice the amount.
three times the amount.
four times the amount.

8. When two stars are very far apart their gravitational potential energy is zero; when they
are separated by a distance d the gravitational potential energy of the system is U. If
the stars are separated by a distance 2d the gravitational potential energy of the system
is
(a)
(b)
(c)
(d)
(e)

U/4
U/2
U
2U
4U

9. A large wedge rests on a horizontal


frictionless surface, as shown. A block
starts from rest and slides down the
inclined surface of the wedge, which is
rough. During the motion of the block, the
center of mass of the block and wedge
(a)
(b)
(c)
(d)
(e)

does not move


moves horizontally with constant speed
moves horizontally with increasing speed
moves vertically with increasing speed
moves both horizontally and vertically

10. Two wheels with fixed hubs, each having


a mass of 1 kg, start from rest, and forces
are applied as shown. Assume the hubs
and spokes are massless, so that the
rotational inertia is I = mR2. In order to
impart identical angular accelerations
about their respective hubs, how large
must F2 be?
(a)
(b)
(c)
(d)
(e)

0.25 N
0.5 N
1N
2N
4N

11. A uniform disk, a thin hoop, and a uniform sphere, all with the same mass and same
outer radius, are each free to rotate about a fixed axis through its center. Assume the
hoop is connected to the rotation axis by light spokes. With the objects starting from
rest, identical forces are simultaneously applied to the rims, as shown. Rank the objects
according to their kinetic energies after a given time t, from least to greatest.
(a)
(b)
(c)
(d)
(e)

disk, hoop, sphere


sphere, disk, hoop
hoop, sphere, disk
disk, sphere, hoop
hoop, disk, sphere

12. A 2-kg rock is suspended by a


massless string from one end of a
uniform 1-meter measuring stick.
What is the mass of the measuring
stick if it is balanced by a support
force at the 0.20-meter mark?
(a)
(b)
(c)
(d)
(e)

0.20 kg
1.00 kg
1.33 kg
2.00 kg
3.00 kg

13. A particle moves along the x-axis. It collides elastically head-on with an identical
particle initially at rest. Which of the following graphs could illustrate the momentum of
each particle as a function of time?

14. When the speed of a rear-drive car is increasing on a horizontal road, the direction of
the frictional force on the tires is
(a)
(b)
(c)
(d)
(e)

backward on the front tires and forward on the rear tires.


forward on the front tires and backward on the rear tires.
forward on all tires.
backward on all tires.
zero.

15. A uniform disk (I = MR2) of mass 8.0 kg can rotate without


friction on a fixed axis. A string is wrapped around its
circumference and is attached to a 6.0 kg mass. The string does
not slip. What is the tension in the cord while the mass is
falling?
(a)
(b)
(c)
(d)
(e)

20.0 N
24.0 N
34.3 N
60.0 N
80.0 N

16. A baseball is dropped on top of a basketball. The basketball hits the


ground, rebounds with a speed of 4.0 m/s, and collides with the baseball as
it is moving downward at 4.0 m/s. After the collision, the baseball moves
upward as shown in the figure and the basketball is instantaneously at rest
right after the collision. The mass of the baseball is 0.2 kg and the mass of
the basketball is 0.5 kg. Ignore air resistance and ignore any changes in
velocities due to gravity during the very short collision times. The speed of
the baseball right after colliding with the upward moving basketball is
(a)
(b)
(c)
(d)
(e)

4.0 m/s
6.0 m/s
8.0 m/s
12.0 m/s
16.0 m/s

17. A small point-like object is thrown horizontally off of a 50.0-m high building with an
initial speed of 10.0 m/s. At any point along the trajectory there is an acceleration
component tangential to the trajectory and an acceleration component perpendicular to
the trajectory. How many seconds after the object is thrown is the tangential
component of the acceleration of the object equal to twice the perpendicular
component of the acceleration of the object? Ignore air resistance.
(a) 2.00 s
(b) 1.50 s
(c) 1.00 s
(d) 0.50 s
(e) The building is not high enough for this to occur.

18. A small chunk of ice falls from rest


down a frictionless parabolic ice sheet
shown in the figure. At the point
labeled A in the diagram, the ice sheet
becomes a steady, rough incline of
angle 30 0 with respect to the
horizontal and friction coefficient k .
3
This incline is of length h and ends
2
at a cliff. The chunk of ice comes to
rest precisely at the end of the incline.
What is the coefficient of friction k ?
(a)
(b)
(c)
(d)
(e)

300

0.866
0.770
0.667
0.385
0.333

19. A non-Hookian spring has force


F = kx 2 where k is the spring constant
and x is the displacement from its
unstretched position. For the system
shown of a mass m connected to an
unstretched spring initially at rest, how
far does the spring extend before the
system momentarily comes to rest?
Assume that all surfaces are frictionless and that the pulley is frictionless as well.
3mg
(a)

2k
mg
(b)

2mg
(c)

3mg

(d)

3 3mg

(e)

k
2

20. A point-like mass moves horizontally between two walls on a frictionless surface with
initial kinetic energy E. With every collision with the walls, the mass loses its
kinetic energy to thermal energy. How many collisions with the walls are necessary
before the speed of the mass is reduced by a factor of 8?
(a)
(b)
(c)
(d)
(e)

3
4
6
8
16

21. If the rotational inertia of a sphere about an axis through the center of the sphere is I,
what is the rotational inertia of another sphere that has the same density, but has twice
the radius?
(a)
(b)
(c)
(d)
(e)

2I
4I
8I
16I
32I

22. Two rockets are in space in a negligible gravitational field. All observations are made
by an observer in a reference frame in which both rockets are initially at rest. The
masses of the rockets are m and 9m. A constant force F acts on the rocket of mass m
for a distance d. As a result, the rocket acquires a momentum p. If the same constant
force F acts on the rocket of mass 9m for the same distance d, how much momentum
does the rocket of mass 9m acquire?
(a) p/9
(b) p/3
(c) p
(d) 3p
(e) 9p

23. If a planet of radius R spins with an angular velocity about an axis through the
North Pole, what is the ratio of the normal force experienced by a person at the equator
to that experienced by a person at the North Pole? Assume a constant gravitational
field g and that both people are stationary relative to the planet and are at sea level.
(a) g R 2
(b) R 2 g
(c) 1 R 2 g
(d) 1 + g R 2
(e) 1 + R 2 g

24. A ball of mass m is launched into the air. Ignore air resistance, but assume that there is
a wind that exerts a constant force Fo in the x direction. In terms of F0 and the
acceleration due to gravity g, at what angle above the positive x-axis must the ball be
launched in order to come back to the point from which it was launched?
(a) tan 1 ( F0 mg )
(b) tan 1 (mg / F0 )
(c) sin 1 ( F0 mg )
(d) the angle depends on the launch speed
(e) no such angle is possible

25. Find the period of small oscillations of a water pogo, which is a stick of mass m in the
shape of a box (a rectangular parallelepiped.) The stick has a length L, a width w and a
height h and is bobbing up and down in water of density . Assume that the water
pogo is oriented such that the length L and width w are horizontal at all times. Hint:
The buoyant force on an object is given by Fbuoy = Vg , where V is the volume of the
medium displaced by the object and is the density of the medium. Assume that at
equilibrium, the pogo is floating.
L
(a) 2
g
(b)

w2 L2 g
mh 2

(c) 2

mh 2
L2 w2 g

(d) 2

m
wL g

(e)

m
wL g

Questions 26 38: Be sure to show all of your work on the corresponding Free Response
Answer Form as well as to record your answer on the optical mark answer sheet.

26. A sled loaded with children starts from rest and slides down a snowy 250 (with respect
to the horizontal) incline traveling 85 meters in 17 seconds. Ignore air resistance. What
is the coefficient of kinetic friction between the sled and the slope? (5 pts.)
(a)
(b)
(c)
(d)
(e)

0.36
0.40
0.43
1.00
2.01

27. A space station consists of two living modules attached to a central hub on opposite
sides of the hub by long corridors of equal length. Each living module contains N
astronauts of equal mass. The mass of the space station is negligible compared to the
mass of the astronauts, and the size of the central hub and living modules is negligible
compared to the length of the corridors. At the beginning of the day, the space station
is rotating so that the astronauts feel as if they are in a gravitational field of strength g.
Two astronauts, one from each module, climb into the central hub, and the remaining
astronauts now feel a gravitational field of strength g . What is the ratio g g in terms
of N? (5 pts)

(a) 2 N /( N 1)
(b) N ( N 1)
(c)

( N 1) N

(d) N ( N 1)
(e) none of the above

Questions 26 38: Be sure to show all of your work on the corresponding Free Response
Answer Form as well as to record your answer on the optical mark answer sheet.

Questions 28-30

A simplified model of a bicycle of mass M has two tires that each comes into contact
with the ground at a point. The wheelbase of this bicycle (the distance between the
points of contact with the ground) is w, and the center of mass of the bicycle is located
midway between the tires and a height h above the ground. The bicycle is moving to
the right, but slowing down at a constant rate. The acceleration has a magnitude a. Air
resistance may be ignored.

Case 1 (Questions 28 29): Assume that the coefficient of sliding friction between
each tire and the ground is , and that both tires are skidding: sliding without
rotating. Express your answers in terms of w, h, M, and g.
28. What is the maximum value of so that both tires remain in contact with the ground?
(5 pts)
(a)
(b)
(c)
(d)
(e)

w
2h
h
2w
2h
w
w
h
none of the above

10

Questions 26 38: Be sure to show all of your work on the corresponding Free Response
Answer Form as well as to record your answer on the optical mark answer sheet.

29. What is the maximum value of a so that both tires remain in contact with the ground?
(5 pts)
wg
h
wg
(b)
2h
hg
(c)
2w
h
(d)
2wg
(e) none of the above
(a)

Case 2 (Question 30): Assume, instead, that the coefficient of sliding friction between
each tire and the ground is different: 1 for the front tire and 2 for the rear tire. Let
1 = 22

30. Assume that both tires are skidding: sliding without rotating. What is the maximum
value of a so that both tires remain in contact with the ground? (5 pts)
wg
h
wg
(b)
3h
2wg
(c)
3h
hg
(d)
2w
(e) none of the above
(a)

11

Questions 26 38: Be sure to show all of your work on the corresponding Free Response
Answer Form as well as to record your answer on the optical mark answer sheet.

Questions 31 33

A thin, uniform rod has mass m and length L. Let the acceleration due to gravity be g.
Let the rotational inertia of the rod about its center be md 2 .

31. Find the ratio L / d. (3 pts)


(a) 3 2
(b) 3
(c) 12
(d) 2 3
(e) none of the above

The rod is suspended from a point a distance kd from the center, and undergoes small
g
oscillations with an angular frequency
.
d

32. Find an expression for in terms of k. (7 pts)


(a) 1 + k 2
(b) 1 + k 2
k
1+ k

(c)

k2
1+ k
(e) none of the above

(d)

33. Find the maximum value of . (5 pts)


(a)
(b)
(c)
(d)
(e)

1
2
1/ 2
does not attain a maximum value
none of the above

12

Questions 26 38: Be sure to show all of your work on the corresponding Free Response
Answer Form as well as to record your answer on the optical mark answer sheet.

Questions 34 36

A point object of mass m is connected to a cylinder of radius R via a massless rope. At


time t = 0 the object is moving with an initial velocity v0 perpendicular to the rope, the
rope has a length L0, and the rope has a non-zero tension. All motion occurs on a
horizontal frictionless surface. The cylinder remains stationary on the surface and does
not rotate. The object moves in such a way that the rope slowly winds up around the
cylinder. The rope will break when the tension exceeds Tmax. Express your answers in
terms of Tmax, m, L0, R, and v0.

34. What is the angular momentum of the object with respect to the axis of the cylinder at
the instant that the rope breaks? (6 pts)
(a) mv0 R
3

m 2 v0
(b)
Tmax
(c) mv0 L0
Tmax R 2
v0
(e) none of the above

(d)

13

Questions 26 38: Be sure to show all of your work on the corresponding Free Response
Answer Form as well as to record your answer on the optical mark answer sheet.

35.What is the kinetic energy of the object at the instant that the rope breaks? (4 pts)
(a)

mv0
2

mv0 R
(b)
2 L0
2

(c)

mv0 R 2
2L0
2

2
2

mv0 L0
(d)
2R2
(e) none of the above

36. What is the length (not yet wound) of the rope? (6 pts)
(a) L0 R
(b) L0 2R
(c) L0 18 R
2

mv0
Tmax
(e) none of the above
(d)

14

Questions 26 38: Be sure to show all of your work on the corresponding Free Response
Answer Form as well as to record your answer on the optical mark answer sheet.

Questions 37 38

A massless elastic cord (that obeys Hookes Law) will break if the tension in the cord
exceeds Tmax. One end of the cord is attached to a fixed point, the other is attached to
an object of mass 3m. If a second, smaller object of mass m moving at an initial speed
v0 strikes the larger mass and the two stick together, the cord will stretch and break, but
the final kinetic energy of the two masses will be zero. If instead the two collide with a
perfectly elastic one-dimensional collision, the cord will still break, and the larger mass
will move off with a final speed of vf. All motion occurs on a horizontal, frictionless
surface.

37. Find vf/v0. (7 pts)


(a) 1 12
(b) 1

(c) 1

(d) 1 3
(e) none of the above

38. Find the ratio of the total kinetic energy of the system of two masses after the perfectly
elastic collision and the cord has broken to the initial kinetic energy of the smaller mass
prior to the collision. (7 pts)
(a)
(b)
(c)
(d)
(e)

14
1/ 3
1/ 2
3/ 4
none of the above

15

Questions 26 38: Be sure to show all of your work on the corresponding Free Response
Answer Form as well as to record your answer on the optical mark answer sheet.

Multiple Choice Answers


1.
2.
3.
4.
5.
6.
7.
8.
9.
10.
11.
12.
13.
14.
15.
16.
17.
18.
19.
20.
21.
22.
23.
24.
25.
26.
27.
28.
29.
30.
31.
32.
33.
34.
35.
36.
37.
38.

e
b
b
c
a
b
d
b
d
d
e
c
d
a
b
b
a
b
a
c
e
d
c
b
d
b
e
a
b
e
d
e
c
b
a
d
c
d

16

2007 F=ma Contest SOLUTIONS


Multiple Choice Answers
1.
2.
3.
4.
5.
6.
7.
8.
9.
10.
11.
12.
13.
14.
15.
16.
17.
18.
19.
20.
21.
22.
23.
24.
25.
26.
27.
28.
29.
30.
31.
32.
33.
34.
35.
36.
37.
38.

e
b
b
c
a
b
d
b
d
d
e
c
d
a
b
b
a
b
a
c
e
d
c
b
d
b
e
a
b
e
d
e
c
b
a
d
c
d

Copyright 2007, American Association of Physics Teachers

Page 1

Solutions to Free Response


26. Since the acceleration is constant and the sled starts from rest,

x =

1 2
at
2

(26-1)

so a = 0.588 m/s2.
With the y-axis perpendicular to the incline, ay = 0, so the normal force is

N = mg cos

(26-2)

Applying Newtons second law parallel to the incline with f = force of kinetic friction

mg sin f = ma

(26-3)

Using f = N along with equations (26-2) and (26-3) we find that

= tan

a
g cos

0.588m / s 2
= tan 25
= 0.40
(10m / s 2 ) cos 25

27. An accelerated reference frame is equivalent to a gravitational field. We will denote all
quantities that change when the astronauts move with a primed superscript after the move. Due to
circular motion and the fact that the radius does not change and that v = r , we find that
g v 2 2
=
=
g v2 2

(27-1)

Angular momentum is conserved since there is no external torque acting on the system. Therefore,
I = I

(27-2)

Since the corridors are long, we can consider the astronauts to be point masses. So, with r = the
distance from the central hub to the living modules, m = the mass of one astronaut, and with two
Copyright 2007, American Association of Physics Teachers

Page 2

living modules each with N astronauts originally, we find that the rotational inertia before the
astronauts move is

I = 2 Nmr 2

(27-3)

After the two astronauts climb into the central hub,


I = 2( N 1)mr 2

(27-4)

When we substitute (27-3) and (27-4) into (27-2) we obtain


2 Nmr 2 = 2( N 1)mr 2

(27-5)

N
=
N 1

(27-6)

Finally, substituting (27-6) into (27-1), we find

g N
= =

g N 1
2

(27-7)

28. For static equilibrium in an accelerated reference frame, we need to calculate torques about the
center of mass. Let N1 be the normal force on the front tire and N2 the normal force on the rear tire.
Let f1 be the force of friction on the front tire and f2 the force of friction on the rear tire. If the front
tire just barely remains in contact with the ground then N1 = f1 = 0.
Then setting the counter-clockwise torque due to friction on the rear tire = the clockwise torque due to
the normal force on the rear tire, we have
f2h = N2

w
2

(28-1)

Substituting f 2 = N 2 into (28-1),

N2h = N2

w
2

Copyright 2007, American Association of Physics Teachers

(28-2)

Page 3

w
2h

(28-3)

29 30. Applying Newtons Second Law to the horizontal direction,


f1 + f 2 = Ma

(29-1)

Setting clockwise torque = counterclockwise torques:


N 2 w N1w
=
+ ( f1 + f 2 )h
2
2

(29-2)

Substituting (28-4) into (28-5) and solving for a,


Mah =

a=

w
( N 2 N1 )
2

w ( N 2 N1 )
Mh
2

(29-3)

(29-4)

The maximum acceleration will clearly occur when N1=0. In that case, N2 = Mg, and

a=

wg
2 h

(29-5)

(This is the answer to question 29. Note that this answer did not depend at all upon whether the
coefficient of sliding friction for each tire and the ground is the same or different. Therefore, this is
the answer to question 30 also.)
31. The rotational inertia of a thin, uniform rod about its center is
I cm =

1
mL2
12

(31-1)

We are given that the rotational inertia of the rod about its center is md2. Setting this expression equal
to (31-1), we obtain
md 2 =

1
mL2
12

Copyright 2007, American Association of Physics Teachers

(31-2)

Page 4

Therefore,
L2 = 12d 2

(31-3)

L
= 12 = 2 3
d

(31-4)

and

32. The torque due to gravity is the same as if the entire mass were located at the center of mass.
Therefore, the gravitational torque on the rod about an axis through the suspension point a distance kd
from the center when the rod making an angle to the vertical is

p = mgkd sin

(32-1)

where the subscript p denotes the pivot point. We now need the parallel axis theorem to find the
rotational inertia about the pivot point.
I p = I cm + mh 2 = md 2 + m(kd ) 2

(32-2)

I p = md 2 (1 + k 2 )

(32-3)

Now, apply the rotational analogue of Newtons Second Law to the axis through the pivot point.
Noting that the force of the pivot does not exert a torque about an axis through the pivot and using
equations (32-1) and (32-3), we find

p = I p
mgkd sin = md 2 (1 + k 2 )

(32-4)
d 2
dt 2

(32-5)

For small oscillations, sin . Therefore,


d 2
gk
=

2
dt
d (1 + k 2 )

Copyright 2007, American Association of Physics Teachers

(32-6)

Page 5

Since an object oscillates with angular frequency when the objects motion is governed by the
differential equation
d 2
= 2
2
dt

(32-7)

we find that
gk
k
=
2
d (1 + k )
1+ k 2

g
d

(32-8)

g
d

(32-9)

k
1+ k 2

(32-10)

=
where

33. To find the value of k that gives the maximum value of , square equation (32-10) and then
differentiate both sides with respect to k.
2

d (1 + k 2 ) k (2k )
1 k 2
=
=
dk
(1 + k 2 ) 2
(1 + k 2 ) 2

d
= 0 when k = 1
dk

(33-1)

(33-2)

Substituting k = 1 into (32-10), we find that

1
2

(33-3)

34 36. Since the velocity is perpendicular to the rope, the rope does not do any work on the object.
Since the object is moving on a horizontal frictionless surface, the net work done on the object is zero
and therefore the change in kinetic energy of the object is zero.
Copyright 2007, American Association of Physics Teachers

Page 6

Thus, the kinetic energy of the object at the instant that the rope breaks is the same as the initial
kinetic energy of the object:
K=

mv02
2

(34-1)

(This is the answer to #35.)


Therefore, the speed of the object is always v0. The angular momentum of the object with respect to
the axis of the cylinder is
L = mv0 r

(34-2)

where r is the radius of the circular orbit (which is the length of the not yet wound rope.)
At the time that the rope breaks, the tension is
Tmax =

mv02
r

(34-3)

Solve equation (34-3) for r


mv02
r =
Tmax

(34-4)

(This is the answer to #36.)


Substitute (34-4) into (34-2).
L=

m 2 v03
Tmax

(34-4)

(This is the answer to #34.)

Copyright 2007, American Association of Physics Teachers

Page 7

37 38. The cord breaks when it has exceeded a certain tension, which happens when it exceeds a
certain potential energy, U0. For the inelastic collision, when the cord is slack, we use conservation of
momentum
mv0 = 4mv

(37-1)

v0
4

(37-2)

v =

The kinetic energy of the two masses immediately after the collision is
K1 =

mv02
8

(37-3)

All of this kinetic energy gets transferred into potential energy so we know that the cord breaks when
U 0 = K1 =

mv02
8

(37-4)

Now for the elastic collision: First, find the velocities of each mass immediately after the collision
while the cord is slack. The easy way to do this is to find that the velocity of the center of mass is

vcm =

v0
4

(37-5)

Then in the center of mass reference frame, before the collision the velocity of m is

3v0
and the
4

v0
. In a one-dimensional elastic collision, in the center of mass reference frame,
4
each blocks velocity after the collision is the same magnitude, but in the opposite direction, of its
velocity before the collision. So the velocity of the 3m object right after the collision in the center of
+v
mass reference frame is 0 . Using (37-5) to transform back to the lab reference frame, we find that
4
v
the velocity of the 3m object immediately after the collision is 0 and therefore its kinetic energy
2
immediately after the collision (while the cord is still slack) is

velocity of 3m is

Copyright 2007, American Association of Physics Teachers

Page 8

3mv02
K2 =
8

(37-6)

But since we know that the cord breaks, we know that U0 of the kinetic energy of the 3m block will be
consumed by the cord. Therefore, the final kinetic energy of the 3m block, using conservation of
energy along with equations (37-4) and (37-6) is
K 3 = K 2 K1 =

3mv02 mv02 mv02

=
8
8
4

(37-7)

Now we can find that

K3 =
vf
v0

3mv 2f
2

mv02
4

1
6

(37-8)

(37-9)

The velocity of the object of mass m in the center of mass reference frame immediately after the
3v0
collision was
. Transforming back to the lab reference frame, we find that the mass m has a
4
v
velocity after the collision of 0 . Therefore, the kinetic energy of m after the elastic collision is
2
K4 =

mv02
8

(37-10)

The total kinetic energy of the system after the elastic collision and the cord is broken, using (37-7)
and (37-10) is
K3 + K 4 =

mv02 mv02 3mv02


+
=
8
4
8

(37-11)

So, the ratio of the total kinetic energy of the system after the elastic collision and the cord is broken
to the initial kinetic energy of the smaller mass prior to the collision is

Copyright 2007, American Association of Physics Teachers

Page 9

3mv02
8 =3
mv02
4
2

Copyright 2007, American Association of Physics Teachers


10

(37-12)

Page

United States Physics Team


Semi Final Contest
2007

2007 Semi-Final Exam


INSTRUCTIONS
DO NOT OPEN THIS TEST UNTIL YOU ARE TOLD TO BEGIN

Work Part A first. You have 90 minutes to complete all four problems.
After you have completed Part A, you may take a break.
Then work Part B. You have 90 minutes to complete both problems.
Show all your work. Partial credit will be given.
Start each question on a new sheet of paper. Be sure to put your name in the upper righthand corner of each page, along with the question number and the page number/total pages
for this problem. For example,
Doe, Jamie
A1 1/3
A hand-held calculator may be used. Its memory must be cleared of data and programs. You
may use only the basic functions found on a simple scientific calculator. Calculators may not
be shared. Cell phones, PDAs, or cameras may not be used during the exam or while the
exam papers are present. You may not use any tables, books, or collections of formulas.
Questions with the same point value are not necessarily of the same difficulty.
Do not discuss the contents of this exam with anyone until after March 27th.
Good luck!
Possibly Useful Information - (Use for both part A and for part B)
Gravitational field at the Earths surface g = 9.8 N/kg
Newtons gravitational constant
G = 6.67 x 10-11 Nm2/kg2
Coulombs constant
k = 1/4 = 8.99 x 109 Nm2/C2
Biot-Savart constant
km = /4 = 10-7 Tm/A
Speed of light in a vacuum
c = 3.00 x 108 m/s
Boltzmanns constant
kB = 1.38 x 10-23 J/K
Avogadros number
NA = 6.02 x 1023 (mol)-1
Ideal gas constant
R = NAkB = 8.31 J/(molK)
Stefan-Boltzmann constant
= 5.67 x 10-8 J/(sm2K4)
Elementary charge
e = 1.602 x 10-19 C
1 electron volt
1 eV = 1.602 x 10-19 J
Plancks constant
h = 6.63 x 10-34 Js = 4.14 x 10-15 eVs
Electron mass
m = 9.109 x 10-31 kg = 0.511 MeV/c2
Binomial expansion
(1 + x)n 1 + nx
for |x| << 1
Small angle approximations
sin
cos 1 1/2 2

Copyright 2007, American Association of Physics Teachers

Semi-Final Exam
Part A
A1. A group of 12 resistors is arranged along the edges of a cube as shown in the diagram below.

The vertices of the cube are labeled a-h.

a. (13 pts) The resistance between each pair of vertices is as follows:


Rab = Rac = Rae = 3.0
Rcg = Ref = Rbd = 8.0
Rcd = Rbf = Reg = 12.0
Rdh = Rfh = Rgh = 1.0
What is the equivalent resistance between points a and h?
b. (12 pts) The three 12.0 resistors are replaced by identical capacitors. Ccd = Cbf = Ceg =
15.0 F. A 12.0 V battery is attached across points a and h and the circuit is allowed to
operate for a long period of time. What is the charge (Qcd, Qbf, Qeg) on each capacitor
after this long period of time?
Copyright 2007, American Association of Physics Teachers

A2. A simple gun can be made from a uniform cylinder of length L0 and inside radius rc. One
end of the cylinder is sealed with a moveable plunger and the other end is plugged with a
cylindrical cork bullet. The bullet is held in place by friction with the walls of the cylinder.
The pressure outside the cylinder is atmospheric pressure, P0 . The bullet will just start to slide
out of the cylinder if the pressure inside the cylinder exceeds Pcr .
a. There are two ways to launch the bullet: either by heating the gas inside the cylinder and
keeping the plunger fixed, or by suddenly pushing the plunger into the cylinder. In either case,
assume that an ideal monatomic gas is inside the cylinder, and that originally the gas is at
temperature T0 , the pressure inside the cylinder is P0 , and the length of the cylinder is L0 .
(8 pts)

i. Assume that we launch the bullet by heating the gas without moving the plunger.
Find the minimum temperature of the gas necessary to launch the bullet. Express
your answer in terms of any or all of the variables: rc , T0 , L0 , P0 , and Pcr .

(8 pts)

ii. Assume, instead that we launch the bullet by pushing in the plunger, and that we
do so quickly enough so that no heat is transferred into or out of the gas. Find the
length of the gas column inside the cylinder when the bullet just starts to move.
Express your answer in terms of any or all of the variables: rc , T0 , L0 , P0 , and Pcr .

b. (9 pts) It is necessary to squeeze the bullet to get it into the cylinder in the first place. The
bullet normally has a radius rb that is slightly larger than the inside radius of the cylinder;
rb rc = r , is small compared to rc . The bullet has a length h  L0 . The walls of the cylinder
apply a pressure to the cork bullet. When a pressure P is applied to the bullet along a given
direction, the bullets dimensions in that direction change by
x P
=
x
E
for a constant E known as Youngs modulus. You may assume that compression along one
direction does not cause expansion in any other direction. (This is true if the so-called Poisson
ratio is close to zero, which is the case for cork.)
If the coefficient of static friction between the cork and the cylinder is , find an expression
for Pcr . Express your answer in terms of any or all of the variables: P0 , , h, E , r , and rc .

Copyright 2007, American Association of Physics Teachers

A3. A volume V f of fluid with uniform charge density is sprayed into a room, forming
spherical drops. As they float around the room, the drops may break apart into smaller drops
or coalesce into larger ones. Suppose that all of the drops have radius R. Ignore inter-drop
forces and assume that V f  R 3 .
(10 pts) a. Calculate the electrostatic potential energy of a single drop. (Hint: suppose the
sphere has radius r. How much work is required to increase the radius by dr?).
(4 pts) b. What is the total electrostatic energy of the drops?
Your answer to (b) should indicate that the total energy increases with R. In the absence of
surface tension, then, the fluid would break apart into infinitesimally small drops. Suppose,
however, that the fluid has a surface tension . (This value is the potential energy per unit
surface area, and is positive.)
(4 pts) c.

What is the total energy of the drops due to surface tension?

(7 pts) d.

What is the equilibrium radius of the drops?

Copyright 2007, American Association of Physics Teachers

A4. A nonlinear circuit element can be made out of a parallel plate capacitor and small balls,
each of mass m, that can move between the plates. The balls collide inelastically with the
plates, dissipate all kinetic energy as thermal energy, and immediately release the charge they
are carrying to the plate. Almost instantaneously, the balls then pick up a small charge of
magnitude q from the plate; the balls are then repelled directly toward the other plate under
electrostatic forces only. Another collision happens, kinetic energy is dissipated, the balls give
up the charge, collect a new charge, and the cycle repeats. There are n0 balls per unit surface
area of the plate. The capacitor has a capacitance C. The separation d between the plates is
much larger than the radius r of the balls. A battery is connected to the plates in order to
maintain a constant potential difference V. Neglect edge effects and assume that magnetic
forces and gravitational forces may be ignored.
(5)

a. Determine the time it takes for one ball to travel between the plates in terms of any or
all of the following variables: m, q, d, and V.

(5)

b. Calculate the kinetic energy dissipated as thermal energy when one ball collides
inelastically with a plate surface in terms of any or all of the following variables: m,
q, d, and V.

(5)

c. Derive an expression for the current between the plates in terms of the permittivity of
free space, 0 , and any or all of the following variables: m, q, n0, C, and V.

(5)

d. Derive an expression for the effective resistance of the device in terms of 0 , and any
or all of the following variables: m, q, n0, C, and V.

(5)

e. Calculate the rate at which the kinetic energy of the balls is converted into thermal
energy in terms of 0 , and any or all of the following variables: m, q, n0, C, and V.

Copyright 2007, American Association of Physics Teachers

Semi-Final Exam
Part B
B1. A certain mechanical oscillator can be modeled as an ideal massless spring connected to a
moveable plate on an incline. The spring has spring constant k, the plate has mass m, and the
incline makes an angle with the horizontal. When the system is operating correctly, the
plate oscillates between points A and B in the figure, located a distance L apart. When the plate
reaches point A it has zero kinetic energy, but then trips a small lever that instantaneously loads
a block of mass M onto the plate. The block and plate then move down the incline to point B,
where the force from the spring stops the plate. At this point, the block falls through a hole in
the incline, allowing the plate to move back up under the force of the spring. Upon returning
to point A it collects another block, and the cycle repeats. Both the plate and the block have a
coefficient of friction with the incline for both kinetic and static friction. It is reasonable that
the motion in either direction is simple harmonic in nature.

(10 pts) a. Let c be the critical value of the coefficient of friction where the block will just
start to slide under the force of gravity on an incline (without the spring acting on
it). Then let =

. Find in terms of g, the acceleration of free fall, and any


2
or all of the following variables: and M.

Copyright 2007, American Association of Physics Teachers

(14 pts) b. In order for this system to work correctly, it is necessary to have the correct ratio
between the mass of the block and the mass of the plate. These masses are chosen
so that the downward moving block and plate just stop at point B while the
M
upward moving plate just stops at point A. Find the ratio R = .
m
(13 pts) c. The system delivers blocks to point B with period T0 , until the blocks run out.
T
After that, the plate alone oscillates with a period T . Find the ratio 0 .
T
(13 pts) d. The plate only oscillates a few times after delivering the last block. At what
distance up the incline, measured from point B, does the plate come to a
permanent stop?

Copyright 2007, American Association of Physics Teachers

B2. A model of the magnetic properties of materials is based upon small magnetic moments
generated by each atom in the material. One source of this magnetic moment is the magnetic
field generated by the electron in its orbit around the nucleus. For simplicity, we will assume
that each atom consists of a single electron of charge e and mass me , a single proton of charge
+e and mass m p  me , and that the electron orbits in a circular orbit of radius R about the
proton.
a. Magnetic Moments.
Assume that the electron orbits in the x-y plane.
(3 pts) i.

Calculate the net electrostatic force on the electron from the proton. Express your
answer in terms of any or all of the following parameters: e, me , m p , R, and the

permittivity of free space, 0 , where


1
0 =
.
4 k
(k is the Coulombs Law constant).
(5 pts) ii.

Determine the angular velocity 0 of the electron around the proton in terms of
any or all of the following parameters: e, me , R, and 0 .

(8 pts) iii. Derive an expression for the magnitude of the magnetic field Be due to the orbital
motion of the electron at a distance z  R from the x-y plane along the axis of
orbital rotation of the electron. Express your answer in terms of any or all of the
following parameters: e, me , R, 0 , z , and the permeability of free space 0 .
(4 pts) iv. A small bar magnet has a magnetic field far from the magnet given by
m
B= 0 3,
2 z
where z is the distance from the magnet on the axis connecting the north and
south poles, m is the magnetic dipole moment, and 0 is the permeability of free
space. Assuming that an electron orbiting a proton acts like a small bar magnet,
find the dipole moment m for an electron orbiting an atom in terms of any or all of
the following parameters: e, me , R, and 0 .

Copyright 2007, American Association of Physics Teachers

b. Diamagnetism.
We model a diamagnetic substance to have all atoms oriented so that the electron orbits are in
the x-y plane, exactly half of which are clockwise and half counterclockwise when viewed
from the positive z axis looking toward the origin. Some substances are predominantly
diamagnetic.
(3 pts) i.

(6 pts) ii.

Calculate the total magnetic moment of a diamagnetic substance with N atoms.


Write your answer in terms of any or all of the following parameters:
e, me , R, N , and 0 .
G
An external magnetic field B0 = B0 z is applied to the substance. Assume that the
introduction of the external field doesnt change the fact that the electron moves
in a circular orbit of radius R. Determine , the change in angular velocity of
the electron, for both the clockwise and counterclockwise orbits. Throughout this
entire problem you can assume that  0 . Write your answer in terms of
e, me , and B0 only.

(6 pts) iii. Assume that the external field is turned on at a constant rate in a time interval t .
That is to say, when t = 0 the external field is zero and when t = t the external
G
field is B0 . Determine the induced emf E experienced by the electron. Write your
answer in terms of any or all of the following parameters: e, me , R, N , B0 , 0 , and
0 .
(6 pts) iv. Verify that the change in the kinetic energy of the electron satisfies K = e E.
This justifies our assumption in (ii) that R does not change.
(6 pts) v.

Determine the change in the total magnetic moment m for the N atoms when the
external field is applied, writing your answer in terms of e, me , R, N , 0 and B0 .

(3 pts) vi. Suppose that the uniform magnetic field used in the previous parts of this problem
is replaced with a bar magnet. Would the diamagnetic substance be attracted or
repelled by the bar magnet? How does your answer show this?

Copyright 2007, American Association of Physics Teachers

2007 Semifinal Exam Solutions

AAPT
AIP

UNITED STATES PHYSICS TEAM


2007

Solutions to Problems
Part A
Question 1
a. There is a high degree of symmetry present. Points b, c, and e are at the same potential;
similarly, points d, f , and g are at the same potential. The circuit then reduces to a series
connection of three parallel resistor clusters.
The three parallel clusters have effective resistances of 1 , 8/5 , and 1/3 . The effective
resistance of the circuit is then 44/15 .
b. After a long time no current will flow through the branches of the circuits containing capacitors. The circuit then reduces to a parallel connection of three series resistor clusters.
The effective resistance of the circuit is 4 , the current through the circuit is then
(12 V)/(4 ) = 3 A.

(A1-1)

The three branches are identical; each then carries 1 A.


The potential drop across each capacitor is the same as the potential drop across the 8
resistors, so
VC = (1 A)(8 ) = 8 V.
(A1-2)
Finally, the charge on each capacitor is
Q = (15 F)(8 V) = 120 C.

(A1-3)

Question 2
a. Two parts, solved individually.
i. Isochoric compression:
Pf
Tf
=
,
Pi
Ti
so
T =

Pcr
T0 .
P0

c
Copyright 2007
American Association of Physics Teachers

(A2-1)

(A2-2)

2007 Semifinal Exam Solutions

ii. Adiabatic compression:


P V = const,

(A2-3)

where = CP /CV = (CV + 1)/CV = 5/3 for a monatomic gas. Consequently


P0 L0 = Pcr L ,
and then


L = L0

P0
Pcr

(A2-4)

5/3

(A2-5)

b. The normal pressure on the bullet comes from


r
E.
rc

(A2-6)

r
E (2rc h),
rc

(A2-7)

P =
Therefore, the normal force on the bullet is
FN =

and finally the force of friction is FN . The force due to the pressure difference between the
inside of the barrel and the outside must equal the normal force, so
(rc2 )(Pcr P0 ) = 2h r E,
and then
Pcr = P0 +

2Eh
r.
rc2

(A2-8)

(A2-9)

Question 3
a. If the sphere has radius r, it has charge
4
q = r3
3

(A3-1)

and thus its surface is at electrostatic potential


V =

q
r2
=
40 r
30

(A3-2)

To increase the radius by dr, an additional charge dq = 4r2 dr must be brought in from
infinity, requiring work
4r4 2
dU = V dq =
dr
(A3-3)
30
Thus to grow the sphere from r = 0 to r = R requires
U=

Z R
4r4 2
0

30

dr =

4R5 2
150

c
Copyright 2007
American Association of Physics Teachers

(A3-4)

2007 Semifinal Exam Solutions

b. Each drop has volume Vd = 43 R3 , so the number of drops is


n=

Vf
Vf
= 4 3
Vd
3 R

(A3-5)

Since we are ignoring inter-drop forces, the total energy of the drops is simply the sum of the
energies of each individual drop:
Ue,tot = nU =

Vf
4
3
3 R

R 2 2
4R5 2
=
Vf
150
50

(A3-6)

c. Each drop has surface area 4R2 and thus surface tension energy 4R2 . As before, the total
energy due to surface tension is just the sum of the energies of the individual drops:
Vf
3
Vf
=
3
R
R
3

Us,tot = 4R2 n = 4R2 4

(A3-7)

d. The total potential energy from both sources is


Utot =

R2 2 3
+
50
R

Vf

(A3-8)

Equilibrium is reached when the total energy is a minimum; since U at both R 0


and R , it must have an interior minimum.
d
Utot =
dR

3
2R2
2
50
R

Vf

(A3-9)

Setting this equal to zero,


2R2
3
= 2
50
R
150
R3 =
22


R=

150
22

(A3-10)
(A3-11)

1

(A3-12)

Question 4
a. The electric field between the plates is given by E = V /d. The force on the charged ball is
then F = Eq = V q/d. The acceleration of the ball is a = V q/md.
Kinematics gives us d = at2 /2 for the time of flight. So
t=

2d/a =

2md2 /qV .

(A4-1)

b. The kinetic energy collected by a ball will be K = qV as it moves between the plates. Thats
what will be dissipated.

c
Copyright 2007
American Association of Physics Teachers

2007 Semifinal Exam Solutions

c. The current is given by I = Q/t. The total number of balls is N = n0 A, where A is the
surface area of a plate. The charge Q is then Q = n0 qA, so the current is
I=

n0 qA
Q
=p
.
t
2md2 /qV

(A4-2)

We cant stop here, since this is not in terms of the allowed variables. The problem is A and
d, but since C = 0 A/d, we have
I =

n0 qA
,
2md2 /qV

A
n0 q
d

C
n0 q
0

(A4-3)

qV
,
2m

(A4-4)

qV
.
2m

(A4-5)

d. R = V /I, so
0 V
V
=
R=
I
Cn0 q

2m
.
qV

(A4-6)

We can simplify, slightly, with


0
R=
Cn0 q

2mV
.
q

(A4-7)

e. P = V I, so
P =V

C
n0 q
0

qV
=
2m

0 2 n0 2 C 2 q 3 V 3
.
2m

(A4-8)

Part B
Question 1
a. To not slip, from a free-body diagram, we must have
mg cos mg sin

(B1-1)

tan .

(B1-2)

tan
.
2

(B1-3)

so
Therefore c = tan and hence
=

b. In one cycle the energy input into the system is


M gL sin ,
the energy of the block dropping.

c
Copyright 2007
American Association of Physics Teachers

(B1-4)

2007 Semifinal Exam Solutions

The energy loss on the way up is


Lmg cos

(B1-5)

L(m + M )g cos

(B1-6)

M gL sin = Lmg cos + L(m + M )g cos

(B1-7)

and the energy loss on the way down is

Thus
and since 2 cos = sin ,
M

m m+M
+
,
2
2
= 2m,
=

R = M/m = 2.

(B1-8)
(B1-9)
(B1-10)

c. The period of a mass m oscillating on a spring of spring constant k is


r

T = 2

m
.
k

(B1-11)

In this case, the friction force is constant on both the up and down trips, and so each trip is
simple harmonic (with different equilibrium points). Hence
r

T0
T0
T0 /T 0

m
+
=
k
r
m
,
= 2
k

1+ 3
.
=
2

3m
,
k

(B1-12)
(B1-13)
(B1-14)

d. As mentioned in part (c), both the up and down trips are simple harmonic, this time with
a mass of m both ways. The equilibrium points for the two trips are different, however. On
the up trip, the equilibrium point is clearly at a distance L/2 from B, since the plate stops
at both B and A and hence those are the endpoints of the oscillation and the equilibrium is
halfway between. For the trip down, the equilibrium point will shift by a distance y such that
ky = 2mg cos = mg sin

(B1-15)

because 2mg cos is the difference between the friction forces on the trip up and the trip
down.
The place where the plate finally comes to a stop is the first place that is at the end of an
oscillation (either up or down) and where the total force being exerted by gravity and the
spring is less than the maximal force of friction. For that to happen, the plate needs to not
have gone past the other equilibrium point during that oscillation.
So we start by determining where the endpoints of the oscillations are. For the first trip up
these are B and A. For the following trip down, the plate stops at a distance of (2mg sin )/k
from B (because the equilibrium shifts up by (mg sin )/k. For the following trip up, the
c
Copyright 2007
American Association of Physics Teachers

2007 Semifinal Exam Solutions

plate stops a distance L (2mg sin )/k from B, since the equilibrium point is again in the
middle of the incline. And so forth.
Thus the stopping points are located at
n(2mg sin )/k and L n(2mg sin )/k

(B1-16)

for integer n. The plate will stop permanently once either


n(2mgsin)/k > L/2

(B1-17)

L n(2mg sin )/k < L/2 + (mg sin )/k,

(B1-18)

or
whichever happens first. (The first condition corresponds to going down and ending up above
the midpoint at the end of the down trip, the second condition corresponds to going up and
stopping below the upper equilibrium.) The second condition can be rewritten as
1
(2mg sin )/k > L/2.
2


n+

(B1-19)

Question 2
a. Magnetic Moments
i. From Coulombs Law,
F =

e2
40 R2

(B2-1)

ii. For circular motion,


me v 2
= me R02 ,
R
The force is provided by the Coulomb force, so
F =

e2
,
40 R2

me R02 =

0 =

e2
40 me R3

(B2-2)

(B2-3)
(B2-4)

iii. From the law of Biot and Savart,


~e =
B
Be =

0 i d~s ~r
,
4
r3
R
0 i
2R 2
,
4
(z + r2 )3/2
0 iR2
.
2z 3
I

For the current, i, we can write


i=
Then
Be =

q
e0
=
.
t
2
0 e0 R2
.
4z 3

c
Copyright 2007
American Association of Physics Teachers

(B2-5)
(B2-6)
(B2-7)

(B2-8)

(B2-9)

2007 Semifinal Exam Solutions

iv. By substitution,
m=

e0 R
.
2

(B2-10)

b. Diamagnetism
i. If half go one way and half go the other, M = 0.
ii. Additional force from magnetism,
FB = qvB0 = eRB0

(B2-11)

modifies previous central force problem to give


me R 2 =

e2
eR0 B0 ,
40 R2

(B2-12)

where the positive sign corresponds to anticlockwise motion, the negative to clockwise
motion.
A little math,
me R( 2 02 ) = eRB0 ,
me ( 0 )( + 0 ) = eB0 ,
me ()(20 ) = e0 B0 ,

(B2-13)
(B2-14)
(B2-15)

where in the last line we have used the approximation 0 . Then


=

eB0
.
2me

(B2-16)

iii. The emf is given by


n

=
,
(B2-17)
t
t
but n/t is a measure of the number of turns made by the electron in a time interval
t, so
0 R
0
n
=
=
.
(B2-18)
t
2R
2
Then
1
0
B0 R2 = 0 b0 R2 .
(B2-19)
E=
2
2
E =n

iv. The change in kinetic energy is given by


1
me 2 R 2 ,
2
= me R2 ,


K =

me R 0 ,


eB0
2
,
= me 0 R
2me
= eE.

c
Copyright 2007
American Association of Physics Teachers

(B2-20)
(B2-21)
(B2-22)
(B2-23)
(B2-24)

2007 Semifinal Exam Solutions

v. M = N m, where N is the number of atoms, and m the change in magnetic moment


in each. The change is
e0 R
.
m =
2
eR
,
=
2
e2 R2 B0
=
,
4me


so
M = N

e2 R2 B0
.
4me

vi. Repelled, by Lenzs law.

c
Copyright 2007
American Association of Physics Teachers

(B2-25)
(B2-26)
(B2-27)

(B2-28)

United States Physics Team


F = ma Contest
2008

2008 F = ma Exam

AAPT
AIP

UNITED STATES PHYSICS TEAM


2008

2008 F = ma Contest
25 QUESTIONS - 75 MINUTES
INSTRUCTIONS
DO NOT OPEN THIS TEST UNTIL YOU ARE TOLD TO BEGIN
Use g = 10 N/kg throughout this contest.
You may write in this booklet of questions. However, you will not receive any credit for
anything written in this booklet.
Your answer to each question must be marked on the optical mark answer sheet.
Select the single answer that provides the best response to each question. Please be sure to
use a No. 2 pencil and completely fill the box corresponding to your choice. If you change an
answer, the previous mark must be completely erased.
Correct answers will be awarded one point; incorrect answers will result in a deduction of
point. There is no penalty for leaving an answer blank.

1
4

A hand-held calculator may be used. Its memory must be cleared of data and programs. You
may use only the basic functions found on a simple scientific calculator. Calculators may
not be shared. Cell phones may not be used during the exam or while the exam papers are
present. You may not use any tables, books, or collections of formulas.
This test contains 25 multiple choice questions. Your answer to each question must be marked
on the optical mark answer sheet that accompanies the test. Only the boxes preceded by
numbers 1 through 25 are to be used on the answer sheet.
All questions are equally weighted, but are not necessarily the same level of difficulty.
In order to maintain exam security, do not communicate any information about
the questions (or their answers or solutions) on this contest until after February
20, 2008.
The question booklet and answer sheet will be collected at the end of this exam. You may
not use scratch paper.

DO NOT OPEN THIS TEST UNTIL YOU ARE TOLD TO BEGIN

c
Copyright 2008
American Association of Physics Teachers

2008 F = ma Exam

1. A bird flying in a straight line, initially at 10 m/s, uniformly increases its speed to 18 m/s while
covering a distance of 40 m. What is the magnitude of the acceleration of the bird?
(a) 0.1 m/s2
(b) 0.2 m/s2
(c) 2.0 m/s2
(d) 2.8 m/s2
(e) 5.6 m/s2
2. A cockroach is crawling along the walls inside a cubical room that has an edge length of 3 m. If the
cockroach starts from the back lower left hand corner of the cube and finishes at the front upper
right hand corner, what is the magnitude of the displacement of the cockroach?

(a) 3 2 m

(b) 3 3 2 m

(c) 3 3 m
(d) 3 m
(e) 9 m
3. The position vs. time graph for an object moving in a straight line is shown below. What is the
instantaneous velocity at t = 2 s?

Position (m)

4
2
Time (s)
0
1

(a) 2 m/s

(b) 12 m/s
(c) 0 m/s

(d) 2 m/s
(e) 4 m/s

c
Copyright 2008
American Association of Physics Teachers

2008 F = ma Exam

The information below is for the next two problems


Shown below is the velocity vs. time graph for a toy car moving along a straight line.

Velocity (m/s)

4
2
Time (s)
0
1

4. What is the maximum displacement from start for the toy car?
(a) 3 m
(b) 5 m
(c) 6.5 m
(d) 7 m
(e) 7.5 m

Time (s)
0
1

Acceleration

Acceleration

5. Which of the following acceleration vs. time graphs most closely represents the acceleration of the
toy car?

Time (s)
0
1

Time (s)
0
1

(c)
Acceleration

(b)
Acceleration

Acceleration

(a)

Time (s)
0
1

(d)

Time (s)
0
1

(e)

c
Copyright 2008
American Association of Physics Teachers

2008 F = ma Exam

6. A cannon fires projectiles on a flat range at a fixed speed but with variable angle. The maximum
range of the cannon is L. What is the range of the cannon when it fires at an angle 6 above the
horizontal? Ignore air resistance.
(a)
(b)
(c)
(d)
(e)

3
2 L
1 L
2
1
L
3
1
2L
1
3L

7. A toboggan sled is traveling at 2.0 m/s across the snow. The sled and its riders have a combined
mass of 120 kg. Another child (mchild = 40 kg) headed in the opposite direction jumps on the sled
from the front. She has a speed of 5.0 m/s immediately before she lands on the sled. What is the
new speed of the sled? Neglect any effects of friction.
(a) 0.25 m/s
(b) 0.33 m/s
(c) 2.75 m/s
(d) 3.04 m/s
(e) 3.67 m/s
8. Riders in a carnival ride stand with their backs against the wall of a circular room of diameter
8.0 m. The room is spinning horizontally about an axis through its center at a rate of 45 rev/min
when the floor drops so that it no longer provides any support for the riders. What is the minimum
coefficient of static friction between the wall and the rider required so that the rider does not slide
down the wall?
(a) 0.0012
(b) 0.056
(c) 0.11
(d) 0.53
(e) 8.9

c
Copyright 2008
American Association of Physics Teachers

2008 F = ma Exam

9. A ball of mass m1 travels along the x-axis in the positive direction with an initial speed of v0 . It
collides with a ball of mass m2 that is originally at rest. After the collision, the ball of mass m1
has velocity v1x x
+ v1y y and the ball of mass m2 has velocity v2x x
+ v2y y.
Consider the following five statements:
I)
II)
III)
IV)
V)

0 = m1 v1x + m1 v2x
m1 v0 = m1 v1y + m2 v2y
0 = m1 v1y + m2 v2y
m1 v0 = m1 v1x + m1 v1y
m1 v0 = m1 v1x + m2 v2x

Of these five statements, the system must satisfy


(a) I and II
(b) III and V
(c) II and V
(d) III and IV
(e) I and III

The following information applies to the next two problems


An experiment consists of pulling a heavy wooden block across a level surface with a spring force
meter. The constant force for each try is recorded, as is the acceleration of the block. The data
are shown below.
Force F in Newtons
acceleration a in meters/second2

3.05
0.095

3.45
0.205

4.05
0.295

4.45
0.405

5.05
0.495

10. Which is the best value for the mass of the block?
(a) 3 kg
(b) 5 kg
(c) 10 kg
(d) 20 kg
(e) 30 kg
11. Which is the best value for the coefficient of friction between the block and the surface?
(a) 0.05
(b) 0.07
(c) 0.09
(d) 0.5
(e) 0.6

c
Copyright 2008
American Association of Physics Teachers

2008 F = ma Exam

12. A uniform disk rotates at a fixed angular velocity on an axis through its center normal to the plane
of the disk, and has kinetic energy E. If the same disk rotates at the same angular velocity about
an axis on the edge of the disk (still normal to the plane of the disk), what is its kinetic energy?
(a)
(b)

1
2E
3
2E

(c) 2E
(d) 3E
(e) 4E
13. A mass is attached to the wall by a spring of constant k. When the spring is at its natural length,
the mass is given a certain initial velocity, resulting in oscillations of amplitude A. If the spring
is replaced by a spring of constant 2k, and the mass is given the same initial velocity, what is the
amplitude of the resulting oscillation?
(a)
(b)
(c)

1
2A
1 A
2

2A

(d) 2A
(e) 4A
14. A spaceborne energy storage device consists of two equal masses connected by a tether and rotating
about their center of mass. Additional energy is stored by reeling in the tether; no external forces
are applied. Initially the device has kinetic energy E and rotates at angular velocity . Energy is
added until the device rotates at angular velocity 2. What is the new kinetic energy of the device?

(a) 2E
(b) 2E

(c) 2 2E
(d) 4E
(e) 8E

c
Copyright 2008
American Association of Physics Teachers

2008 F = ma Exam

15. A uniform round tabletop of diameter 4.0 m and mass 50.0 kg rests on massless, evenly spaced legs
of length 1.0 m and spacing 3.0 m. A carpenter sits on the edge of the table. What is the maximum
mass of the carpenter such that the table remains upright? Assume that the force exerted by the
carpenter on the table is vertical and at the edge of the table.

1.0 m

3.0 m

4.0 m

(a) 67 kg
(b) 75 kg
(c) 81 kg
(d) 150 kg
(e) 350 kg
16. A massless spring with spring constant k is vertically mounted so that bottom end is firmly attached
to the ground, and the top end free. A ball with mass m falls vertically down on the top end of
the spring, becoming attached, so that the ball oscillates vertically on the spring. What equation
describes the acceleration a of the ball when it is at a height y above the original position of the
top end of the spring? Let down be negative, and neglect air resistance; g is the magnitude of the
acceleration of free fall.
(a) a = mv 2 /y + g
(b) a = mv 2 /k g

(c) a = (k/m)y g

(d) a = (k/m)y + g
(e) a = (k/m)y g

c
Copyright 2008
American Association of Physics Teachers

2008 F = ma Exam

17. A mass m is resting at equilibrium suspended from a vertical spring of natural length L and spring
constant k inside a box as shown:

The box begins accelerating upward with acceleration a. How much closer does the equilibrium
position of the mass move to the bottom of the box?
(a) (a/g)L
(b) (g/a)L
(c) m(g + a)/k
(d) m(g a)/k
(e) ma/k

18. A uniform circular ring of radius R is fixed in place. A particle is placed on the axis of the ring
at a distance much greater than R and allowed to fall towards the ring under the influence of the
rings gravity. The particle achieves a maximum speed v. The ring is replaced with one of the same
(linear) mass density but radius 2R, and the experiment is repeated. What is the new maximum
speed of the particle?
(a)
(b)

1
2v
1 v
2

(c) v

(d) 2v
(e) 2v
19. A car has an engine which delivers a constant power. It accelerates from rest at time t = 0, and at
t = t0 its acceleration is a0 . What is its acceleration at t = 2t0 ? Ignore energy loss due to friction.
(a)
(b)

1
2 a0
1 a0
2

(c) a0

(d) 2a0
(e) 2a0
c
Copyright 2008
American Association of Physics Teachers

2008 F = ma Exam

20. The Youngs modulus, E, of a material measures how stiff it is; the larger the value of E, the
more stiff the material. Consider a solid, rectangular steel beam which is anchored horizontally to
the wall at one end and allowed to deflect under its own weight. The beam has length L, vertical
thickness h, width w, mass density , and Youngs modulus E; the acceleration due to gravity is
g. What is the distance through which the other end moves? (Hint: you are expected to solve this
problem by eliminating implausible answers. All of the choices are dimensionally correct.)


(a) h exp gL
E
2

(b) 2 gh
E

(c) 2Lh
(d)
(e)

3 gL4
2 Eh2
EL
3 gh

21. Consider a particle at rest which may decay into two (daughter) particles or into three (daughter)
particles. Which of the following is true in the two-body case but false in the three-body case?
(There are no external forces.)
(a) The velocity vectors of the daughter particles must lie in a single plane.
(b) Given the total kinetic energy of the system and the mass of each daughter particle, it
is possible to determine the speed of each daughter particle.
(c) Given the speed(s) of all but one daughter particle, it is possible to determine the speed
of the remaining particle.
(d) The total momentum of the daughter particles is zero.
(e) None of the above.
22. A bullet of mass m1 strikes a pendulum of mass m2 suspended from a pivot by a string of length
L with a horizontal velocity v0 . The collision is perfectly inelastic and the bullet sticks to the
bob. Find the minimum velocity v0 such that the bob (with the bullet inside) completes a circular
vertical loop.

(a) 2 Lg

(b) 5Lg

(c) (m1 + m2 )2 Lg/m1

(d) (m1 m2 ) Lg/m2

(e) (m1 + m2 ) 5Lg/m1

c
Copyright 2008
American Association of Physics Teachers

2008 F = ma Exam

10

23. Consider two uniform spherical planets of equal density but unequal radius. Which of the following
quantities is the same for both planets?
(a) The escape velocity from the planets surface.
(b) The acceleration due to gravity at the planets surface.
(c) The orbital period of a satellite in a circular orbit just above the planets surface.
(d) The orbital period of a satellite in a circular orbit at a given distance from the planets
center.
(e) None of the above.
24. A ball is launched upward from the ground at an initial vertical speed of v0 and begins bouncing
vertically. Every time it rebounds, it loses a proportion of the magnitude of its velocity due to the
inelastic nature of the collision, such that if the speed just before hitting the ground on a bounce is
v, then the speed just after the bounce is rv, where r < 1 is a constant. Calculate the total length
of time that the ball remains bouncing, assuming that any time associated with the actual contact
of the ball with the ground is negligible.

(a)
(b)
(c)
(d)
(e)

2v0 1
g 1r
v0 r
g 1r
2v0 1r
g
r
2v0 1
g 1r 2
2v0
1
g 1+(1r)2

25. Two satellites are launched at a distance R from a planet of negligible radius. Both satellites are
launched in the tangential direction. The first satellite launches correctly at a speed v0 and enters
a circular orbit. The second satellite, however, is launched at a speed 12 v0 . What is the minimum
distance between the second satellite and the planet over the course of its orbit?
(a)
(b)
(c)
(d)
(e)

1 R
2
1
2R
1
3R
1
4R
1
7R

c
Copyright 2008
American Association of Physics Teachers

2008 F = ma Exam

This page is intentionally blank

c
Copyright 2008
American Association of Physics Teachers

11

ANSWERS TO 2008 Fnet = ma CONTEST


A1
A2
A3
A4
A5
A6
A7
A8
A9
A10
A11
A12
A13
A14
A15
A16
A17
A18
A19
A20
A21
A22
A23
A24
A25

D
C
A
D
C
A
A
C
B
B
A
D
B
B
D
E
E
C
B
D
B
E
C
A
E

United States Physics Team


Quarter Final Contest
2008

2008 Quarter-final Exam

AAPT
AIP

UNITED STATES PHYSICS TEAM


2008

2008 Quarter-Final Exam


4 QUESTIONS - 60 MINUTES
INSTRUCTIONS
DO NOT OPEN THIS TEST UNTIL YOU ARE TOLD TO BEGIN
Show all your work. Partial credit will be given.
Start each question on a new sheet of paper. Put your name in the upper right-hand corner of
each page, along with the question number and the page number/total pages for this problem.
For example,
Doe, Jamie
Prob. 1 - P. 1/3
A hand-held calculator may be used. Its memory must be cleared of data and programs. You
may use only the basic functions found on a simple scientific calculator. Calculators may not
be shared.
Cell phones may not be used during the exam or while the exam papers are present. You
may not use any tables, books, or collections of formulas.
Each of the four questions is worth 25 points. The questions are not necessarily of the same
difficulty. Good luck!
In order to maintain exam security, do not communicate any information about
the questions (or their answers or solutions) on this contest until after March 10,
2008.

c
Copyright 2008
American Association of Physics Teachers

2008 Quarter-final Exam

1. A charged particle with charge q and mass m is given an initial kinetic energy K0 at the
middle of a uniformly charged spherical region of total charge Q and radius R. q and Q have
opposite signs. The spherically charged region is not free to move. Throughout this problem
consider electrostatic forces only.

(a) Find the value of K0 such that the particle will just reach the boundary of the spherically
charged region.
(b) How much time does it take for the particle to reach the boundary of the region if it
starts with the kinetic energy K0 found in part (a)?

2. A uniform pool ball of radius r and mass m begins at rest on a pool table. The ball is given a
horizontal impulse J of fixed magnitude at a distance r above its center, where 1 1.
The coefficient of kinetic friction between the ball and the pool table is . You may assume
the ball and the table are perfectly rigid. Ignore effects due to deformation. (The moment of
inertia about the center of mass of a solid sphere of mass m and radius r is Icm = 52 mr 2 .)

J
r
r

(a) Find an expression for the final speed of the ball as a function of J, m, and .
(b) For what value of does the ball immediately begin to roll without slipping, regardless
of the value of ?

c
Copyright 2008
American Association of Physics Teachers

2008 Quarter-final Exam

3. A block of mass m slides on a circular track of radius r whose wall and floor both have
coefficient of kinetic friction with the block. The size of the block is small compared to the
radius of the track. The floor lies in a horizontal plane and the wall is vertical. The block is
in constant contact with both the wall and the floor. The block has initial speed v0 .

(a) Let the block have kinetic energy E after traveling through an angle . Derive an
expression for dE
d in terms of g, r, , m and E.
(b) Suppose the block circles the track exactly once before coming to a halt. Determine v0
in terms of g, r, and .

4. Two beads, each of mass m, are free to slide on a rigid, vertical hoop of mass mh . The beads
are threaded on the hoop so that they cannot fall off of the hoop. They are released with
negligible velocity at the top of the hoop and slide down to the bottom in opposite directions.
The hoop remains vertical at all times. What is the maximum value of the ratio m/mh such
that the hoop always remains in contact with the ground? Neglect friction.
beads released with negligible velocity, slide on hoop wire
vertical circular hoop

Ground

c
Copyright 2008
American Association of Physics Teachers

2008 Quarter-final Exam - Solutions

2008 Quarter-Final Exam Solutions


1. A charged particle with charge q and mass m starts with an initial kinetic energy K at the
middle of a uniformly charged spherical region of total charge Q and radius R. q and Q have
opposite signs. The spherically charged region is not free to move.
(a) Find the value of K0 such that the particle will just reach the boundary of the spherically
charged region.
(b) How much time does it take for the particle to reach the boundary of the region if it
starts with the kinetic energy K0 found in part (a)?
Solution:
Assume that q is negative and Q is positive.
(a) Apply Gausss Law to a spherical shell of radius r where r < R. Then,
4r3
0

E4r2 =
where the charge density =

3Q
4R3

Solving for the Electric Field at a distance r from the center, we find
E=

r
30

We now find the potential difference between the center of the sphere and the outer boundary
of the charged cloud.
R

Z
V =

Edr
0

Substituting in our expression for E, we now find


V =

R2
60

Using conservation of energy,


Klost = Ugained

K0 = qV =

qQ
8R0

(b) Apply Newtons Second Law to the object of charge q when it is located a distance r
away from the center of the sphere.
c
Copyright 2008
American Association of Physics Teachers

2008 Quarter-final Exam - Solutions

Fnet = ma
(q)E = m

d2 r
dt2

d2 r
qr
=
2
dt
30 m
We recognize that this is the differential equation for simple harmonic motion
d2 r
= 2 r
dt2
where =

q
30 m .

Since the charge has the minimum kinetic energy needed to reach the surface, the trip from
the center of the sphere to the outer boundary is one-fourth of a cycle of SHM.
Therefore,

T
=
t= =
4
2
2

30 m
q

.
Substituting, in for rho, we find

t=
2

40 mR3
qQ

.
2. A uniform pool ball of radius r begins at rest on a pool table. The ball is given a horizontal
impulse J of fixed magnitude at a distance r above its center, where 1 1. The
coefficient of kinetic friction between the ball and the pool table is . You may assume the
ball and the table are perfectly rigid. Ignore effects due to deformation. (The moment of
inertia about the center of mass of a solid sphere of mass m and radius r is Icm = 52 mr2 .)
(a) Find an expression for the final speed of the ball as a function of J, m, and .
(b) For what value of does the ball immediately begin to roll without slipping, regardless
of the value of .
.
Solution 1:
(a) Consider an axis perpendicular to the initial impulse and coplanar with the table. (Throughout this solution we consider only torques and angular momenta with respect to this axis.)
After the initial impulse, the torque about this axis is always zero, so angular momentum is
conserved. The initial impulse occurs a perpendicular distance ( + 1) r from the axis, so the
angular momentum is
L = ( + 1) rJ
After the ball has skidded along the table for a certain distance, it will begin to roll without
v
slipping. At that point, = rf .
c
Copyright 2008
American Association of Physics Teachers

2008 Quarter-final Exam - Solutions

Meanwhile, its moment of inertia about this axis is (by the parallel-axis theorem) I = Icm +
mr2 = 75 mr2 , so that its final angular velocity is given by
L = I
7
( + 1) rJ = mr2
5
7
( + 1) J = mvf
5
.
Therefore,
vf =

5J
(1 + )
7m

.
(b) The ball acquires linear momentum J as a result of the horizontal impulse, so its initial
velocity v is given by
mv = J
We want the initial angular velocity and initial velocity to satisfy the no-slip condition v = r;
thus
7
( + 1) J = J
5
2
=
5
.
Solution 2:.
Consider torques and angular momenta about the center of mass. If the horizontal impulse is
large compared with the horizontal impulse from friction during the time that the cue stick
is in contact with the ball, then angular impulse = change in angular momentum becomes:
2
Jr = mr2 0
5
.
Linear impulse = change in linear momentum yields:
J = mv0
.
(b) We want the initial angular velocity and initial velocity to satisfy the no-slip condition
v0 = 0 r; thus
2
2
J = mv0 = J
5
5
.
2
=
5
.

c
Copyright 2008
American Association of Physics Teachers

2008 Quarter-final Exam - Solutions

(a) In the case that the ball does not immediately begin to roll without slipping, friction will
exert an angular impulse, f tr, about the center of mass of the ball as the ball skids along the
surface:
2
f tr = mr2 (f 0 )
5
.
Friction will also exert a linear impulse f t that will cause a change in linear momentum:
f t = m(vf v0 )
.
Combining the last two equations:
2
m(vf v0 ) = mr(f 0 )
5
After the ball has skidded along the table for a certain distance, it will begin to roll without
v
slipping. At that point, = rf .
2
2
vf + v0 = vf r0
5
5
Now, using the relationships between the impulse J and the initial angular and linear momentum,
vf +

J
2
J
= vf
m
5
m

.
J
7
(1 + ) = vf
m
5
.
vf =

5J
(1 + )
7m

.
3. A block of mass m slides on a circular track of radius r whose wall and floor both have
coefficient of kinetic friction with the block. The floor lies in a horizontal plane and the
wall is vertical. The block is in constant contact with both the wall and the floor. The block
has initial speed v0 .
(a) Let the block have kinetic energy E after traveling through an angle . Derive an
expression for dE
d in terms of g, r, , m and E.
(b) Suppose the block circles the track exactly once before coming to a halt. Determine v0
in terms of g, r, and .

c
Copyright 2008
American Association of Physics Teachers

2008 Quarter-final Exam - Solutions

Solution Let v be the speed of the block after it has traveled through an angle . When we
apply Newtons Second Law to the block as it is traveling around the circular path of radius
r at a speed v, we find that the force of the wall on the block, Fw , is
Fw =

mv 2
r

and the force of the floor on the block, FN , is


FN = mg
Therefore, the total force of friction on the block is
FF ric = m(

v2
+ g)
r

.
Let dE denote the loss of kinetic energy as a result of the work W that friction does on the
block when it has traveled through an angle d. The block will travel a distance ds = rd as
it travels through an angle d.
Then,
dE = W = FF ric ds = FF ric rd
.
Now, substituting in the expressions for FF ric , we find
dE = m(v 2 + gr)d
.

dE
= (mv 2 + mgr) = (2E + mgr)
d

where we have simply used the fact that the kinetic energy E is E =

mv 2
2 .

(b) Now, we separate variables and solve the differential equation.


Z
Z
dE
= d
2E + mgr
.

1
ln |2E + mgr| = + C
2

.
Exponentiating both sides, yields
2E + mgr = Ce2
Let E0 denote the initial kinetic energy. Then, using initial conditions, we find that C =
2E0 + mgr.
Now, using the fact that E = 0 when = 2, we obtain
mgr = (2E0 + mgr)e4
c
Copyright 2008
American Association of Physics Teachers

2008 Quarter-final Exam - Solutions

.
Solving for E0 ,

1
E0 = mgr(e4 1)
2

.
Then, using E0 =

mv02
2 ,

and we find that


v0 =

p
gr(e4 1)

.
4. Two beads, each of mass m, are free to slide on a rigid, vertical hoop of mass mh . The beads
are threaded on the hoop so that they cannot fall off of the hoop. They are released with
negligible velocity at the top of the hoop and slide down to the bottom in opposite directions.
What is the maximum value of the ratio m/mh such that the hoop always remains in contact
with the ground? Neglect friction.
Solution 1: Draw a free-body diagram for each bead; let FN be the (inward) normal force
exerted by the hoop on the bead. Let be the angular position of the bead, measured from
the top of the hoop, and let the hoop have radius r. We see that
FN + mg cos = m

v2
r

v2
mg cos
r
is given by

FN = m
The (downward) vertical component FN y

FN y = FN cos
From Newtons third law, the two beads together exert an upwards vertical force on the hoop
given by
Fu = 2FN y
 2

v
Fu = 2m cos
g cos
r
noting that the beads clearly reach the same position at the same time.
Meanwhile, when each bead is at a position it has moved through a vertical distance
r (1 cos ). Thus from energy conservation,
1
mv 2 = mgr (1 cos )
2
v2
= 2g (1 cos )
r
Inserting this into the previous result,
Fu = 2m cos (2g (1 cos ) g cos )

Fu = 2mg 2 cos 3 cos2
c
Copyright 2008
American Association of Physics Teachers

2008 Quarter-final Exam - Solutions

If the beads ever exert an upward force on the hoop greater than mh g, the hoop will leave
the ground; i.e., the condition for the hoop to remain in contact with the ground is that for
all ,
Fu mh g
We can replace the left side by its maximum value. Letting s = cos ,

Fu = 2mg 2s 3s2
d
Fu = 2mg (2 6s)
ds
The derivative is zero at s = 13 , where
2
Fu(max) = mg
3
So our condition is

2
mg mh g
3
3
m

mh
2

Solution 2: As before, we apply energy conservation to find the speed of the beads:
p
v = 2gr (1 cos )
The vertical (downward) component of the beads velocity is thus
vy = v sin
p
vy = sin 2gr (1 cos )
While the hoop is in contact with the ground, the beads are the only part of the system in
motion, so the momentum of the system is simply the beads momentum:
py = 2mvy
p
py = 2m sin 2gr (1 cos )
The net (downward) force on the system is
dpy
dpy d
dpy v
=
=
dt
d dt
d r
 1p
p

d 
Fnet = 2m 2gr
sin 1 cos
2gr (1 cos )
dt
r



1
Fnet = 4mg cos 1 cos + sin
sin
1 cos
2 1 cos


1
2
2
Fnet = 4mg cos cos + sin
2

2
Fnet = 2mg 2 cos 3 cos + 1
Fnet =

c
Copyright 2008
American Association of Physics Teachers

2008 Quarter-final Exam - Solutions

This downward force is provided by gravity and the normal force upward on the hoop:
Fnet = 2mg + mh g FN
If the hoop is to remain in contact with the ground, the normal force can never be negative:
FN 0
2mg + mh g Fnet 0

2mg + mh g 2mg 2 cos 3 cos2 + 1 0

2mg 2 cos 3 cos2 mh g
and we proceed as above.

c
Copyright 2008
American Association of Physics Teachers

United States Physics Team


Semi Final Contest
2008

2008 Semifinal Exam

AAPT
AIP

UNITED STATES PHYSICS TEAM


2008
Semifinal Exam

DO NOT DISTRIBUTE THIS PAGE


Important Instructions for the Exam Supervisor
This examination consists of three parts.
Part A has four questions and is allowed 90 minutes.
Part B has two questions and is allowed 90 minutes.
Part C has one question and is allowed 20 minutes. The answer for Part C will not be used for team
selection, but will be used for special recognition from the Optical Society of America.
The first page that follows is a cover sheet. Examinees may keep the cover sheet for all three parts of
the exam.
The three parts are then identified by the center header on each page. Examinees are only allowed to
do one part at a time, and may not work on other parts, even if they have time remaining.
Allow 90 minutes to complete Part A. Do not let students look at Part B or Part C. Collect the answers
to Part A before allowing the examinee to begin Part B. Examinees are allowed a 10 to 15 minutes
break between parts A and B.
Allow 90 minutes to complete Part B. Do not let students look at Part C or go back to Part A. Collect
the answers to part B before allowing the examinee to begin Part C. Examinees are allowed a 10 to 15
minutes break between Parts B and C.
Allow 20 minutes to complete Part C. This part is optional; scores on Part C will not be used to select
the US Team. Examinees may not go back to Part A or B.
Ideally the test supervisor will divide the question paper into 4 parts: the cover sheet (page 2), Part
A (pages 3-7), Part B (pages 8-10), and Part C (page 11). Examinees should be provided the parts
individually, although they may keep the cover sheet.
The supervisor must collect all examination questions, including the cover sheet, at the end of the exam,
as well as any scratch paper used by the examinees. Examinees may not take the exam questions. The
examination questions may be returned to the students after March 31, 2008.
Examinees are allowed calculators, but they may not use symbolic math, programming, or graphic
features of these calculators. Calculators may not be shared and their memory must be cleared of data
and programs. Cell phones, PDAs or cameras may not be used during the exam or while the exam
papers are present. Examinees may not use any tables, books, or collections of formulas.
Please provide the examinees with graph paper for Part A.

c
Copyright 2008
American Association of Physics Teachers

2008 Semifinal Exam

Cover Sheet

AAPT
AIP

UNITED STATES PHYSICS TEAM


2008

Semifinal Exam
INSTRUCTIONS
DO NOT OPEN THIS TEST UNTIL YOU ARE TOLD TO BEGIN
Work Part A first. You have 90 minutes to complete all four problems. Each question is worth 25
points. Do not look at Parts B or C during this time.
After you have completed Part A you may take a break.
Then work Part B. You have 90 minutes to complete both problems. Each question is worth 50 points.
Do not look at Parts A or C during this time.
Show all your work. Partial credit will be given. Do not write on the back of any page. Do not write
anything that you wish graded on the question sheets.
Start each question on a new sheet of paper. Put your school ID number, your name, the question
number and the page number/total pages for this problem, in the upper right hand corner of each
page. For example,
School ID #
Doe, Jamie
A1 - 1/3
A hand-held calculator may be used. Its memory must be cleared of data and programs. You may use
only the basic functions found on a simple scientific calculator. Calculators may not be shared. Cell
phones, PDAs or cameras may not be used during the exam or while the exam papers are present.
You may not use any tables, books, or collections of formulas.
Questions with the same point value are not necessarily of the same difficulty.
Part C is an optional part of the test. You will be given 20 additional minutes to complete Part C.
Your score on Part C will not affect the selection for the US Team, but can be used for special prizes
and recognition to be awarded by the Optical Society of America.
In order to maintain exam security, do not communicate any information about the
questions (or their answers/solutions) on this contest until after March 31, 2008.
Possibly Useful Information. You may use this sheet for all three parts of the exam.
g = 9.8 N/kg
G = 6.67 1011 N m2 /kg2
9
2
2
k = 1/40 = 8.99 10 N m /C
km = 0 /4 = 107 T m/A
8
c = 3.00 10 m/s
kB = 1.38 1023 J/K
23
1
NA = 6.02 10 (mol)
R = NA kB = 8.31 J/(mol K)
= 5.67 108 J/(s m2 K4 )
e = 1.602 1019 C
1eV = 1.602 1019 J
h = 6.63 1034 J s = 4.14 1015 eV s
31
2
me = 9.109 10
kg = 0.511 MeV/c
(1 + x)n 1 + nx for |x| 1
1 3
cos 1 21 2 for || 1
sin 6 for || 1
c
Copyright 2008
American Association of Physics Teachers

2008 Semifinal Exam

Part A

Part A
Question A1
Four square metal plates of area A are arranged at an even spacing d as shown in the diagram. (Assume
that A d2 .)
Plate 1
Plate 2
Plate 3
d
Plate 4
d
d

Plates 1 and 4 are first connected to a voltage source of magnitude V0 , with plate 1 positive; plates 2 and
3 are then connected together with a wire. The wire is subsequently removed. Finally, the voltage source
attached between plates 1 and 4 is replaced with a wire. The steps are summarized in the diagrams below.

Step 1

Step 2

Step 3

Find the resulting potential difference V12 between plates 1 and 2; like wise find V23 and V34 , defined
similarly.
Assume, in each case, that a positive potential difference means that the top plate is at a higher potential
than the bottom plate.

c
Copyright 2008
American Association of Physics Teachers

2008 Semifinal Exam

Part A

Question A2
A simple heat engine consists of a moveable piston in a cylinder filled with an ideal monatomic gas. Initially
the gas in the cylinder is at a pressure P0 and volume V0 . The gas is slowly heated at constant volume. Once
the pressure reaches 32P0 the piston is released, allowing the gas to expand so that no heat either enters
or escapes the gas as the piston moves. Once the pressure has returned to P0 the outside of the cylinder is
cooled back to the original temperature, keeping the pressure constant. For the monatomic ideal gas you
should assume that the molar heat capacity at constant volume is given by CV = 32 R, where R is the ideal
gas constant. You may express your answers in fractional form or as decimals. If you choose decimals, keep
three significant figures in your calculations. The diagram below is not necessarily drawn to scale.

Pressure

32P0

P0

V0

Vmax

Volume

a. Let Vmax be the maximum volume achieved by the gas during the cycle. What is Vmax in terms of
V0 ? If you are unable to solve this part of the problem, you may express your answers to the remaining
parts in terms of Vmax without further loss of points.
b. In terms of P0 and V0 determine the heat added to the gas during a complete cycle.
c. In terms of P0 and V0 determine the heat removed from the gas during a complete cycle.
d. What is the efficiency of this cycle?

c
Copyright 2008
American Association of Physics Teachers

2008 Semifinal Exam

Part A

Question A3
A certain planet of radius R is composed of a uniform material that, through radioactive decay, generates a
net power P . This results in a temperature differential between the inside and outside of the planet as heat
is transfered from the interior to the surface.
The rate of heat transfer is governed by the thermal conductivity. The thermal conductivity of a material is a measure of how quickly heat flows through that material in response to a temperature gradient.
Specifically, consider a thin slab of material of area A and thickness x where one surface is hotter than the
other by an amount T . Suppose that an amount of heat Q flows through the slab in a time t. The
thermal conductivity k of the material is then
k=

Q 1 x
.
t A T

It is found that k is approximately constant for many materials; assume that it is constant for the planet.
For the following assume that the planet is in a steady state; temperature might depend on position, but
does not depend on time.
a. Find an expression for the temperature of the surface of the planet assuming blackbody radiation, an
emissivity of 1, and no radiation incident on the planet surface. You may express your answer in terms
of any of the above variables and the Stephan-Boltzmann constant .
b. Find an expression for the temperature difference between the surface of the planet and the center of
the planet. You may express your answer in terms of any of the above variables; you do not need to
answer part (a) to be able to answer this part.

c
Copyright 2008
American Association of Physics Teachers

2008 Semifinal Exam

Part A

Question A4
A tape recorder playing a single tone of frequency f0 is dropped from rest at a height h. You stand directly
underneath the tape recorder and measure the frequency observed as a function of time. Here t = 0 s is the
time at which the tape recorder was dropped.
t (s)
2.0
4.0
6.0
8.0
10.0

f (Hz)
581
619
665
723
801

The acceleration due to gravity is g = 9.80 m/s2 and the speed of sound in air is vsnd = 340 m/s. Ignore
air resistance. You might need to use the Doppler shift formula for co-linear motion of sources and observers
in still air,
vsnd vobs
f = f0
vsnd vsrc
where f0 is the emitted frequency as determined by the source, f is the frequency as detected by the observer,
and vsnd , vsrc , and vobs are the speed of sound in air, the speed of the source, and the speed of the observer.
The positive and negative signs are dependent upon the relative directions of the motions of the source and
the observer.
a. Determine the frequency measured on the ground at time t, in terms of f0 , g, h, and vsnd . Consider
only the case where the falling tape recorder doesnt exceed the speed of sound vsnd .
b. Verify graphically that your result is consistent with the provided data.
c. What (numerically) is the frequency played by the tape recorder?
d. From what height h was the tape recorder dropped?

c
Copyright 2008
American Association of Physics Teachers

2008 Semifinal Exam

Part A

STOP: Do Not Continue to Part B


If there is still time remaining for Part A, you should review your work for
Part A, but do not continue to Part B until instructed by your exam
supervisor.

c
Copyright 2008
American Association of Physics Teachers

2008 Semifinal Exam

Part B

Part B
Question B1
A platform is attached to the ground by an ideal spring of constant k; both the spring and the platform
have negligible mass; assume that your mass is mp . Sitting on the platform is a rather large lump of clay of
mass mc = rmp , with r some positive constant that measures the ratio mc /mp . You then gently step onto
the platform, and the platform settles down to a new equilibrium position, a vertical distance D below the
original position. Throughout the problem assume that you never lose contact with the platform.

h
D

a. You then slowly pick up the lump of clay and hold it a height h above the platform. Upon releasing
the clay you and the platform will oscillate up and down; you notice that the clay strikes the platform
after the platform has completed exactly one oscillation. Determine the numerical value of the ratio
h/D.
b. Assume the resulting collision between the clay and the platform is completely inelastic. Find the
ratio of the amplitude of the oscillation of the platform after the collision (Af ) to the amplitude of the
oscillations of the platform before the collision (Ai ). Determine Af /Ai in terms of the mass ratio r and
any necessary numerical constants.
c. Sketch a graph of the position of the platform as a function of time, with t = 0 corresponding to the
moment when the clay is dropped. Show one complete oscillation after the clay has collided with the
platform. It is not necessary to use graph paper.
d. The above experiment is only possible if the mass ratio r is less than some critical value rc . Otherwise,
despite the clay having been dropped from the height determined in part (a), the oscillating platform
will hit the clay before the platform has completed one full oscillation. On your graph in part (c)
sketch the position of the clay as a function of time relative to the position of the platform for the
mass ratio r = rc .

c
Copyright 2008
American Association of Physics Teachers

2008 Semifinal Exam

Part B

Question B2
Consider a parallel plate capacitor with the plates vertical. The plates of the capacitor are rigidly supported
in place. The distance between the plates is d. The plates have height h and area A d2 . Assume
throughout this problem that the force of air resistance may be neglected; however, the force of gravity
cannot be neglected. Neglect any edge effects as well as any magnetic effects.
d/2

String

Rigid Support

h
h
h/2

a. A small metal ball with a mass M and a charge q is suspended from a string of length L that is tied
to a rigid support. When the capacitor is not charged, the metal ball is located at the center of the
capacitor at a distance d/2 from both plates and at a height h/2 above the bottom edge of the plates.
If instead a constant potential difference V0 is applied across the plates, the string will make an angle
0 to the vertical when the metal ball is in equilibrium.
i. Determine 0 in terms of the given quantities and fundamental constants.
ii. The metal ball is then lifted until it makes an angle to the vertical where is only slightly
greater than 0 . The metal ball is then released from rest. Show that the resulting motion is
simple harmonic motion and find the period of the oscillations in terms of the given quantities
and fundamental constants.
iii. When the ball is at rest in the equilibrium position 0 , the string is cut. What is the maximum
value for V0 so that the ball will not hit one of the plates before exiting? Express your answer in
terms of the given quantities and fundamental constants.
b. Suppose instead that the ball of mass M and charge q is released from rest at a point halfway between
the plates at a time t = 0. Now, an AC potential difference V (t) = V0 sin t is also placed across the
capacitor. The ball may hit one of the plates before it falls (under the influence of gravity) out of the
region between the plates. If V0 is sufficiently large, this will
p range of angular
p only occur for some
frequencies min < < max . You may assume that min g/h and max g/h. Making these
assumptions, find expressions for min and max in terms of the given quantities and/or fundamental
constants.
c. Assume that the region between the plates is not quite a vacuum, but instead humid air with a uniform
resistivity . Ignore any effects because of the motion of the ball, and assume that the humid air doesnt
change the capacitance of the original system.
i. Determine the resistance between the plates.
ii. If the plates are originally charged to a constant potential source V0 , and then the potential is
removed, how much time is required for the potential difference between the plates to decrease to
a value of V0 /e, where ln e = 1?
iii. If the plates are instead connected to an AC potential source so that the potential difference
across the plates is V0 sin t, determine the amplitude I0 of the alternating current through the
potential source.

c
Copyright 2008
American Association of Physics Teachers

2008 Semifinal Exam

Part B

10

STOP: Do Not Continue to Part C


If there is still time remaining for Part B, you should review your work for
Part B, but do not continue to Part C until instructed by your exam
supervisor. You may not return to Part A

c
Copyright 2008
American Association of Physics Teachers

2008 Semifinal Exam

Part C

11

Optical Society of America Bonus Question


Researchers have developed a lens made of liquid. The spherical lens consists of a droplet of transparent
liquid resting on an electrically controllable surface. When the voltage of the surface is changed, the droplet
itself changes shape; it either tries to ball-up more strongly or it becomes flatter. The figure below is a
sketch of the liquid lens and several parameters that describe it, including the thickness of the lens (t), the
radius of curvature of the top surface (R) and the contact angle (), which represents the angle between the
flat surface beneath the droplet and the tangent to the curved surface at the point of contact.

a. When a certain voltage is applied, both the contact angle and lens thickness increase (and the lens
becomes more curved). In this case, is the liquid attracted or repelled by the surface?
b. Express the contact angle as a function of R and t.
c. The total volume of the liquid lens is an important parameter because as the liquid lens changes shape,
its volume is conserved. Calculate the volume of the lens as a function of R and t.
d. Use your result to part (b) to eliminate the variable t from your expression for the volume and find
V (R, ).
e. By changing the voltage on the control surface, the contact angle, , can be changed, which in turn
changes the focal length of the lens, f . The lensmakers formula can be used to calculate the focal
length and is given by


1
1
1
,

= (nliquid nair )
f
R1
R2
where nliquid and nair are the refractive indices of the liquid in the lens and air around it, and R1 and
R2 are the radii of curvature of the two surfaces of the lens. In figure 1, R1 is the curved face and R2
is the flat face. Use the lensmakers formula to calculate the focal length of the lens in terms of the
total volume of the liquid, the contact angle, and the relevant refractive indices.
Sidenote: liquid lenses are interesting because they are electrically controllable, variable focus lenses
that can be very compact. People are working on putting them into cell phone cameras for ultracompact zoom lenses. For more information on this type of liquid lens, see T. Krupenkin, S. Yang,
and P. Mach, Tunable liquid microlens, Appl. Phys. Lett. 82, 316-318 (2003).

c
Copyright 2008
American Association of Physics Teachers

2008 Semifinal Exam

AAPT
AIP

UNITED STATES PHYSICS TEAM


2008
Semifinal Exam
6 QUESTIONS - Several MINUTES
INSTRUCTIONS

So
lut
ion
s

DO NOT OPEN THIS TEST UNTIL YOU ARE TOLD TO BEGIN


Show all your work. Partial credit will be given.

Start each question on a new sheet of paper. Put your name in the upper right-hand corner of each
page, along with the question number and the page number/total pages for this problem. For example,
Doe, Jamie

Prob. 1 - P. 1/3

A hand-held calculator may be used. Its memory must be cleared of data and programs. You may use
only the basic functions found on a simple scientific calculator. Calculators may not be shared.
Cell phones may not be used during the exam or while the exam papers are present. You may not use
any tables, books, or collections of formulas.
Each of the four questions in part A are worth 25 points. Each of the two questions in part B are
worth 50 points. The questions are not necessarily of the same difficulty. Good luck!
In order to maintain exam security, do not communicate any information about the
questions (or their answers or solutions) on this contest until after April 10, 2008.

DO NOT OPEN THIS TEST UNTIL YOU ARE TOLD TO BEGIN

c
Copyright 2008
American Association of Physics Teachers

2008 Semifinal Exam

Part A
Question A1
Four square metal plates of area A are arranged at an even spacing d as shown in the diagram. (Assume
that A >> d2 .)
Plate 1
Plate 2
Plate 3
d
Plate 4
d
d

So
lut
ion
s

Plates 1 and 4 are first connected to a voltage source of magnitude V0 , with plate 1 positive. Plates 2
and 3 are then connected together with a wire, which is subsequently removed. Finally, the voltage source
attached between plates 1 and 4 is replaced with a wire. The steps are summarized in the diagram below.

(a)

(b)

(c)

What is the resulting potential difference between


a. Plates 1 and 2 (Call it V1 ),

b. Plates 2 and 3 (Call it V2 ), and


c. Plates 3 and 4 (Call it V3 ).

Assume, in each case, that a positive potential difference means that the top plates is at a high potential
than the bottom plate.
Solution

There are two fairly easy ways to do this problem, one rather straightforward application of capacitors, the
other a more elegant, and much, much short, application of boundary conditions in electric fields.
The first method involves treating the problem as three series capacitors. Each has an identical capacitance C. The figure below then show the three steps.

C1

C1

C1

C2

C2

C2

C3

C3

C3

Since C2 is shorted out originally, then effectively there are only two capacitors in series, so the voltage
drop across each is V0 /2, where the a positive potential difference means that the top plate of any given
capacitor is positive. The top plate of C1 will then have a positive charge of q0 = CV0 /2. Note that this
means that the bottom plate of the top capacitor will have a negative charge of q0 . Removing the shorting
wire across C2 will not change the charges or potential drops across the other two capacitors. Removing the
source V0 will also make no difference.
c
Copyright 2008
American Association of Physics Teachers

2008 Semifinal Exam

Shorting the top plate of C1 with the bottom plate of C3 will make a difference. Positive charge will flow
out of top plate of C1 into the bottom plate of C3 . Also, negative charge will flow out of the bottom plate
of C1 into the top plate of C2 . The result is that C1 will acquire a potential difference of V1 , C2 a potential
difference of V2 , and C3 a potential difference of V3 . Let the final charge on the top plate of each capacitor
also be labeled as q1 , q2 , and q3 .
The last figure implies that
V1 + V2 + V3 = 0.
By symmetry, we have
V1 = V3 .
so
2V1 = V2 .
By charge conservation between the bottom plate of C1 and the top plate of C2 we have

But q = CV , so

So
lut
ion
s

q0 = q1 + q2 .
1
V0 = V1 + V2
2

Combining the above we get

1
V0
2
1
V0
3

1
V2 + V2 ,
2

V2 .

Finally, solving for V1 , we get V1 = V0 /6.


Alternatively, we could focus on the plate arrangement and the fact that across a boundary |E | =
|/0 |, a consequence of Gausss Law. Also, we have, for parallel plate configurations, |V | = |Ed|. Since 0
and d are the same for each of the three regions, it is sufficient to simply look at the behavior of the electric
fields.

E0

E0

E1

E2

E1

In the first picture we require that 2E0 = V0 /d. The charge density on the second plate requires that
E = E0 . In the last picture we have 2E1 +E2 = 0, since the potential between the top plate and the bottom
plate is zero. But we also have, on the second plate, E = E1 E2 . Combining, E0 = 12 E2 E2 = 23 E2 ,
and therefore V2 = 31 V0 , and V1 = V0 /6.

c
Copyright 2008
American Association of Physics Teachers

2008 Semifinal Exam

Question A2
A simple heat engine consists of a piston in a cylinder filled with an ideal monatomic gas. Initially the gas
in the cylinder is at a pressure P0 and volume V0 . The gas is slowly heated at constant volume. Once the
pressure reaches 32P0 the piston is released, allowing the gas to expand so that no heat either enters or
escapes the gas as the piston moves. Once the pressure has returned to P0 the the outside of the cylinder
is cooled back to the original temperature, keeping the pressure constant. For the monatomic ideal gas
you should assume that the specific heat capacity at constant volume is given by CV = 32 nR, where n is
the number of moles of the gas present and R is the ideal gas constant. You may express your answers in
fractional form or as decimals. If you choose decimals, keep three significant figures in your calculations.
The diagram below is not necessarily drawn to scale.

Pressure

So
lut
ion
s

32P0

P0

V0

Vmax

Volume

a. Let Vmax be the maximum volume achieved by the gas during the cycle. What is Vmax in terms of
V0 ? If you are unable to solve this part of the problem, you may expressyour answers to the remaining
parts in terms of Vmax without further loss of points.
b. In terms of P0 and V0 determine the heat added to the gas during a complete cycle.
c. In terms of P0 and V0 determine the heat removed from the gas during a complete cycle.
d. Defining efficiency e as the net work done by the gas divided by the heat added to the gas, what is the
efficiency of this cycle?
e. Determine the ratio between the maximum and minimum temperatures during this cycle.
Solution
It is convenient to construct two tables and solve this problem in a manner similar to a Sudoku puzzle.
Defining point 1 to be the initial point, and measuring P , V , and T in terms of P0 , V0 , and T0 , while
measuring Q, W , and U in terms of nRT0 = P0 V0 , we have initially
Point
1
2
3

P
1
32
1

V
1
1

T
1

Process
12
23
31
net

W
0

0
0

c
Copyright 2008
American Association of Physics Teachers

2008 Semifinal Exam

The obvious values have been filled in: the initial conditions, and zeroes corresponding to Q = 0 along
an adiabat, W = 0 along a constant volume process, and finally U = 0 for a net process.
The convention that will be used here is Q + W = U .
The ideal gas law, P V /T = nR, can be used to quickly determine T2 , since P/T is a constant for that
process. One can then use
Q = CV T
to find Q12 . The table values will now read
Point
1
2
3

P
1
32
1

V
1
1

Process
12
23
31
net

T
1
32

Q
3
2 31

W
0

For the adiabatic process we have P V is a constant, where = CP /CV . Students who dont know this
can derive it, although it will take some time.

So
lut
ion
s

The derivation is straightforward enough. Along an adiabatic process, Q = 0, so from Q + W = U


P dV =

Rearranging,

3
3
nR dT = (P dV + V dP )
2
2

0 = 5P dV + 3V dP

or

0=

Integrating,

dP
5 dV
+
3 V
P

Constant =

5
ln V + ln P
3

which can be written in the more familiar form

P V = Constant.

The factor of 32 was chosen so that the results are nice answers.

One can then find V3 (and, for that matter, Vmax ) by using this, and get
V3
=
V2

P2
P3

1/

= (32)3/5 = 8

Putting this in the table, and then quickly applying the ideal gas law to find T3 then enables the finding of
Q31 , since along this process
5
Q = CP T = nRT.
2
The tables now look like
Point
1
2
3

P
1
32
1

V
1
1
8

T
1
32
8

Process
12
23
31
net

3
2 31

W
0

0
25 7
0

It is now possible to determine Qnet and, from Q + W = U , Wnet .


So the tables now look like
Point
1
2
3

P
1
32
1

V
1
1
8

T
1
32
8

Process
12
23
31
net

Q
3
2 31

0
25 7
29

W
0

29

c
Copyright 2008
American Association of Physics Teachers

2008 Semifinal Exam

The negative net work reflects that the gas does work on the outside world. The efficiency of the process
is then
58
29
=
e=
93/2
93
It isnt much more work to fill in all of the values for both tables, a task that each student ought be able
to do. One key point will be process 3 1, where W = P V . the the rest are filled in by applications of
Q + W = U .
P
1
32
1

V
1
1
8

T
1
32
8

Process
12
23
31
net

Q
3
2 31

0
25 7
29

W
0
-36
7
29

U
3
2 31
-36
32 7
0

So
lut
ion
s

Point
1
2
3

c
Copyright 2008
American Association of Physics Teachers

2008 Semifinal Exam

Question A3
A certain planet of radius R is composed of a uniform material that, through radioactive decay, generates a
net power P . This results in a temperature differential between the inside and outside of the planet as heat
is transfered from the interior to the surface.
The rate of heat transfer is governed by the thermal conductivity. The thermal conductivity of a material is a measure of how quickly heat flows through that material in response to a temperature gradient.
Specifically, consider a thin slab of material of area A and thickness x where one surface is hotter than the
other by an amount T . Suppose that an amount of heat Q flows through the slab in a time t. The
thermal conductivity k of the material is then
Q 1 x
.
t A T

k=

It is found that k is approximately constant for many materials; assume that it is constant for the planet.
For the following assume that the planet is in a steady state; temperature might depend on position, but
does not depend on time.

So
lut
ion
s

a. Find an expression for the temperature of the surface of the planet assuming blackbody radiation, an
emissivity of 1, and no radiation incident on the planet surface. You may express your answer in terms
of any of the above variables and the Stephan-Boltzmann constant .
b. Find an expression for the temperature difference between the surface of the planet and the center of
the planet. You may express your answer in terms of any of the above variables; you do not need to
answer part (a) to be able to answer this part.
Solution

For the first question, apply the Boltzmann equation, and

P = ATs4

where A is the surface area of the planet, and Ts the temperature at the center. Then
Ts =

P
4R2

1/4

For the second question, it is reasonable to assume that the temperature depends on the distance form the
center only. Then the definition of k gives for a spherical shell of thickness dr
k=

Q 1 dr
.
t 4r2 dT

The heat through the shell depends on the power radiated from within the shell. Since the planet is
uniform, this depends on the volume according to
4
r3
r3
Q
= P 43 3 = P 3
t
R
3 R

so that rearrangement yields


dT =

P
r dr
4kR3

Integrating between the center and the surface,


T =

P
,
8kR

which could be used to find the temperature of the interior.


c
Copyright 2008
American Association of Physics Teachers

2008 Semifinal Exam

Question A4
A tape recorder playing a single tone of frequency f0 is dropped from rest at a height h. You stand directly
underneath the tape recorder and measure the frequency observed as a function of time. Here t = 0s is the
time at which the tape recorder was dropped.
t (s)
2.0
4.0
6.0
8.0
10.0

f (Hz)
581
619
665
723
801

So
lut
ion
s

The acceleration due to gravity is g = 9.80 m/s2 and the speed of sound in air is va = 340 m/s. Ignore
air resistance. You might need to use the Doppler shift formula for co-linear motion of sources and observers
in still air,
va vo
f = f0
va vs
where f0 is the emitted frequency as determined by the source, f is the frequency as detected by the observer,
and va , vs , and vo are the speed of sounds in air, the speed of the source, and the speed of the observer. The
positive and negative signs are dependent upon the relative directions of the source and the observer.
a. Determine the frequency measured on the ground at time t, in terms of f0 , g, h, and va .
b. Verify graphically that your result is consistent with the provided data.
c. What (numerically) is the frequency played by the tape recorder?
d. From what height h was the tape recorder dropped?

Solution

The position of the tape recorder above the ground at a time t is given by
1
y = h gt2
2

and the speed of the tape recorder is given by

vs = gt

The observer hears the sound emitted from the tape recorder a time t earlier, since it takes time for the
sound to travel to the listener. In this case,
y = va t
So at time t the listener is hearing the tape recorder when it had emitted at time t = t t, or
t = t

h
g 2
+
(t )
va
2va

Solve this for t , first by rearranging,


g 2
(t ) va t + (va t h) = 0
2
and the by applying the quadratic formula

t =

va

p
va 2 + 2gh 2gva t
.
g

c
Copyright 2008
American Association of Physics Teachers

2008 Semifinal Exam

This might not look right, but in the limit of small h and large va , it does reduce to the expected t = t if
one keeps the negative result.
Consequently,
p
vs = va 2 + 2gh 2gva t va

gives the velocity of that source had when it emitted the sound heard at time t. This result is negative,
indicating motion down, and toward the observer, so one must use the positive sign in the denominator of
the Doppler shift formula.
Applying the Doppler shift formula,
va
f = f0 p
2
va + 2gh 2gva t

So
lut
ion
s

which, in the limit of large va and small h, reduces to




g
f = f0 1 + t
va

Keeping to the correct expression, we can rearrange it as




2gh 2g
1
1
1
+
=

t
f2
f0 2
va 2
va

which would graph as a straight line by plotting t horizontally and 1/f 2 vertically. The slope of the line
would yield
2g

va f0 2
while the vertical intercept would yield

1
f0 2



2gh
1+ 2
va

Using this hint, a quick table of data to consider graphing would be


t (s)
2.0
4.0
6.0
8.0
10.0

The slope is 1.75 107 s.


Then

f0 =

f (Hz)
581
619
665
723
801

1/f 2 (106 s2 )
2.96
2.61
2.26
1.91
1.56

2(9.8)
Hz = 574Hz
(1.75 107 )(340)

The intercept is 3.31 106 s2 . This yields a height in meters given by


h = (340)

(340)2
(3.31)

= 533
(0.175)
2(9.8)

Clearly, an impressive building; and a more impressive tape player, that it could be heard from such a
distance!

c
Copyright 2008
American Association of Physics Teachers

2008 Semifinal Exam

10

Part B
Question B1

So
lut
ion
s

A platform is attached to the ground by an ideal spring of constant k; both the spring and the platform
have negligible mass. Sitting on the platform is a rather large lump of clay of mass mc . You then gently
step onto the platform, and the platform settles down to a new equilibrium position, a vertical distance D
below the original position. Assume that your mass is mp .

a. You then pick up the lump of clay and hold it a height h above the platform. Upon releasing the clay
the you and the platform will oscillate up and down; you notice that the clay strikes the platform after
the platform has completed exactly one oscillation. Determine h in terms of any or all of k, D, the
masses mp and mc , the acceleration of free fall g, and any necessary numerical constants. You must
express your answer in the simplest possible form.
b. Assume the resulting collision between the clay and the platform is completely inelastic. Find the ratio
of the amplitude of the oscillation of the platform before the collision (Ai ) and the amplitude of the
oscillations of the platform after the collision (Af ). Determine Af /Ai in terms of any or all of k, D,
the masses mp and mc , the acceleration of free fall g, and any necessary numerical constants.
c. Sketch a graph of the position of the platform as a function of time, with t = 0 corresponding to the
moment when the clay is dropped. Show one complete oscillation after the clay has collided with the
platform.
d. The above experiment is only possible if the ratio mc /mp is smaller than some critical value rc ,
otherwise the clay will hit the platform before one complete oscillation. An estimate for the value
of the critical ratio rc can be obtained by assuming the clay hits the platform after exactly one-half
of an oscillation. Assume that h is the same as is determined by part (a), and use this technique
to determine rc in terms of any or all of k, D, the mass mp , the acceleration of free fall g, and any
necessary numerical constants.
e. Is this estimate for rc too large or too small? You must defend your answer with an appropriate
diagram.
Solution
Stepping on the platform will lower it a distance D. This means that the spring constant of the platform
spring is given by
kD = mp g.
c
Copyright 2008
American Association of Physics Teachers

2008 Semifinal Exam

11

If the lump of clay is removed, then the equilibrium position of the platform would rise a distance A
given by
kA = mc g.
This would also be the amplitude of the oscillations after the clay is released, so
mc g
Ai =
k
The frequency of oscillation of the plate without the clay is
s
1
k
f=
2 mp
The time for a complete oscillation is
T = 2
If the clay falls a distance h, then

mp
k

So
lut
ion
s

mp
1 2
gT = 2 2 g
= 2 2 H.
2
k
When the plate is at the stating point it is at rest. The clay will hit it with a speed given by
h=

v0 = gT.

Conservation of momentum in an inelastic collision will then result in a final speed of the clay + platform
system of
mc
vf = v0
.
mc + mp
The kinetic energy just after collision will be

1
(mc + mp )vf 2 .
2
So the amplitude of the resulting oscillations will be given by
K=

or

1
1
kAf 2 = (mc + mp )vf 2
2
2
r
mc + mp
Af = vf
k

Gluing stuff together

Af
Ai

r
vf mc + mp
,
Ai
k
r
mc + mp
v0
mc
=
.
,
Ai mc + mp
k
r
mc
gT k
mc + mp
.
,
=
mc g mc + mp
k
s
k
= T
mp + mc
=

And then, combining with our previous expression for T ,


s
r
mp
k
Af
= 2
,
Ai
k
mp + mc
r
mp
= 2
mp + mc
c
Copyright 2008
American Association of Physics Teachers

2008 Semifinal Exam

12

If, instead, the clay manages to hit the platform at the top of an oscillation, then the distance the clay
would fall would only be
h 2A
and the time required would be
r

mp
.
k

1
g
2

 2
T
,
2

T
=
2
Then
h 2A =
where that complicated looking thing is actually

1
h.
4
So
=

1
h + 2A,
4

3
h
8

A,

3 2
D
4

A.

So
lut
ion
s
h

But from the very first two equations,

A
3
mc
=
= 2 .
mp
D
4

In this scenario the clay passes the platform three times: once at the highest point, once some distance
further on, and once when the at the lowest point. That means that if A were smaller, it might be possible
to find a value of A such that the clay just barely touches the platform once before hitting the platform at
the bottom. Consequently, this is an over estimate for A, and an overestimate for mc , and therefore, and
overestimate for the ratio. A larger ratio would guarantee collision, the actual critical ratio would be smaller
than this.

c
Copyright 2008
American Association of Physics Teachers

2008 Semifinal Exam

13

Question B2
Consider a parallel plate capacitor with the plates vertical. The plates of the capacitor are rigidly supported
in place. The distance between the plates is d. The plates have height h and area A >> d2 . Assume
throughout this problem that air resistance may be neglected; however, the force of gravity cannot be
neglected.
d/2

String

Rigid Support

h
h

So
lut
ion
s

h/2

a. A metal ball with a mass M and a charge q is suspended from a string that is tied to a rigid support.
When the capacitor is not charged, the metal ball is located at the center of the capacitor (at a distance
d/2 from both plates and at a height h/2 above the bottom edge of the plates. If instead a constant
potential difference V0 is applied across the plates, the string will an angle 0 to the vertical when the
metal ball is in equilibrium.
i. Determine 0 in terms of the given quantities and fundamental constants.
ii. The metal ball is then lifted until it makes an angle to the vertical where is only slightly greater
than 0 . The metal ball is released from rest. Show that the resulting motion is simple harmonic
motion and find the period of the oscillations in terms of the given quantities and fundamental
constants.
iii. When the ball is at rest in its equilibrium position, the string is cut. What is the maximum value
for V0 so that the ball will not hit one of the plates before exiting? Express your answer in terms
of the given quantities and fundamental constants.
b. Suppose instead that the ball of mass M and charge q is released from rest at a point halfway between
the plates at a time t = 0. Now, an AC potential difference V (t) = V0 sin t is also placed across
the capacitor. For what range of angular frequencies will the ball not hit either plate before it falls,
under the influence of gravity, out of the region between the plates? Consider only two scenarios:
either g >> h 2 or g << h 2 . Express your answer in terms of the given quantities and fundamental
constants.
Solution
The electric field between the plates is given by
E = V0 /d
There are then three forces on the hanging ball: electrostatic, gravitational, and the tension in the string.
They are
FE = qE = qV0 /d
and
Fg = M g
and then tension FT .
c
Copyright 2008
American Association of Physics Teachers

2008 Semifinal Exam

14

In equilibrium, the string must make an angle such that the horizontal and vertical components of the
tension balance the electrostatic and gravitational forces. Consequently,
tan 0 =

FE
qV0
=
Fg
M gd

The tension in the string is given by


FT 2 = Fg 2 + FE 2 .
A simple pendulum oscillates with a period according to
s
L
T = 2
g

So
lut
ion
s

where g is the acceleration of free fall. In this case, if the string were cut, the instantaneous acceleration of
the ball would be FT /M , so the period of small oscillations would be
r
ML
T = 2
FT
This is not a particularly pretty thing to substitute for, but yields
s
ML
T = 2 p
2
2
M g + V0 2 q 2 /d2

which can be simplified, somewhat, to give

T = 2

L
g

1+

V0 q
M gd

2 ! 14

An astute reader would recognize a few trigonometric identities and write


s
L cos 0
T = 2
g

which is certainly much more compact.


Upon cutting the string the ball will move in a straight line, tangent to the angle the string originally
made. It wont move with constant speed. So it will leave the region between the plates a distance
x1 = (L + h/2) tan 0

away from the center line. Set this equal to d/2, the condition that it hits one of the plates, and solve for
V0 :
d
2

(L + h/2)

V0

M gd2
(2L + h)q

qV0
M gd

If the ball is instead released from rest in the oscillating field then it will experience an oscillating force
in the x direction given by
V0
sin t.
FE =
d
Hence, the x component of the acceleration will be given by
ax =

qV0
sin t.
Md

c
Copyright 2008
American Association of Physics Teachers

2008 Semifinal Exam

15

The ball is released from rest, so this can be directly integrated to give the x component of the velocity
vx =

qV0
(1 cos t) .
M d

This can be directly integrated to find the position relative to the center, which we will define as x = 0:


1
qV0
t sin t
x=
M d

The ball hits one of the plates if this value exceeds d/2 while it is still in the region between the plates.
Solving for equality,
sin t
M d2
=t
2qV0

is the concern. t is given by the equation of a falling object, or

So
lut
ion
s

h
1
= gt2 ,
2
2

p
so t = h/g.
Unfortunately, this cant be solved for .
p
Consider first the condition that g << h 2 . Then it is reasonable to neglect 1/ compared to g/h,
and the equation simplifies to
M d2 p
= g/h
2qV0
with solution

2qV0 p
g/h
M d2
At higher frequencies the ball will miss, falling out of the plates before it hits either side.
Consider instead the condition that g >> h 2 . This is a slowly oscillating field, and one can then
approximate the sine function as
1
sin t t 3 t3 .
6
yielding
M d2
1
= 2 t3
2qV0
6
or
3M d2 p 3 3
h /g
2 =
qV0
1 =

as the second critical frequency. At lower frequencies the ball will not hit either side before it falls out of the
region between the plates.
Note that the product of these two angular frequencies is
1 2 = 6

h
g

c
Copyright 2008
American Association of Physics Teachers

2008 Semifinal Exam

16

Optical Society of America Bonus Question


Researchers have a developed a lens made of liquid. The spherical lens consists of a droplet of transparent
liquid resting on a electrically controllable surface. When the voltage of the surface is changed, the droplet
changes itself shape; it either tries to ball-up more strongly or it becomes flatter. Figure 1 is a sketch of
the liquid lens and several parameters that describe it, including the thickness of the lens (t), the radius
of curvature of the top surface (R) and the contact angle (), which represents the angle between the flat
surface beneath the droplet and the tangent to the curved surface at the point of contact.
a. When a certain voltage is applied, both the contact angle and lens thickness increase (and the lens
becomes more curved). In this case, is the liquid attracted or repelled by the surface?
b. Express the contact angle as a function of R and t.
c. The total volume of the liquid lens is an important parameter because as the liquid lens changes shape,
its volume is conserved. Calculate the volume of the lens as a function of R and t.

So
lut
ion
s

d. Use your result to part (b) to eliminate the variable t from your expression for the volume and find
V (R, ).
e. By changing the voltage on the control surface, the contact angle, , can be changed, which in turn
changes the focal length of the lens, f . The lensmakers formula can be used to calculate the focal
length and is given by


1
1
1
,

= (nliquid nair )
f
R1
R2
where nliquid and nair are the refractive indices of the liquid in the lens and air around it, and R1 and
R2 are the radii of curvature of the two surfaces of the lens. In figure 1, R1 is the curved face and R2
is the flat face. Use the lensmakers formula to calculate the focal length of the lens in terms of the
total volume of the liquid, the contact angle, and the relevant refractive indices.
Sidenote: liquid lenses are interesting because they are electrically controllable, variablefocus lenses
that can be very compact. People are working on putting them into cell phone cameras for ultracompact zoom lenses. For more information on this type of liquid lens, see T. Krupenkin, S. Yang,
and P. Mach, Tunable liquid microlens, Appl. Phys. Lett. 82, 316-318 (2003).

c
Copyright 2008
American Association of Physics Teachers

United States Physics Team

F = ma Contest
2009

2009 F = ma Exam

AAPT
AIP

UNITED STATES PHYSICS TEAM


2009
2009 F = ma Contest
25 QUESTIONS - 75 MINUTES
INSTRUCTIONS

DO NOT OPEN THIS TEST UNTIL YOU ARE TOLD TO BEGIN


Use g = 10 N/kg throughout this contest.
You may write in this booklet of questions. However, you will not receive any credit for anything
written in this booklet.
Your answer to each question must be marked on the optical mark answer sheet.
Select the single answer that provides the best response to each question. Please be sure to use a No. 2
pencil and completely fill the box corresponding to your choice. If you change an answer, the previous
mark must be completely erased.
Correct answers will be awarded one point; incorrect answers will result in a deduction of
There is no penalty for leaving an answer blank.

1
4

point.

A hand-held calculator may be used. Its memory must be cleared of data and programs. You may use
only the basic functions found on a simple scientific calculator. Calculators may not be shared. Cell
phones may not be used during the exam or while the exam papers are present. You may not use any
tables, books, or collections of formulas.
This test contains 25 multiple choice questions. Your answer to each question must be marked on the
optical mark answer sheet that accompanies the test. Only the boxes preceded by numbers 1 through
25 are to be used on the answer sheet.
All questions are equally weighted, but are not necessarily the same level of difficulty.
In order to maintain exam security, do not communicate any information about the
questions (or their answers or solutions) on this contest until after February 20, 2009.
The question booklet and answer sheet will be collected at the end of this exam. You may not use
scratch paper.

DO NOT OPEN THIS TEST UNTIL YOU ARE TOLD TO BEGIN

c
Copyright 2009
American Association of Physics Teachers

2009 F = ma Exam

1. A 0.3 kg apple falls from rest through a height of 40 cm onto a flat surface. Upon impact, the apple comes
to rest in 0.1 s, and 4 cm2 of the apple comes into contact with the surface during the impact. What is the
average pressure exerted on the apple during the impact? Ignore air resistance.
(A) 67,000 Pa
(B) 21,000 Pa CORRECT
(C) 6,700 Pa
(D) 210 Pa
(E) 67 Pa
The following information is used for questions 2 and 3.
Three blocks of identical mass are placed on a frictionless table as shown. The center block is at rest, whereas
the other two blocks are moving directly towards it at identical speeds v. The center block is initially closer
to the left block than the right one. All motion takes place along a single horizontal line.

2. Suppose that all collisions are instantaneous and perfectly elastic. After a long time, which of the following
is true?
(A) The center block is moving to the left.
(B) The center block is moving to the right.
(C) The center block is at rest somewhere to the left of its initial position.
(D) The center block is at rest at its initial position. CORRECT
(E) The center block is at rest somewhere to the right of its initial position.
3. Suppose, instead, that all collisions are instantaneous and perfectly inelastic. After a long time, which of
the following is true?
(A) The center block is moving to the left.
(B) The center block is moving to the right.
(C) The center block is at rest somewhere to the left of its initial position.
(D) The center block is at rest at its initial position.
(E) The center block is at rest somewhere to the right of its initial position. CORRECT
4. A spaceman of mass 80 kg is sitting in a spacecraft near the surface of the Earth. The spacecraft is
accelerating upward at five times the acceleration due to gravity. What is the force of the spaceman on the
spacecraft?
(A) 4800 N CORRECT
(B) 4000 N
(C) 3200 N
(D) 800 N
(E) 400 N

c
Copyright 2009
American Association of Physics Teachers

2009 F = ma Exam

5. Three equal mass satellites A, B, and C are in coplanar orbits around a planet as shown in the figure. The
magnitudes of the angular momenta of the satellites as measured about the planet are LA , LB , and LC .
Which of the following statements is correct?
A
B

(A) LA > LB > LC CORRECT


(B) LC > LB > LA

(C) LB > LC > LA


(D) LB > LA > LC
(E) The relationship between the magnitudes is different at various instants in time.
6. An object is thrown with a fixed initial speed v0 at various angles relative to the horizon. At some constant
height h above the launch point the speed v of the object is measured as a function of the initial angle .
Which of the following best describes the dependence of v on ? (Assume that the height h is achieved, and
assume that there is no air resistance.)
(A) v will increase monotonically with .
(B) v will increase to some critical value vmax and then decrease.
(C) v will remain constant, independent of . CORRECT
(D) v will decrease to some critical value vmin and then increase.
(E) None of the above.
7. A bird is flying in a straight line initially at 10 m/s. It uniformly increases its speed to 15 m/s while covering
a distance of 25 m. What is the magnitude of the acceleration of the bird?
(A) 5.0 m/s2
(B) 2.5 m/s2 CORRECT

(C) 2.0 m/s2

(D) 0.5 m/s2


(E) 0.2 m/s2

c
Copyright 2009
American Association of Physics Teachers

2009 F = ma Exam

The following information is used for questions 8 and 9.

Angular Velocity (rad/s)

A flat disk rotates about an axis perpendicular to the plane of the disk and through the center of the disk
with an angular velocity as shown in the graph below.

4
2
Time (s)

0
1

8. Determine the angular acceleration of the disk when t = 2.0 s.


(A) -12 rad/s2.
(B) -8 rad/s2 .
(C) -4 rad/s2 .
(D) -2 rad/s2 . CORRECT
(E) 0 rad/s2 .

9. Through what net angle does the disk turn during the 3 seconds?
(A) 9 rad.
(B) 8 rad.
(C) 6 rad.
(D) 4 rad.
(E) 3 rad. CORRECT
10. A person standing on the edge of a fire escape simultaneously launches two apples, one straight up with a
speed of 7 m/s and the other straight down at the same speed. How far apart are the two apples 2 seconds
after they were thrown, assuming that neither has hit the ground?
(A) 14 m
(B) 20 m
(C) 28 m CORRECT

(D) 34 m

(E) 56 m

c
Copyright 2009
American Association of Physics Teachers

2009 F = ma Exam

11. A 2.25kg mass undergoes an acceleration as shown below. How much work is done on the mass?

Acceleration (m/s/s)

4
2
Position (m)

0
2

10

12

(A) 36 J CORRECT
(B) 22 J

(C) 5 J
(D) -17 J
(E) -36 J
12. Batman, who has a mass of M = 100 kg, climbs to the roof of a 30 m building and then lowers one end of
a massless rope to his sidekick Robin. Batman then pulls Robin, who has a mass of m = 75 kg, up the roof
of the building. Approximately how much total work has Batman done after Robin is on the roof?
(A) 60 J
(B) 7 103 J

(C) 5 104 J CORRECT

(D) 600 J

(E) 3 104 J
13. Lucy (mass 33.1 kg), Henry (mass 63.7 kg), and Mary (mass 24.3 kg) sit on a lightweight seesaw at evenly
spaced 2.74 m intervals (in the order in which they are listed; Henry is between Lucy and Mary) so that the
seesaw balances. Who exerts the most torque (in terms of magnitude) on the seesaw? Ignore the mass of
the seesaw.
(A) Henry
(B) Lucy CORRECT
(C) Mary
(D) They all exert the same torque.
(E) There is not enough information to answer the question.

c
Copyright 2009
American Association of Physics Teachers

2009 F = ma Exam

14. A wooden block (mass M ) is hung from a peg by a massless rope. A speeding bullet (with mass m and
initial speed v0 ) collides with the block at time t = 0 and embeds in it. Let S be the system consisting of
the block and bullet. Which quantities are conserved between t = 10 s and t = +10 s?

(A) The total linear momentum of S.


(B) The horizontal component of the linear momentum of S.
(C) The mechanical energy of S.
(D) The angular momentum of S as measured about a perpendicular axis through the peg.
(E) None of the above are conserved. CORRECT
15. A 22.0 kg suitcase is dragged in a straight line at a constant speed of 1.10 m/s across a level airport floor
by a student on the way to the 40th IPhO in Merida, Mexico. The individual pulls with a 1.00 x 102 N
force along a handle with makes an upward angle of 30.0 degrees with respect to the horizontal. What is
the coefficient of kinetic friction between the suitcase and the floor?
(A) k = 0.013
(B) k = 0.394
(C) k = 0.509 CORRECT
(D) k = 0.866
(E) k = 1.055
16. Two identical objects of mass m are placed at either end of a spring of spring constant k and the whole
system is placed on a horizontal frictionless surface. At what angular frequency does the system oscillate?
p
k/m
(A)
p
(B)
2k/m CORRECT
p
(C)
k/2m
p
(D) 2 k/m
p
(E) k/m/2
17. You are given a standard kilogram mass and a tuning fork that is calibrated in Hz. You are also provided
with a complete collection of laboratory equipment, but none of it is calibrated in SI units. You do not know
the values of any fundamental constants. Which of the following quantities could you measure in SI units?
(A) The acceleration due to gravity.
(B) The speed of light in a vacuum.
(C) The density of room temperature water.
(D) The spring constant of a given spring. CORRECT
(E) The air pressure in the room.
c
Copyright 2009
American Association of Physics Teachers

2009 F = ma Exam

18. A simple pendulum of length L is constructed from a point object of mass m suspended by a massless string
attached to a fixed pivot point. A small peg is placed a distance 2L/3 directly below the fixed pivot point
so that the pendulum would swing as shown in the figure below. The mass is displaced 5 degrees from the
vertical and released. How long does it take to return to its starting position?

Fixed Pivot Point

L
Small Peg

Point Object of mass m


q 
q 
L
2
1
+
g
3
q 

Lg 2 + 23
q

Lg 1 + 31
q

Lg 1 + 3

q 
Lg 1 + 13 CORRECT

(A)
(B)
(C)
(D)
(E)

19. A certain football quarterback can throw a football a maximum range of 80 meters on level ground. What
is the highest point reached by the football if thrown this maximum range? Ignore air friction.
(A) 10 m
(B) 20 m CORRECT

(C) 30 m

(D) 40 m
(E) 50 m
20. Consider a completely inelastic collision between two lumps of space goo. Lump 1 has mass m and originally
moves directly north with a speed v0 . Lump 2 has mass 3m and originally moves directly east with speed
v0 /2. What is the final speed of the masses after the collision? Ignore gravity, and assume the two lumps
stick together after the collision.
(A) 7/16 v0

(B) 5/8 v0

(C) 13/8 v0 CORRECT

(D) 5/8 v0
p
(E) 13/8 v0

c
Copyright 2009
American Association of Physics Teachers

2009 F = ma Exam

The following information is used for questions 21 and 22.


Two stars orbit their common center of mass as shown in the diagram below. The masses of the two stars
are 3M and M . The distance between the stars is d.

21. What is the value of the gravitational potential energy of the two star system?
(A) GM
d
(B)

3GM 2
d

(C) GM
d2

(D) 3GM
d CORRECT
(E) 3GM
d2

22. Determine the period of orbit for the star of mass 3M .


q
d3
(A) GM
CORRECT
q
d3
(B) 3
4
GM
q
d3
(C) 3GM
q
d3
(D) 2 GM
q
d3
(E) 4 GM
23. A mass is attached to an ideal spring. At time t = 0 the spring is at its natural length and the mass is given
an initial velocity; the period of the ensuing (one-dimensional) simple harmonic motion is T . At what time
is the power delivered to the mass by the spring first a maximum?
(A) t = 0
(B) t = T /8
(C) t = T /4
(D) t = 3T /8 CORRECT
(E) t = T /2

c
Copyright 2009
American Association of Physics Teachers

2009 F = ma Exam

24. A uniform rectangular wood block of mass M , with length b and height a, rests on an incline as shown.
The incline and the wood block have a coefficient of static friction, s . The incline is moved upwards from
an angle of zero through an angle . At some critical angle the block will either tip over or slip down the
plane. Determine the relationship between a, b, and s such that the block will tip over (and not slip) at
the critical angle. The box is rectangular, and a 6= b.
b

(A) s > a/b


(B) s > 1 a/b

(C) s > b/a CORRECT


(D) s < a/b
(E) s < b/a 1
25. Two discs are mounted on thin, lightweight rods oriented through their centers and normal to the discs.
These axles are constrained to be vertical at all times, and the discs can pivot frictionlessly on the rods. The
discs have identical thickness and are made of the same material, but have differing radii r1 and r2 . The
discs are given angular velocities of magnitudes 1 and 2 , respectively, and brought into contact at their
edges. After the discs interact via friction it is found that both discs come exactly to a halt. Which of the
following must hold? Ignore effects associated with the vertical rods.

r1

r2

(A) 1 2 r1 = 2 2 r2
(B) 1 r1 = 2 r2
(C) 1 r1 2 = 2 r2 2
(D) 1 r1 3 = 2 r2 3 CORRECT
(E) 1 r1 4 = 2 r2 4

c
Copyright 2009
American Association of Physics Teachers

United States Physics Team


Quarter Final Contest
2009

2009 Quarter-final Exam

AAPT
AIP

UNITED STATES PHYSICS TEAM


2009

Quarterfinal Exam

DO NOT DISTRIBUTE THIS PAGE


Important Instructions for the Exam Supervisor
This examination consists of one part.
The first page that follows is a cover sheet. Examinees may keep the cover sheet during the
exam.
Allow 60 minutes to complete the exam. Examinees may read the cover sheet before beginning
the exam, but may not look at the examination questions until the 60 minute time period
begins.
The supervisor must collect all examination questions, including the cover sheet, at the end
of the exam, as well as any scratch paper used by the examinees. Examinees may not take
the exam questions. The examination questions may be returned to the students after March
8, 2009.
Examinees are allowed calculators, but they may not use symbolic math, programming, or
graphic features of these calculators. Calculators may not be shared and their memory must
be cleared of data and programs. Cell phones, PDAs or cameras may not be used during
the exam or while the exam papers are present. Examinees may not use any tables, books,
or collections of formulas.
The examinees will need to use a ruler for one of the questions on this exam.
They may not share rulers with other examinees.
AAPT must receive the students answer papers no later than Thursday, March 5, 2009.
Marking of papers will occur that weekend, and the semifinalists will be selected by March
8, 2009. It will not be possible to mark late papers.

c
Copyright 2009
American Association of Physics Teachers

2009 Quarter-final Exam

AAPT
AIP

UNITED STATES PHYSICS TEAM


2009

2009 Quarter-Final Exam


4 QUESTIONS - 60 MINUTES
INSTRUCTIONS
DO NOT OPEN THIS TEST UNTIL YOU ARE TOLD TO BEGIN
Show all your work. Partial credit will be given.
Start each question on a new sheet of paper. Put your name in the upper right-hand corner of
each page, along with the question number and the page number/total pages for this problem.
For example,
Doe, Jamie
Prob. 1 - P. 1/3
A ruler will be required on this exam.
A hand-held calculator may be used. Its memory must be cleared of data and programs. You
may use only the basic functions found on a simple scientific calculator. Calculators may not
be shared.
Cell phones may not be used during the exam or while the exam papers are present. You
may not use any tables, books, or collections of formulas.
Each of the four questions is worth 25 points. The questions are not necessarily of the same
difficulty. Good luck!
In order to maintain exam security, do not communicate any information about
the questions (or their answers or solutions) on this contest until after March 10,
2009.

c
Copyright 2009
American Association of Physics Teachers

2009 Quarter-final Exam

1. Below is an image of Fomalhaut b, the first extrasolar planet to be observed directly by visible
light, obtained by the Hubble Space Telescope.

Astronomy Picture of the Day 2008 November 14 http://antwrp.gsfc.nasa.gov/apod/ap081114.html


Image Credit: NASA, ESA, P. Kalas, J. Graham, E. Chiang, E. Kite (Univ. California, Berkeley), M.
Clampin (NASA/Goddard), M. Fitzgerald (Lawrence Livermore NL), K. Stapelfeldt, J. Krist (NASA/JPL)

The scale of the larger diagram is shown on the lower left; the 13 refers to the angle, in
arc-second, subtended by 100 AU at the distance of Fomalhaut. The scale of the inset can
be determined by size of the small box.
(a) A planet is in a circular orbit of radius R and period T . Derive an expression for the
mass of a star in terms of R, T , and the gravitational constant G. You may assume that
the mass of the planet is much, much less than the mass of the star.
(b) From the image, estimate the mass of the star Fomalhaut in solar masses. You may
assume that the orbit of Fomalhaut b is circular and ignore any errors associated with
the fact that the plane of the image is not coincident with the plane of the orbit. Recall
that the radius of the Earths orbit around the Sun is 1 AU. You do not need to do
an error analysis, but the number of significant digits in your answer ought reflect the
accuracy of your answer.

c
Copyright 2009
American Association of Physics Teachers

2009 Quarter-final Exam

Solution
The force between the star of mass M and the planet of mass m is given by
F =

GM m
R2

Assuming that M  m, then the center of mass of the system is located at the star, and therefore
the orbital radius of the planet is effectively R. For a circular orbit,
F =m

v2
(2R/T )2
R
=m
= 4 2 m 2 .
R
R
T

Equating,
GM
4 2 R
= 2 ,
2
T
R
or the more familiar Keplers law
GM
R3
=
2
T
4 2
Rearranging,
M = 4 2

GR3
T2

To find the mass of Fomalhaut b, Mb in terms of solar masses Ms , we simply require a ratio:
Rb 3 /Tb 2
Mb
=
,
Ms
Re 3 /Te 2
where b refers to the planet around Fomalhaut b, while e refers to Earth. Then
Mb
=
Ms

Rb
Re

3 

Te
Tb

2
.

From the diagram we can obtain the ratios. Note that the 13 arcsecond notation is not needed for
this computation, it simply is a comparison of the size of the object as seen from Earth. As such,
seen from Earth, the orbital dust ring subtends about the same angle as does the planet Jupiter!
A quick estimate yields Rb /Re 110, while slightly more work is required for the periods.
The inset box is 10 times the scale of the main image.
The planet moves 1.4 AU during the two year period. Since the orbital circumference is 690 AU,
that means it takes 980 years to complete an orbit. Then Tb /Te = 980, and Mb /Ms 1.4. Allowing
for 10% error in each measurement, the results would be between 0.87 and 2.2.
Finally, after marking the quarter-final answers, it was mildly entertaining to consider the range of
values obtained, from a minimum of 1019 (about 1011 kg, or the mass of a pile of rocks some 300
meters tall) to a maximum of 1025 (About 1000 times greater than the mass of the universe!).

c
Copyright 2009
American Association of Physics Teachers

2009 Quarter-final Exam

2. A ball of mass m is thrown vertically upward with a speed of v0 . The ball is subject to an
air resistance force that is proportional to the velocity, F = kv. The ball rises up to height
of h and then returns to the starting point after some total time tf . The acceleration of free
fall is g.
Determine the following:
(a) An expression for the velocity as a function of time in terms of any or all of the constants.
(b) Sketch the graph of velocity vs time. On the sketch indicate the time to rise to the
highest point, tr , and the total time of flight, tf . You do not need to find either tr or tf
at this point, but your graph should reflect relative positions of both.
(c) Find the time to rise, tr , to the highest point, h, in terms of any or all of the constants.

Solution
Two forces act on the ball: gravity, and air friction. They are not necessarily equal, so the net
force, and hence an expression for the acceleration, would be given by
ma = mg kv
where up is positive. We cant simply rearrange this to solve for v, since a is a variable. We can,
however, rearrange to prepare for integration, since a = dv/dt:


dv
k
= g+ v
dt
m
or

dv
 = dt.
k
g+m
v

Integrating both sides


m
ln
k
Assume t0 is zero. Then

mg/k + v
mg/k + v0


= (t t0 ).
k

v = (mg/k + v0 )e m t mg/k
Note that as t the ball will approach a constant speed of vt = mg/k. Of course, it will
probably hit the ground first, but if it were thrown up in the air over the edge of an infinitely deep
cliff....
The ball will rise until the velocity is zero, or when


vt
vt
= tr .
ln
g
vt + v0
or, if you prefer,


v0
vt
tr = ln 1 +
g
vt
To find the height we need to integrate one more time. Since v = dx/dt we can write


gt
dx = (vt + v0 )e vt vt dt
c
Copyright 2009
American Association of Physics Teachers

2009 Quarter-final Exam

which integrates quickly to




vt
h = (vt + v0 )
g



vg tr


1 (vt tr )

Gluing together with the expression for tr ,




v0
vt v0 vt 2

ln 1 +
.
h=
g
g
vt
Showing that it reduces to h = v02 /2g in the limit as k 0 is left as an exercise for the reader.
3. A parallel plate
capacitor is made of two square parallel plates of area A, and separated by
a distance d  A. The capacitor is connected to a battery with potential V and allowed
to fully charge. The battery is then disconnected. A square metal conducting slab also with
area A but thickness d/2 is then fully inserted between the plates, so that it is always parallel
to the plates. How much work has been done on the metal slab while it is being inserted?

Solution
The capacitance of a thin rectangular capacitor is given by
C = 0 A/d
If you forgot this, then derive it. The electric field between the plates is given by
E=

Q
0 Ad

so the potential difference between the plates must be


V = Ed =

Q
0 A

and the capacitance is


C = Q/V = 0 A/d
For the original capacitor well call this C0 .
The energy stored in a capacitor is U = 12 QV = 12 CV0 2 , so once charged, the initial energy is
U0 =

1 0 A 2
V
2 d 0

Note, we are going to need to know the initial charge sooner or later, which is
Q0 = C0 V0 =

0 AV0
d

Inserting the conducting plate can be treated two ways: it results in the creation of two capacitors,
in series, each with area A but differing separation. The net charge Q0 wont change, thats why
we needed it above, but the resulting capacitance of the system will be
1/Ce = 1/C1 + 1/C2
c
Copyright 2009
American Association of Physics Teachers

2009 Quarter-final Exam

For our capacitors, the only thing that changes is d, which well call x1 and x2 . Then
1/Ce = x1 /0 A + x2 /0 A
which gives us
Ce =

0 A
= 2C0
d/2

Since the new total separation x1 + x2 is just d d/2.


The energy in the system after inserting the plate is
1
1
U = Q0 V = Q20 /Ce
2
2
since V is not constant!
Finally,
1
1
U = Q20 /2C0 = U0
2
2
The work done on the plate is the same as the work done on the system, which is given by
W = U U0
or

1 0 A 2
V .
4 d 0
This means that the plate is sucked into the original capacitor, and we have to fight to hold it
out.
W =

4. A certain electric battery is not quite ideal. It can be thought of as a perfect cell with constant
output voltage V0 connected in series to a resistance r, but there is no way to remove this
internal resistance from the battery.
R
Battery
V0

N parallel bulbs (4 are shown)

The battery is connected to N identical lightbulbs in parallel. The bulbs each have a fixed
resistance R, independent of the current through them.
(a) Derive an expression for the total power dissipated by the N bulbs in terms of r, R, and
V0 .
c
Copyright 2009
American Association of Physics Teachers

2009 Quarter-final Exam

(b) It is observed that the configuration dissipates more total power through the bulbs when
N = 5 than it does for any other value of N . In terms of R, what range of values is
possible for r?

Solution
The effective resistance of N identical resistors in parallel is
Re = R/N
The total resistance of the circuit is the r + R/N .
The current drawn is then
I=

V0
.
r + R/N

The power dissipated by the parallel resistance is given by


P = I 2 Re ,
so
P =

R
V0 2
V0 2
=
N
R
(r + R/N )2 N
(N r + R)2

One can simply start with the fact that the maximum power is dissipated when the external
resistance is equal to the internal resistance. Or, take the derivative of P with respect to R and set
it equal to zero. In either case, you will get
r = Re =

R
N

Dont assume, however, that the answer is then r = R/5. Since we cant have half a bulb, we
dont actually know that the power is a maximum for the circuit, only that it is the maximum
configuration. It could be that a larger power output could occur with a resistance between R/4
and R/5, or maybe between R/5 and R/6.
A rough guess would be to assume
R
R
r
5.5
4.5
A better estimate would be to assume a quadratic function of 1/N with respect to P in the vicinity
of Pmax (think Taylor expansion of P ), and then the important point is the halfway point between
1/N and 1/(N 1), or
1
1
N 1/2
1/N + 1/(N 1)
=
+
=
2
2N
2(N 1)
N (N 1)
So the bounds are

4.5R
5.5R
r
30
20
The correct approach is to assume the power outputs at N and N 1 are equal, and solve for the
necessary value of r. Then
N
N 1
=
2
(N r + R)
(N r r + R)2
c
Copyright 2009
American Association of Physics Teachers

2009 Quarter-final Exam

or
N 3 r2 + 2N 2 (R r)r + N (R r)2 = N 3 r2 + 2N 2 Rr + N R2 N 2 r2 2N Rr R2
Thankfully, we loose the cubic term
2N 2 r2 2N Rr + N r2 = N 2 r2 2N Rr R2 .
To save space, flip the to the one term that doesnt have it.
N 2 r 2 N r 2 = R2
Only positive answers have meaning, so
1
R
r=p
N (N 1)
which give the correct bounds as
R
R
r
30
20
If you knew about geometric means, you might have been able to just write this down.

c
Copyright 2009
American Association of Physics Teachers

United States Physics Team


Semi Final Contest
2009

2009 Semifinal Exam

AAPT
AIP

UNITED STATES PHYSICS TEAM


2009

Semifinal Exam

DO NOT DISTRIBUTE THIS PAGE


Important Instructions for the Exam Supervisor
This examination consists of two parts.
Part A has four questions and is allowed 90 minutes.
Part B has two questions and is allowed 90 minutes.
The first page that follows is a cover sheet. Examinees may keep the cover sheet for both
parts of the exam.
The parts are then identified by the center header on each page. Examinees are only allowed
to do one part at a time, and may not work on other parts, even if they have time remaining.
Allow 90 minutes to complete Part A. Do not let students look at Part B. Collect the answers
to Part A before allowing the examinee to begin Part B. Examinees are allowed a 10 to 15
minutes break between parts A and B.
Allow 90 minutes to complete Part B. Do not let students go back to Part A.
Ideally the test supervisor will divide the question paper into 3 parts: the cover sheet (page
2), Part A (pages 3-4), and Part B (pages 6-7). Examinees should be provided parts A and
B individually, although they may keep the cover sheet.
The supervisor must collect all examination questions, including the cover sheet, at the end
of the exam, as well as any scratch paper used by the examinees. Examinees may not take
the exam questions. The examination questions may be returned to the students after March
31, 2009.
Examinees are allowed calculators, but they may not use symbolic math, programming, or
graphic features of these calculators. Calculators may not be shared and their memory must
be cleared of data and programs. Cell phones, PDAs or cameras may not be used during
the exam or while the exam papers are present. Examinees may not use any tables, books,
or collections of formulas.
Please provide the examinees with graph paper for Part A.

c
Copyright 2009
American Association of Physics Teachers

2009 Semifinal Exam

AAPT
AIP

Cover Sheet

UNITED STATES PHYSICS TEAM


2009

Semifinal Exam
INSTRUCTIONS
DO NOT OPEN THIS TEST UNTIL YOU ARE TOLD TO BEGIN
Work Part A first. You have 90 minutes to complete all four problems. Each question is
worth 25 points. Do not look at Part B during this time.
After you have completed Part A you may take a break.
Then work Part B. You have 90 minutes to complete both problems. Each question is worth
50 points. Do not look at Part A during this time.
Show all your work. Partial credit will be given. Do not write on the back of any page. Do
not write anything that you wish graded on the question sheets.
Start each question on a new sheet of paper. Put your school ID number, your name, the
question number and the page number/total pages for this problem, in the upper right hand
corner of each page. For example,
School ID #
Doe, Jamie
A1 - 1/3
A hand-held calculator may be used. Its memory must be cleared of data and programs. You
may use only the basic functions found on a simple scientific calculator. Calculators may not
be shared. Cell phones, PDAs or cameras may not be used during the exam or while the
exam papers are present. You may not use any tables, books, or collections of formulas.
Questions with the same point value are not necessarily of the same difficulty.
In order to maintain exam security, do not communicate any information about
the questions (or their answers/solutions) on this contest until after March 31,
2009.
Possibly Useful Information. You may
g = 9.8 N/kg
k = 1/40 = 8.99 109 N m2 /C2
c = 3.00 108 m/s
NA = 6.02 1023 (mol)1
= 5.67 108 J/(s m2 K4 )
1eV = 1.602 1019 J
me = 9.109 1031 kg = 0.511 MeV/c2
sin 16 3 for ||  1

use this sheet for both parts of the exam.


G = 6.67 1011 N m2 /kg2
km = 0 /4 = 107 T m/A
kB = 1.38 1023 J/K
R = NA kB = 8.31 J/(mol K)
e = 1.602 1019 C
h = 6.63 1034 J s = 4.14 1015 eV s
(1 + x)n 1 + nx for |x|  1
cos 1 21 2 for ||  1

c
Copyright 2009
American Association of Physics Teachers

2009 Semifinal Exam

Part A

Part A
Question A1
A hollow cylinder has length l, radius r, and thickness d, where l  r  d, and is made of a
material with resistivity . A time-varying current I flows through the cylinder in the tangential
direction. Assume the current is always uniformly distributed along the length of the cylinder. The
cylinder is fixed so that it cannot move; assume that there are no externally generated magnetic
fields during the time considered for the problems below.

I
r

a. What is the magnetic field strength B inside the cylinder in terms of I, the dimensions of the
cylinder, and fundamental constants?
b. Relate the emf E developed along the circumference of the cylinder to the rate of change of
the current dI
dt , the dimensions of the cylinder, and fundamental constants.
c. Relate E to the current I, the resistivity , and the dimensions of the cylinder.
d. The current at t = 0 is I0 . What is the current I(t) for t > 0?

Solution
The magnetic field through the inside of the cylinder is given by
B = 0 I/l
so the magnetic flux is
B = BA = 0 r2 I/l
The inductance is then
L = B /I = 0 r2 /l
Induced emf as a function of changing current is then
E = L

dI
0 r2 dI
=
dt
l dt

But that induced emf will be driving the current, so


E = IR

c
Copyright 2009
American Association of Physics Teachers

2009 Semifinal Exam

Part A

where R is the resistance, given by


Length
Area
Here the legnth is the circumference, 2r, while the area is the cross sectional area of the conductor,
ld. Therefore,
2r
E = I
ld
Combining the above, we get a differential equation,
R=

2r
0 r2 dI
=
ld
l dt

which can be written more simply as


I =
where

dI
dt

2
0 rd

=
and the solution is then

I(t) = I(0)et

Question A2
A mixture of 32 P and 35 S (two beta emitters widely used in biochemical research) is placed next to
a detector and allowed to decay, resulting in the data below. The detector has equal sensitivity to
the beta particles emitted by each isotope, and both isotopes decay into stable daughters.
You should analyze the data graphically. Error estimates are not required.
Day
0
5
10
20
30

Activity
64557
51714
41444
27020
18003

Day
40
60
80
100
150

a. Determine the half-life of each isotope.


b. Determine the ratio of the number of
sample.

Activity
12441
6385
3855
2734
1626

35 S

32 P

Day
200
250
300

Activity
1121
673
467

has a significantly longer half-life than

atoms to the number of

35 S

32 P.

atoms in the original

Question A3
Two stars, each of mass M and separated by a distance d, orbit about their center of mass. A
planetoid of mass m (m  M ) moves along the axis of this system perpendicular to the orbital
plane.

c
Copyright 2009
American Association of Physics Teachers

2009 Semifinal Exam

Part A

axis perpendicular to plane of orbit

Let Tp be the period of simple harmonic motion for the planetoid for small displacements from
the center of mass along the z-axis, and let Ts be the period of motion for the two stars. Determine
the ratio Tp /Ts .
This problem was adapted from a problem by French in Newtonian Mechanics.

Solution
This problem starts out similar to the quarterfinal exam.
The center of mass of the two stars is given by
R = d/2.
The force of attraction between the two stars is given by
F = GM 2 /d2
The orbital period T will be given by the centripetal force law
4 2 R
M2
=
G
T2
d2
or
r
r
d3
R3
Ts = 2
= 4
2GM
GM

The planetoid is a distance z above the plane. The distance to either star is then R2 + z 2 .
Only the part of the force that is perpendicular to the plane survives the vector addition, so the
net force is given by
GmM
z

F =2 2
.
2
2
R +z
R + z2
This expression is exact. But if z  d, then we can approximate this as
F =M

GmM
z
R3
This is a restoring force; the planetoid will execute simple harmonic oscillations with period
r
m
Tp = 2
k
F 2

where k is the effective force constant, or


r
Tp = 2

R3
= 2
GM

d3
8GM

so
Tp /Ts = 1/2
c
Copyright 2009
American Association of Physics Teachers

2009 Semifinal Exam

Part A

Question A4
A potato gun fires a potato horizontally down a half-open cylinder of cross-sectional area A. When
the gun is fired, the potato slug is at rest, the volume between the end of the cylinder and the
potato is V0 , and the pressure of the gas in this volume is P0 . The atmospheric pressure is Patm ,
where P0 > Patm . The gas in the cylinder is diatomic; this means that Cv = 5R/2 and Cp = 7R/2.
The potato moves down the cylinder quickly enough that no heat is transferred to the gas. Friction
between the potato and the barrel is negligible and no gas leaks around the potato.
potato

closed end

open end

The parameters P0 , Patm , V0 , and A are fixed, but the overall length L of the barrel may be
varied.
a. What is the maximum kinetic energy Emax with which the potato can exit the barrel? Express
your answer in terms of P0 , Patm , and V0 .
b. What is the length L in this case? Express your answer in terms of P0 , Patm , V0 , and A.

Solution
So long as the pressure inside the cylinder is greater than the external air pressure, the potato
will accelerate. Therefore, maximum energy will be transferred to the potato if the cylinder is
exactly long enough so that the final pressure inside the volume of the cylinder is Patm .
The energy of an ideal diatomic gas is given by
Cv nRT
but, by the ideal gas law,
P V = nRT
so the energy in the gas is
Cv P V
for any volume and pressure. Maximum energy is delivered to the potato when the final pressure
is atmospheric, so the work done by the gas on the potato is
Cv (P0 V0 Patm Vf )
But the potato is moving against air, so the actual energy given to the potato is
Emax = Cv (P0 V0 Patm Vf ) Patm (Vf V0 )
The relationship determining Vf is that of adiabatic expansion,

Vf = V0

P0
Patm

1/
,

c
Copyright 2009
American Association of Physics Teachers

2009 Semifinal Exam

Part A

where = Cp /Cv . Things dont simplify much, unless we use the numerical values of Cv and .
Then
5
7
Emax = P0 V0 Patm Vf + Patm V0 ,
2
2
or


5
7
2/7
5/7
V0
Emax =
P0 + Patm Patm P0
2
2
The length of the tube is simply
V0
L = Vf /A =
A

P0
Patm

5/7
.

c
Copyright 2009
American Association of Physics Teachers

2009 Semifinal Exam

Part A

STOP: Do Not Continue to Part B

If there is still time remaining for Part A, you should review your work for
Part A, but do not continue to Part B until instructed by your exam
supervisor.

c
Copyright 2009
American Association of Physics Teachers

2009 Semifinal Exam

Part B

Part B
Question B1
A bowling ball and a golf ball are dropped together onto a flat surface from a height h. The bowling
ball is much more massive than the golf ball, and both have radii much less than h. The bowling
ball collides with the surface and immediately thereafter with the golf ball; the balls are dropped
so that all motion is vertical before the second collision, and the golf ball hits the bowling ball at
an angle from its uppermost point, as shown in the diagram. All collisions are perfectly elastic,
and there is no surface friction between the bowling ball and the golf ball.

l
After the collision the golf ball travels in the absence of air resistance and lands a distance l
away. The height h is fixed, but may be varied. What is the maximum possible value of l, and
at what angle is it achieved?
You may present your results as decimals, but remember that you are not allowed to use
graphical or algebraic functions of your calculator.

Solution
Both balls arrive at the surface with a speed v0 , which we can determine as usual via conservation
of energy:
1
mv 2 = mgh
2 0
p
v0 = 2gh
After the bowling ball collides with the surface, it is traveling upwards at v0 and the golf ball
is traveling downwards at v0 . The subsequent collision is most easily understood in the reference
c
Copyright 2009
American Association of Physics Teachers

2009 Semifinal Exam

Part B

10

frame of the bowling ball; in this frame, the golf ball is traveling downwards at 2v0 and the bowling
ball is stationary. Since the bowling ball is very massive compared to the golf ball, the golf ball
will rebound at the same speed 2v0 . Since there is no surface friction, the angle of reflection will be
equal to the angle of incidence. Therefore the golf ball will emerge at an angle 2 to the vertical.
In the bowling balls reference frame, then, the golf ball emerges with horizontal velocity
2v0 sin 2 and upward vertical velocity 2v0 cos 2. Transforming these back to the original frame
of reference, we obtain the initial conditions for the golf balls projectile motion:
vx = 2v0 sin 2
vy = 2v0 cos 2 + v0
Meanwhile, the time of flight t of the golf ball is given by
1
vy t gt2 = 0
2
t=

2vy
g

and therefore the range is


l = vx t
2vx vy
l=
g
(This is a well-known result, and students may quote it directly.)
Combining with our previous result,
l=

2
(2v0 sin 2)(2v0 cos 2 + v0 )
g

l=

1
8v0 2
sin 2 (cos 2 + )
g
2

and inserting the result for v0 ,

1
l = 16h sin 2 (cos 2 + )
2
As a convenience set = 2. Then
1
l = 16h sin (cos + )
2


dl
1
2
= 16h cos (cos + ) sin
d
2

Setting this to zero to find the maximum,


cos2 sin2 +
2 cos2 +

cos =

1
cos = 0
2

1
cos 1 = 0
2
q
12 14 8
4

c
Copyright 2009
American Association of Physics Teachers

2009 Semifinal Exam

Part B

Taking the positive root,

cos =

33 1
= 0.593
8

and
sin =
So at maximum

p
1 cos2 = 0.805

1
l = 16h 0.805 (0.593 + )
2
l = 14.08 h

This occurs at
cos 2 = 0.593
= 0.468 = 26.8

c
Copyright 2009
American Association of Physics Teachers

11

2009 Semifinal Exam

Part B

12

Question B2
An electric dipole consists of two charges of equal magnitude q and opposite sign, held rigidly apart
by a distance d. The dipole moment is defined by p = qd.
Now consider two identical, oppositely oriented electric dipoles, separated by a distance r, as
shown in the diagram.

d
r

a. It is convenient when considering the interaction between the dipoles to choose the zero of
potential energy such that the potential energy is zero when the dipoles are very far apart
from each other. Using this convention, write an exact expression for the potential energy of
this arrangement in terms of q, d, r, and fundamental constants.
b. Assume that d  r. Give an approximation of your expression for the potential energy
to lowest order in d. Rewrite this approximation in terms of only p, r, and fundamental
constants.
c. What is the force (magnitude and direction) exerted on one dipole by the other? Continue
to make the assumption that d  r, and again express your result in terms of only p, r, and
fundamental constants.
d. What is the electric field near dipole B produced by dipole A? Continue to make the assumption that d  r and express your result in terms of only p, r, and fundamental constants.

Solution
a. There are two +q - q pairs separated by a distance d, each having potential energy

q2
40 d

There are two +q - q pairs separated by a distance r, each having potential energy

q2
40 r

There are a +q - +q pair and a q - q pair separated by a distance r2 + d2 , each having


potential energy
q2

40 r2 + d2
Note that the latter two terms go to zero as r becomes large, whereas the first term is not
dependent on r. Thus the given zero convention will include only the latter two terms:


q2
2
2
U=
+
40
r
r2 + d2
c
Copyright 2009
American Association of Physics Teachers

2009 Semifinal Exam

b. We have

Part B

13

2q 2
1
q
U=
40 r
1+


d 2
r

Using the binomial approximation (1 + x)n 1 + nx,


2q 2
U
40 r

1
1
2

!
 2
d
1
r

q 2 d2
40 r3

p2
40 r3

or, in terms of p,

c. We can infer by symmetry that the force must be in the direction along the line separating the
dipoles. Since the potential energy decreases with decreasing distance, the force is attractive.
Its magnitude can be determined by taking the derivative of the potential energy:
F =

dU
p2
= 3
dr
40 r4

with the negative sign confirming that the force is attractive.


One can, of course, also use an approach analogous to the previous one, i.e. write down the
force exactly and use a binomial approximation as above. One must take care to account for
the fact that the force between like-signed charges is not exactly in the same direction as that
between opposite-signed charges.

c
Copyright 2009
American Association of Physics Teachers

United States Physics Team


F = ma Contest
2010

2010 F = ma Exam

AAPT
AIP

UNITED STATES PHYSICS TEAM


2010
2010 F = ma Contest

Entia non multiplicanda sunt praeter necessitatem

nt
ial

25 QUESTIONS - 75 MINUTES
INSTRUCTIONS

DO NOT OPEN THIS TEST UNTIL YOU ARE TOLD TO BEGIN


Use g = 10 N/kg throughout this contest.

You may write in this booklet of questions. However, you will not receive any credit for anything written in
this booklet.
Your answer to each question must be marked on the optical mark answer sheet.

de

Select the single answer that provides the best response to each question. Please be sure to use a No. 2 pencil
and completely fill the box corresponding to your choice. If you change an answer, the previous mark must
be completely erased.
1
4

point. There is

nfi

Correct answers will be awarded one point; incorrect answers will result in a deduction of
no penalty for leaving an answer blank.

A hand-held calculator may be used. Its memory must be cleared of data and programs. You may use only
the basic functions found on a simple scientific calculator. Calculators may not be shared. Cell phones may
not be used during the exam or while the exam papers are present. You may not use any tables, books, or
collections of formulas.

Co

This test contains 25 multiple choice questions. Your answer to each question must be marked on the optical
mark answer sheet that accompanies the test. Only the boxes preceded by numbers 1 through 25 are to be
used on the answer sheet.
All questions are equally weighted, but are not necessarily the same level of difficulty.
In order to maintain exam security, do not communicate any information about the questions
(or their answers or solutions) on this contest until after February 8, 2010.
The question booklet and answer sheet will be collected at the end of this exam. You may not use scratch
paper.

DO NOT OPEN THIS TEST UNTIL YOU ARE TOLD TO BEGIN

c
Copyright 2010
American Association of Physics Teachers

2010 F = ma Exam

Questions 1 to 3 refer to the figure below which shows a representation of the motion of a squirrel
as it runs in a straight-line along a telephone wire. The letters A through E refer to the indicated
times.

time

nt
ial

1. If the graph is a graph of POSITION vs. TIME, then the squirrel has the greatest speed at what time(s) or during
what time interval(s)?
(A) From A to B
(B) From B to C only
(C) From B to D CORRECT
(D) From C to D only
(E) From D to E

de

2. If, instead, the graph is a graph of VELOCITY vs. TIME, then the squirrel has the greatest speed at what time(s)
or during what time interval(s)?
(A) at B
(B) at C
(C) at D

nfi

(D) at both B and D CORRECT


(E) From C to D

3. If, instead, the graph is a graph of ACCELERATION vs. TIME and the squirrel starts from rest, then the squirrel
has the greatest speed at what time(s) or during what time interval?
(A) at B

Co

(B) at C CORRECT
(C) at D

(D) at both B and D


(E) From C to D

4. Two teams of movers are lowering a piano from the window of a 10 floor apartment building. The rope breaks
when the piano is 30 meters above the ground. The movers on the ground, alerted by the shouts of the movers
above, first notice the piano when it is 14 meters above the ground. How long do they have to get out of the way
before the piano hits the ground?
(A) 0.66 sec CORRECT
(B) 0.78 sec
(C) 1.67 sec
(D) 1.79 sec
(E) 2.45 sec

c
Copyright 2010
American Association of Physics Teachers

2010 F = ma Exam

5. Two projectiles are launched from a 35 meter ledge as shown in the diagram. One is launched from a 37 degree
angle above the horizontal and the other is launched from 37 degrees below the horizontal. Both of the launches
are given the same initial speed of v0 = 50 m/s.
Projectile 1

35 m

Projectile 2

nt
ial

The difference in the times of flight for these two projectiles, t1 t2 , is closest to
(A) 3 s
(B) 5 s
(C) 6 s CORRECT
(D) 8 s
(E) 10 s

6. A projectile is launched across flat ground at an angle to the horizontal and travels in the absence of air resistance.
It rises to a maximum height H and lands a horizontal distance R away. What is the ratio H/R?

(B) 2 tan
(D)
(E)

2
tan
1
2 tan
1
4 tan

CORRECT

nfi

(C)

de

(A) tan

7. Harry Potter is sitting 2.0 meters from the center of a merry-go-round when Draco Malfoy casts a spell that glues
Harry in place and then makes the merry-go-round start spinning on its axis. Harry has a mass of 50.0 kg and can
withstand 5.0 gs of acceleration before passing out. What is the magnitude of Harrys angular momentum when
he passes out?
(A) 200 kgm2 /s

Co

(B) 330 kgm2 /s


(C) 660 kgm2 /s

(D) 1000 kgm2 /s CORRECT


(E) 2200 kgm2 /s

8. A car attempts to accelerate up a hill at an angle to the horizontal. The coefficient of static friction between the
tires and the hill is > tan . What is the maximum acceleration the car can achieve (in the direction upwards
along the hill)? Neglect the rotational inertia of the wheels.
(A) g tan
(B) g( cos sin ) CORRECT
(C) g( sin )
(D) g cos
(E) g( sin cos )

c
Copyright 2010
American Association of Physics Teachers

2010 F = ma Exam

9. A point object of mass M hangs from the ceiling of a car from a massless string of length L. It is observed to make
an angle from the vertical as the car accelerates uniformly from rest. Find the acceleration of the car in terms of
, M , L, and g.

(A) M g sin
(C) g tan CORRECT
(D) g cot
(E) M g tan

nt
ial

(B) M gL tan

10. A block of mass m1 is on top of a block of mass m2 . The lower block is on a horizontal surface, and a rope can
pull horizontally on the lower block. The coefficient of kinetic friction for all surfaces is . What is the resulting
acceleration of the lower block if a force F is applied to the rope? Assume that F is sufficiently large so that the
top block slips on the lower block.
1

de

(A) a2 = (F g(2m1 + m2 ))/m2 CORRECT


(B) a2 = (F g(m1 + m2 ))/m2

nfi

(C) a2 = (F g(m1 + 2m2 ))/m2


(D) a2 = (F + g(m1 + m2 ))/m2
(E) a2 = (F g(m2 m1 ))/m2

Co

11. The three masses shown in the accompanying diagram are equal. The pulleys are small, the string is lightweight,
and friction is negligible. Assuming the system is in equilibrium, what is the ratio a/b? The figure is not drawn to
scale!
a
b

(A) 1/2
(B) 1

(C) 3
(D) 2

(E) 2 3 CORRECT

c
Copyright 2010
American Association of Physics Teachers

2010 F = ma Exam

12. A ball with mass m projected horizontally off the end of a table with an initial kinetic energy K. At a time t after
it leaves the end of the table it has kinetic energy 3K. What is t? Neglect air resistance.
p
(A) (3/g) K/m
p
(B) (2/g) K/m CORRECT
p
(C) (1/g) 8K/m
p
(D) (K/g) 6/m
p
(E) (2K/g) 1/m

nt
ial

13. A ball of mass M and radius R has a moment of inertia of I = 25 M R2 . The ball is released from rest and rolls
down the ramp with no frictional loss of energy. The ball is projected vertically upward off a ramp as shown in
the diagram, reaching a maximum height ymax above the point where it leaves the ramp. Determine the maximum
height of the projectile ymax in terms of h.

(C)
(D)
(E)

25
49 h
2
5h
5
7 h
7
5h

CORRECT

nfi

(B)

de

(A) h

14. A 5.0 kg block with a speed of 8.0 m/s travels 2.0 m along a horizontal surface where it makes a head-on, perfectly
elastic collision with a 15.0 kg block which is at rest. The coefficient of kinetic friction between both blocks and
the surface is 0.35. How far does the 15.0 kg block travel before coming to rest?
(A) 0.76 m

(B) 1.79 m CORRECT

Co

(C) 2.29 m
(D) 3.04 m
(E) 9.14 m

c
Copyright 2010
American Association of Physics Teachers

2010 F = ma Exam

The following figure is used for questions 15 and 16.

v0

A small block of mass m is moving on a horizontal table surface at initial speed v0 . It then moves smoothly onto
a sloped big block of mass M . The big block can also move on the table surface. Assume that everything moves
without friction.

(A) h =

v02
2g

(B) h =

2
1 M v0
g m+M

(C) h =

2
1 M v0
2g m+M

nt
ial

15. A small block moving with initial speed v0 moves smoothly onto a sloped big block of mass M . After the small
block reaches the height h on the slope, it slides down. Find the height h.

CORRECT

mv02

(D) h =

1
2g m+M

(E) h =

v02
g

(A) v = v0
m
m+M v0

(C) v =

M
m+M v0

(D) v =

M m
m v0
M m
m+M v0

(E) v =

CORRECT

nfi

(B) v =

de

16. Following the previous set up, find the speed v of the small block after it leaves the slope.

17. Four masses m are arranged at the vertices of a tetrahedron of side length a. What is the gravitational potential
energy of this arrangement?
2

(B) 3 Gm
a

(C) 4 Gm
a

Co

(A) 2 Gm
a

(D) 6 Gm
CORRECT
a
(E) 12 Gm
a

c
Copyright 2010
American Association of Physics Teachers

2010 F = ma Exam

nt
ial

The following graph of potential energy is used for questions 18 through 20.

(B)

Co

nfi

(A)

de

18. Which of the following represents the force corresponding to the given potential?

(C)

(D)

(E) CORRECT

c
Copyright 2010
American Association of Physics Teachers

2010 F = ma Exam

19. Consider the following graphs of position vs. time.

I.

II.

III.

nt
ial

Which of the graphs could be the motion of a particle in the given potential?
(A) I
(B) III
(C) I and II
(D) I and III CORRECT
(E) I, II, and III

Co

nfi

de

20. Consider the following graph of position vs. time, which represents the motion of a certain particle in the given
potential.

What is the total energy of the particle?


(A) -5 J CORRECT
(B) 0 J
(C) 5 J

(D) 10 J
(E) 15 J

c
Copyright 2010
American Association of Physics Teachers

2010 F = ma Exam

21. The gravitational self potential energy of a solid ball of mass density and radius R is E. What is the gravitational
self potential energy of a ball of mass density and radius 2R?
(A) 2E
(B) 4E
(C) 8E
(D) 16E
(E) 32E CORRECT

A
B

de

(A) A
(B) B
(C) C

nfi

(D) D CORRECT
(E) Remains vertical

nt
ial

22. A balloon filled with helium gas is tied by a light string to the floor of a car; the car is sealed so that the motion
of the car does not cause air from outside to affect the balloon. If the car is traveling with constant speed along a
circular path, in what direction will the balloon on the string lean towards?

23. Two streams of water flow through the U-shaped tubes shown. The tube on the left has cross-sectional area A, and
the speed of the water flowing through it is v; the tube on the right has cross-sectional area A0 = 1/2A. If the net
force on the tube assembly is zero, what must be the speed v 0 of the water flowing through the tube on the right?

Co

Neglect gravity, and assume that the speed of the water in each tube is the same upon entry and exit.

(A) 1/2v

(B) v

(C) 2v CORRECT
(D) 2v
(E) 4v

c
Copyright 2010
American Association of Physics Teachers

2010 F = ma Exam

10

24. A uniform circular disk of radius R begins with a mass M ; about an axis through the center of the disk and
perpendicular to the plane of the disk the moment of inertia is I0 = 21 M R2 . A hole is cut in the disk as shown in
the diagram. In terms of the radius R and the mass M of the original disk, what is the moment of inertia of the
resulting object about the axis shown?

R/2

axis of rotation

(A) (15/32)M R2

nt
ial

(B) (13/32)M R2 CORRECT


(C) (3/8)M R2
(D) (9/32)M R2
(E) (15/16)M R2

nfi

de

25. Spaceman Freds spaceship (which has negligible mass) is in an elliptical orbit about Planet Bob. The minimum
distance between the spaceship and the planet is R; the maximum distance between the spaceship and the planet
is 2R. At the point of maximum distance, Spaceman Fred is traveling at speed v0 . He then fires his thrusters so
that he enters a circular orbit of radius 2R. What is his new speed?

2R

Co

p
3/2v0 CORRECT

(B) 5v0
p
(C)
3/5v0

(D) 2v0
(A)

(E) 2v0

c
Copyright 2010
American Association of Physics Teachers

United States Physics Team


Semi Final Contest
2010

2010 Semifinal Exam

AAPT
AIP

UNITED STATES PHYSICS TEAM


2010

Semifinal Exam

DO NOT DISTRIBUTE THIS PAGE


Important Instructions for the Exam Supervisor
This examination consists of two parts.
Part A has four questions and is allowed 90 minutes.
Part B has two questions and is allowed 90 minutes.
The first page that follows is a cover sheet. Examinees may keep the cover sheet for both
parts of the exam.
The parts are then identified by the center header on each page. Examinees are only allowed
to do one part at a time, and may not work on other parts, even if they have time remaining.
Allow 90 minutes to complete Part A. Do not let students look at Part B. Collect the answers
to Part A before allowing the examinee to begin Part B. Examinees are allowed a 10 to 15
minutes break between parts A and B.
Allow 90 minutes to complete Part B. Do not let students go back to Part A.
Ideally the test supervisor will divide the question paper into 3 parts: the cover sheet (page
2), Part A (pages 3-4), and Part B (pages 6-7). Examinees should be provided parts A and
B individually, although they may keep the cover sheet.
The supervisor must collect all examination questions, including the cover sheet, at the end
of the exam, as well as any scratch paper used by the examinees. Examinees may not take
the exam questions. The examination questions may be returned to the students after March
31, 2010.
Examinees are allowed calculators, but they may not use symbolic math, programming, or
graphic features of these calculators. Calculators may not be shared and their memory must
be cleared of data and programs. Cell phones, PDAs or cameras may not be used during
the exam or while the exam papers are present. Examinees may not use any tables, books,
or collections of formulas.

c
Copyright 2010
American Association of Physics Teachers

2010 Semifinal Exam

AAPT
AIP

Cover Sheet

UNITED STATES PHYSICS TEAM


2010

Semifinal Exam
INSTRUCTIONS
DO NOT OPEN THIS TEST UNTIL YOU ARE TOLD TO BEGIN
Work Part A first. You have 90 minutes to complete all four problems. Each question is
worth 25 points. Do not look at Part B during this time.
After you have completed Part A you may take a break.
Then work Part B. You have 90 minutes to complete both problems. Each question is worth
50 points. Do not look at Part A during this time.
Show all your work. Partial credit will be given. Do not write on the back of any page. Do
not write anything that you wish graded on the question sheets.
Start each question on a new sheet of paper. Put your school ID number, your name, the
question number and the page number/total pages for this problem, in the upper right hand
corner of each page. For example,
School ID #
Doe, Jamie
A1 - 1/3
A hand-held calculator may be used. Its memory must be cleared of data and programs. You
may use only the basic functions found on a simple scientific calculator. Calculators may not
be shared. Cell phones, PDAs or cameras may not be used during the exam or while the
exam papers are present. You may not use any tables, books, or collections of formulas.
Questions with the same point value are not necessarily of the same difficulty.
In order to maintain exam security, do not communicate any information about
the questions (or their answers/solutions) on this contest until after March 31,
2010.
Possibly Useful Information. You may
g = 9.8 N/kg
k = 1/40 = 8.99 109 N m2 /C2
c = 3.00 108 m/s
NA = 6.02 1023 (mol)1
= 5.67 108 J/(s m2 K4 )
1eV = 1.602 1019 J
me = 9.109 1031 kg = 0.511 MeV/c2
sin 16 3 for ||  1

use this sheet for both parts of the exam.


G = 6.67 1011 N m2 /kg2
km = 0 /4 = 107 T m/A
kB = 1.38 1023 J/K
R = NA kB = 8.31 J/(mol K)
e = 1.602 1019 C
h = 6.63 1034 J s = 4.14 1015 eV s
(1 + x)n 1 + nx for |x|  1
cos 1 21 2 for ||  1

c
Copyright 2010
American Association of Physics Teachers

2010 Semifinal Exam

Part A

Part A
Question A1
An object of mass m is sitting at the northernmost edge of a stationary merry-go-round of radius
R. The merry-go-round begins rotating clockwise (as seen from above) with constant angular
acceleration of . The coefficient of static friction between the object and the merry-go-round is
s .
a. Derive an expression for the magnitude of the objects velocity at the instant when it slides
off the merry-go-round in terms of s , R, , and any necessary fundamental constants.
b. For this problem assume that s = 0.5, = 0.2 rad/s2 , and R = 4 m. At what angle, as
measured clockwise from north, is the direction of the objects velocity at the instant when
it slides off the merry-go-round? Report your answer to the nearest degree in the range 0 to
360 .

Solution
The object will begin to slide when the force required to keep it accelerating according to the
motion of the merry-go-round exceeds the maximum static force of friction.
The angular speed of the merry-go-round is = t.
Since F = ma, we need consider the magnitude of the acceleration of the object. There are two
components, the tangential and radial.
at = R
ar = 2 R
p
a =
ar 2 + at 2
We also have for the static frictional force Ff s N , where N is the normal force and N = mg,
where g is the acceleration of gravity. Consequently,
ar 2 + at 2 = s 2 g 2
is the condition for slipping. Combining the above expressions,
2 R 2 + 4 R 2 = s 2 g 2 ,

2 R 2 1 + 2 t 4 = s 2 g 2 ,
s r
1 s 2 g 2
1
t =
2 R 2
Since the object moves off with a tangential velocity once it begins to slip, we have
q p
vt = R = Rt = R s 2 g 2 2 R2
The angular position made by the merry-go-round is then given by = 12 t2 , so
1 p 2 2
=
s g 2 R 2
2R
For the numbers given,
p
1
=
(0.5)2 (9.8)2 (0.2)2 (4)2 = 3.021rad,
2(0.2)(4)
or 173 degrees. Add 90 for the angle as measured from north, or 263 degrees.
c
Copyright 2010
American Association of Physics Teachers

2010 Semifinal Exam

Part A

Question A2
A spherical shell of inner radius a and outer radius b is made of a material of resistivity and
negligible dielectric activity. A single point charge q0 is located at the center of the shell. At time
t = 0 all of the material of the shell is electrically neutral, including both the inner and outer
surfaces. What is the total charge on the outer surface of the shell as a function of time for t > 0?
Ignore any effects due to magnetism or radiation; do not assume that b a is small.

Solution
The material of the shell will remain electrically neutral, although a charge Q will build up on
the inner surface while a charge of +Q will build up on the outer surface. By spherical symmetry
and Gausss law we can conclude that the electric field in the material of the shell will be given by
E=

1 q0 Q(t)
.
40
r2

This will cause a current density


J=

E
1 q0 Q(t)
=
.

40
r2

and therefore a current


I = JA =

1
(q0 Q)
0

where Q is still a function of time. But I = dQ/dt, so


dt
dQ
=
q0 Q
0
which can easily be integrated to yield

ln
or

q0
q0 Q


=

t
0

Q
= 1 et/0
q0

c
Copyright 2010
American Association of Physics Teachers

2010 Semifinal Exam

Part A

Question A3
A cylindrical pipe contains a movable piston that traps 2.00 mols of air. Originally, the air is at
one atmosphere of pressure, a volume V0 , and at a temperature of T0 = 298 K. First (process A)
the air in the cylinder is compressed at constant temperature to a volume of 14 V0 . Then (process
B) the air is allowed to expand adiabatically to a volume of V = 15.0 L. After this (process C)
this piston is withdrawn allowing the gas to expand to the original volume V0 while maintaining a
constant temperature. Finally (process D) while maintaining a fixed volume, the gas is allowed to
return to the original temperature T0 . Assume air is a diatomic ideal gas, no air flows into, or out
of, the pipe at any time, and that the temperature outside the remains constant always. Possibly
useful information: Cp = 72 R, Cv = 52 R, 1 atm = 1.01 105 Pa.
a. Draw a P-V diagram of the whole process.
b. How much work is done on the trapped air during process A?
c. What is the temperature of the air at the end of process B?

Solution
a. Isotherm, adiabat, isotherm, isochoric

b.
1
V1 = V0
4
For an ideal gas along an isotherm: PV = nRT
P1 V1 = P0 V0 =

P0
V1
=
V0
P1

W = nRT ln

V1
V0

J
W = 2 mols 8.31
298 K ln
molK

 
1
4

W = 6866 J (work done by the gas)


W = 6866 J (work done on the gas)
c. Adiabatic process
T1 V11 = T2 V21
T1 = T0 = 298 K from isothermal process in part b
 1
V1
T2 = T0
V2
1=

Cp
1=
Cv

7
2
5
2

R
7
2
1= 1=
5
5
R

c
Copyright 2010
American Association of Physics Teachers

2010 Semifinal Exam

Part A


T2 = 298 K

V1
V2

2
5

P0 V0 = nRT0
V0 =

nRT0
P0

P0 = 1 atm = 1.01 105 Pa


J 298 K
molK
1.01 105 Pa

2 mols 8.31
V0 =

V0 = 0.0490 m3
1
1
V1 = V0 = 0.0490 m3 = 0.0123 m3
4
4
1 m3
103 mL 1 cm3

6
= 0.0150 m3
1L
1 mL 10 cm3

2
0.0123 5
T2 = 298 K
0.0150

V2 = 15.0 L = 15.0 L

T2 = 275 K
d. Lowest Pressure From the P-V diagram, the lowest pressure occurs at the end of the second
isothermal process.
P3 V3 = nRT3
P3 =

nRT3
V3

T3 = T2 = 275 K
V3 = V0 = 0.0490 m3
J 275 K
molK
0.0490 m3

2.00 mols 8.31


P3 =

P3 = 9.33 104 Pa

c
Copyright 2010
American Association of Physics Teachers

2010 Semifinal Exam

Part A

Question A4
The energy radiated by the Sun is generated primarily by the fusion of hydrogen into helium-4. In
stars the size of the Sun, the primary mechanism by which fusion takes place is the proton-proton
chain. The chain begins with the following reactions:
2 p X1 + e+ + X2 (0.42 MeV)

(A4-1)

p + X1 X3 + (5.49 MeV)

(A4-2)

The amounts listed in parentheses are the total kinetic energy carried by the products, including
gamma rays. p is a proton, e+ is a positron, is a gamma ray, and X1 , X2 , and X3 are particles
for you to identify.
The density of electrons in the Suns core is sufficient that the positron is annihilated almost
immediately, releasing an energy x:
e+ + e 2 (x)

(A4-3)

Subsequently, two major processes occur simultaneously. The pp I branch is the single reaction
2 X3 4 He + 2 X4 (y),

(A4-4)

which releases an energy y. The pp II branch consists of three reactions:


X3 + 4 He X5 +

(A4-5)

X5 + e X6 + X7 (z)

(A4-6)

X6 + X4 2 4 He

(A4-7)

where z is the energy released in step A4-6.


a. Identify X1 through X7 . X2 and X7 are neutral particles of negligible mass. It is useful to
know that the first few elements, in order of atomic number, are H, He, Li, Be, B, C, N, O.
b. The mass of the electron is 0.51 MeV/c2 , the mass of the proton is 938.27 MeV/c2 , and
the mass of the helium-4 nucleus is 3 727.38 MeV/c2 . Find the energy released during the
production of one helium-4 nucleus, including the kinetic energy of all products and all energy
carried by gamma rays.
c. Find the unknown energies x and y above.
d. Step (A4-6) does not proceed as follows because there is insufficient energy.
X5 X6 + e+ + X7
What constraint does this fact place on z?
e. In which of the reaction steps is the energy carried by any given product the same every time
the step occurs? Assume that the kinetic energy carried in by the reactants in each step is
negligible, and that the products are in the ground state.

c
Copyright 2010
American Association of Physics Teachers

2010 Semifinal Exam

Part A

Solution
a. We know that in all nuclear processes, total charge is conserved, lepton number (electrons plus
neutrinos minus positrons minus antineutrinos), and baryon number (neutrons plus protons)
are conserved. As X2 is a neutral particle of negligible mass, X1 must have charge +1 and
contain two baryons. Thus it is 2 H. X2 is then a neutral particle with lepton number +1 and
is an electron neutrino.
Similar reasoning shows that X3 is 3 He, X4 is a proton (or 1 H, X5 is 7 Be, X6 is 7 Li, and X7
is an electron neutrino.
b. The overall reaction (not including gamma rays) is
4 p + 2 e 4 He + 2 e
as can be seen from conservation considerations or by combining the given reactions. The
energy released is the difference in mass between the reactants and the products; using the
given values, this is 26.72 MeV.
c. x is simply twice the mass of the electron, 1.02 MeV. To compute y, note that we can sum
the other known energies to obtain the result from the previous problem:
2(0.42 MeV + 5.49 MeV + 1.02 MeV) + y = 26.72 MeV
where the factor of 2 arises because two 3 He are produced in the course of the combined
reaction. Solving, y = 12.86 MeV.
d. The forbidden reaction produces an energy of z 2(0.51 MeV), as it differs from the naturally
occurring one by the consumption of one fewer electron and the production of an additional
positron. Since it is forbidden, we know that z 2(0.51 MeV) < 0; in other words, z <
1.02 MeV.
e. It is a well-known result that reactions with two products have a single set of product energies,
whereas those with three or more products produce a spectrum of output energies. Students
may quote this result; alternatively, observe that in a two-product reaction the conservation
of momentum and of energy give two equations in the two unknowns, fixing their values,
whereas there are insufficient equations in the case of three or more products.
The reactions with only two products are (A4-2), (A4-3), (A4-5), and (A4-7).

c
Copyright 2010
American Association of Physics Teachers

2010 Semifinal Exam

Part A

STOP: Do Not Continue to Part B

If there is still time remaining for Part A, you should review your work for
Part A, but do not continue to Part B until instructed by your exam
supervisor.

c
Copyright 2010
American Association of Physics Teachers

2010 Semifinal Exam

Part B

10

Part B
Question B1
A thin plank of mass M and length L rotates about a pivot at its center. A block of mass m  M
slides on the top of the plank. The system moves without friction. Initially, the plank makes an
angle 0 with the horizontal, the block is at the upper end of the plank, and the system is at rest.
Throughout the problem you may assume that  1, and that the physical dimensions of the
block are much, much smaller than the length of the plank.
L/2

Let x be the displacement of the block along the plank, as measured from the pivot, and let
be the angle between the plank and the horizontal. You may assume that centripetal acceleration
of the block is negligible compared with the linear acceleration of the block up and down the plank.
a. For a certain value of 0 , x = k throughout the motion, where k is a constant. What is this
value of 0 ? Express your answer in terms of M , m, and any fundamental constants that you
require.
b. Given that 0 takes this special value, what is the period of oscillation of the system? Express
your answer in terms of M , m, and any fundamental constants that you require.
c. Determine the maximum value of the ratio between the centripetal acceleration of the block
and the linear acceleration of the block along the plank, writing your answer in terms of m
and M , therefore justifying our approximation.

Solution
Eventually we will need to compute the rotational inertia of a long plank. The answer is
I=

1
M L2
12

and it is acceptable to just write it down with no work.


Assume the block is at point x away from the central pivot. The magnitude of the torque on
the plank is then given by
= ymg cos mgx
so the angular acceleration is
=

mg
x
I

c
Copyright 2010
American Association of Physics Teachers

2010 Semifinal Exam

Part B

11

It is fair to neglect the rotational inertia of the block, since m  M .


A similar expression exists for the linear acceleration of the board
a = g sin g
Assume that the motion of the block is given by x = L2 cos t, then the acceleration is given by
a = L2 2 cos t. Similarly, = 0 cos t, so = 0 2 cos t. Note that is not the angular
velocity of the plank!
Combine the equations and get two equation of the form
0 2 =
and

mg L
I 2

L 2
= g0
2

which can be combined to yield


0 2
L/2
02

mL
,
I 2
m
= 3
M
=

and
2g
=
L
2

r
m
3 .
M

We neglected centripetal acceleration of the block. Is this reasonable? It would be on the order of
 2
d
ac = x
= x0 2 2 sin 2 t.
dt
Now a = x 2 , so by considering the ratio of ac /a, we get
ac
= 0 2 sin2 t,
a
an expression which is always small.
The kinetic energy of the system is given by the sum of the energy of the block and the rotating
plank,

2


1
L
1 1
2
m
sin t +
M L (0 sin t)2
2
2
2 12
which can be simplified to
1
(L sin t)2
2

m M 2
+
0
4
12

using the above results, the kinetic energy reduces to


m
mgL0
(L sin t)2 =
sin2 t
4
2
The potential energy is given by the location of the block only, so
mgh = mgx = mg

L
mgL0
cos t0 cos t =
cos2 t.
2
2

c
Copyright 2010
American Association of Physics Teachers

2010 Semifinal Exam

Part B

12

Thankfully, this does result in the expected behavior of a system undergoing simple harmonic
motion. The kinetic energy equals the potential energy when t is equal to /4, so the position of
the block is

L 2
x=
2 2
and

2
= 0
2
so
r
L
m
L
y = x = 0 =
3
4
4
M
of course, if we had thought about this, we could have simply written down this last line without
worrying about computing the kinetic or potential energy at all!

c
Copyright 2010
American Association of Physics Teachers

2010 Semifinal Exam

Part B

13

Question B2
These three parts can be answered independently.
a. One pair of ends of two long, parallel wires are connected by a resistor, R = 0.25 , and a
fuse that will break instantaneously if 5 amperes of current pass through it. The other pair
of ends are unconnected. A conducting rod of mass m is free to slide along the wires under
the influence of gravity. The wires are separated by 30 cm, and the rod starts out 10 cm
from the resistor and fuse. The whole system is placed in a uniform, constant magnetic field
of B = 1.2 T as shown in the figure. The resistance of the rod and the wires is negligible.
When the rod is released is falls under the influence of gravity, but never loses contact with
the long parallel wires.

Resistor

Fuse

Sliding Rod

The magnetic field is


directed into the page

i. What is the smallest mass needed to break the fuse?


ii. How fast is the mass moving when the fuse breaks?
b. A fuse is composed of a cylindrical wire with length L and radius r  L. The resistivity
(not resistance!) of the fuse is small, and given by f . Assume that a uniform current I flows
through the fuse. Write your answers below in terms of L, r, f , I, and any fundamental
constants.
i. What is the magnitude and direction of the electric field on the surface of the fuse wire?
ii. What is the magnitude and direction of the magnetic field on the surface of the fuse
wire?
iii. The Poynting vector, ~S is a measure of the rate of electromagnetic energy flow through
~ B,
~
a unit surface area; the vector gives the direction of the energy flow. Since ~S = 1 E
0

~ and B
~ are the electric and magnetic
where 0 is the permeability of free space and and E
field vectors, find the magnitude and direction of the Poynting vector associated with
the current in the fuse wire.
c. A fuse will break when it reaches its melting point. We know from modern physics that a
hot object will radiate energy (approximately) according to the black body law P = AT 4 ,
where T is the temperature in Kelvin, A the surface area, and is the Stefan-Boltzmann
constant. If Tf = 500 K is the melting point of the metal for the fuse wire, with resistivity
f = 120 n m, and If = 5 A is the desired breaking current, what should be the radius of
the wire r?
c
Copyright 2010
American Association of Physics Teachers

2010 Semifinal Exam

Part B

14

Solution
a. Magnetic flux
B = BA
B = 1.2T
A = 0.1 m 0.3 m = 0.03 m2
B = 1.2 T 0.03 m2
B = 3.6 102 Wb
b. Mass needed to break fuse
F = iL B = iLB sin ( = 90 ) = iLB
F = 5 amps 0.3 m 1.2 T
F = 1.8 N
To cause a current of 5 amperes, a force of 1.8 N is required.
F = ma
m=

1.8 N
9.8 m
s2

m = 0.18 kg
c. A released mass of m = 0.18 kg will produce a 5 ampere current once it reaches a certain
velocity. That velocity is:
F = iLB
i=
F =
E=

E
R

E
LB
R

dB
dBA
dBLx
dx
=
=
= BL
= BLv
dt
dt
dt
dt
i=

v=

BLv
R

F =

B 2 L2 v
R

v=

FR
B 2 L2

1.8 N 0.25
m
= 3.47
2
2
(1.2 T) (0.3 m)
s

c
Copyright 2010
American Association of Physics Teachers

2010 Semifinal Exam

Part B

15

A quick application of E = V /L, where L is the length of the fuse wire, followed by V = IRf ,
yields
IRf
E=
.
L
The resistance of the fuse wire is Rf = f L/r2 , so
E=

If
r2

The magnetic field is found from Amperes law,


B=

0 I
2 r

and then the Poynting vector is


I 2 f
2r3
It is directed inward toward the wire, making it hotter.
The total power radiated inward is then found by multiplying by the surface area of a cylinder,
or
P = I 2 Rf .
S=

What a surprise.
Finally, the temperature increases until there is a power balance, so
Tf 4 =

If 2
2 2 r3

This can be solved to find the radius needed for the fuse wire,
s
(5A)2 (120 109 m)
r= 3
= 0.35 mm
2 2 (5.67 108 J/(s m2 K4 ))(500K)4

c
Copyright 2010
American Association of Physics Teachers

United States Physics Team


F = ma Contest Papers
2011

2011 F = ma Exam

AAPT
AIP

UNITED STATES PHYSICS TEAM


2011
2011 F = ma Contest
25 QUESTIONS - 75 MINUTES
INSTRUCTIONS

DO NOT OPEN THIS TEST UNTIL YOU ARE TOLD TO BEGIN


Use g = 10 N/kg throughout this contest.
You may write in this booklet of questions. However, you will not receive any credit for anything written in
this booklet.
Your answer to each question must be marked on the optical mark answer sheet.
Select the single answer that provides the best response to each question. Please be sure to use a No. 2 pencil
and completely fill the box corresponding to your choice. If you change an answer, the previous mark must
be completely erased.
Correct answers will be awarded one point; incorrect answers will result in a deduction of
no penalty for leaving an answer blank.

1
4

point. There is

A hand-held calculator may be used. Its memory must be cleared of data and programs. You may use only
the basic functions found on a simple scientific calculator. Calculators may not be shared. Cell phones may
not be used during the exam or while the exam papers are present. You may not use any tables, books, or
collections of formulas.
This test contains 25 multiple choice questions. Your answer to each question must be marked on the optical
mark answer sheet that accompanies the test. Only the boxes preceded by numbers 1 through 25 are to be
used on the answer sheet.
All questions are equally weighted, but are not necessarily the same level of difficulty.
In order to maintain exam security, do not communicate any information about the questions
(or their answers or solutions) on this contest until after February 20, 2011.
The question booklet and answer sheet will be collected at the end of this exam. You may not use scratch
paper.

DO NOT OPEN THIS TEST UNTIL YOU ARE TOLD TO BEGIN

c
Copyright 2011
American Association of Physics Teachers

2011 F = ma Exam

1. A cyclist travels at a constant speed of 22.0 km/hr except for a 20 minute stop. The cyclists average speed was
17.5 km/hr. How far did the cyclist travel?
(A) 28.5 km
(B) 30.3 km
(C) 31.2 km
(D) 36.5 km
(E) 38.9 km
The correct answer is A
Questions 2 to 4 refer to the three graphs below which show velocity of three objects as a function
of time. Each object is moving only in one dimension.

+2
0
2
0

4
6
time (s)

Object I

10

+4
velocity (m/s)

+4
velocity (m/s)

velocity (m/s)

+4

+2
0
2
0

4
6
time (s)

+2
0
2

10

Object II

2. Rank the magnitudes of the average acceleration during the ten second interval.
(A) I > II > III
(B) II > I > III
(C) III > II > I
(D) I > II = III
(E) I = II = III
The correct answer is E
3. Rank the magnitudes of the maximum velocity achieved during the ten second interval.
(A) I > II > III
(B) II > I > III
(C) III > II > I
(D) I > II = III
(E) I = II = III
The correct answer is D
4. Rank the magnitudes of the distance traveled during the ten second interval.
(A) I > II > III
(B) II > I > III
(C) III > II > I
(D) I = II > III
(E) I = II = III
The correct answer is B

c
Copyright 2011
American Association of Physics Teachers

4
6
time (s)

Object III

10

2011 F = ma Exam

5. A crude approximation is that the Earth travels in a circular orbit about the Sun at constant speed, at a distance
of 150,000,000 km from the Sun. Which of the following is the closest for the acceleration of the Earth in this orbit?
(A) exactly 0 m/s2
(B) 0.006 m/s2
(C) 0.6 m/s2
(D) 6 m/s2
(E) 10 m/s2
The correct answer is B
6. A child is sliding out of control with velocity vc across a frozen lake. He runs head-on into another child, initially
at rest, with 3 times the mass of the first child, who holds on so that the two now slide together. What is the
velocity of the couple after the collision?
(A) 2vc
(B) vc
(C) vc /2
(D) vc /3
(E) vc /4
The correct answer is E
7. An ice skater can rotate about a vertical axis with an angular velocity 0 by holding her arms straight out. She
can then pull in her arms close to her body so that her angular velocity changes to 20 , without the application of
any external torque. What is the ratio of her final rotational kinetic energy to her initial rotational kinetic energy?

(A) 2
(B) 2

(C) 2 2
(D) 4
(E) 8
The correct answer is B
8. When a block of wood with a weight of 30 N is completely submerged under water the buoyant force on the block
of wood from the water is 50 N. When the block is released it floats at the surface. What fraction of the block will
then be visible above the surface of the water when the block is floating?
(A) 1/15
(B) 1/5
(C) 1/3
(D) 2/5
(E) 3/5
The correct answer is D

c
Copyright 2011
American Association of Physics Teachers

2011 F = ma Exam

9. A spring has an equilibrium length of 2.0 meters and a spring constant of 10 newtons/meter. Alice is pulling on
one end of the spring with a force of 3.0 newtons. Bob is pulling on the opposite end of the spring with a force of
3.0 newtons, in the opposite direction. What is the resulting length of the spring?
(A) 1.7 m
(B) 2.0 m
(C) 2.3 m
(D) 2.6 m
(E) 8.0 m
The correct answer is C
10. Which of the following changes will result in an increase in the period of a simple pendulum?
(A) Decrease the length of the pendulum
(B) Increase the mass of the pendulum
(C) Increase the amplitude of the pendulum swing
(D) Operate the pendulum in an elevator that is accelerating upward
(E) Operate the pendulum in an elevator that is moving downward at constant speed.
The correct answer is C

water level

time
(B)

water level

time
(A)

water level

water level

water level

11. A large metal cylindrical cup floats in a rectangular tub half-filled with water. The tap is placed over the cup and
turned on, releasing water at a constant rate. Eventually the cup sinks to the bottom and is completely submerged.
Which of the following five graphs could represent the water level in the sink as a function of time?

time
(D)

time
(E)

The correct answer is C

c
Copyright 2011
American Association of Physics Teachers

time
(C)

2011 F = ma Exam

12. You are given a large collection of identical heavy balls and lightweight rods. When two balls are placed at the ends
of one rod and interact through their mutual gravitational attraction (as is shown on the left), the compressive
force in the rod is F . Next, three balls and three rods are placed at the vertexes and edges of an equilateral triangle
(as is shown on the right). What is the compressive force in each rod in the latter case?

(A)
(B)

1 F
3

3
2 F

(C) F

(D) 3F
(E) 2F
The correct answer is C
13. The apparatus in the diagram consists of a solid cylinder of radius 1 cm attached at the center to two disks of
radius 2 cm. It is placed on a surface where it can roll, but will not slip. A thread is wound around the central
cylinder. When the thread is pulled at the angle = 90 to the horizontal (directly up), the apparatus rolls to the
right. Which below is the largest value of for which it will not roll to the right when pulling on the thread?

(A) = 15
(B) = 30
(C) = 45
(D) = 60
(E) None, the apparatus will always roll to the right
The correct answer is D

c
Copyright 2011
American Association of Physics Teachers

2011 F = ma Exam

14. You have 5 different strings with weights tied at various point, all hanging from the ceiling, and reaching down to
the floor. The string is released at the top, allowing the weights to fall. Which one will create a regular, uniform
beating sound as the weights hit the floor?

(A)

(B)

(C)

(D)

(E)

The correct answer is D


15. A vertical mass-spring oscillator is displaced 2.0 cm from equilibrium. The 100 g mass passes through the equilibrium point with a speed of 0.75 m/s. What is the spring constant of the spring?
(A) 90 N/m
(B) 100 N/m
(C) 110 N/m
(D) 140 N/m
(E) 160 N/m
The correct answer is D

c
Copyright 2011
American Association of Physics Teachers

2011 F = ma Exam

Questions 16 and 17 refer to the information and diagram below.


Jonathan using rope to lift a box with Becky in
it; the box is hanging off the side of a bridge,
Jonathan is on the top. A rope is hooked up from
the box and passes a fixed railing; Jonathan holds
the box up by pressing the rope against the railing with a massless, frictionless physics textbook.
The static friction coefficient between the rope and
railing is s ; the kinetic friction coefficient between
the rope and railing is k < s ; the mass of the box
and Becky combined is M ; and the initial height
of the bottom of the box above the ground is h.
Assume a massless rope.

Loose rope

fixed hard
railing

Jonathan, pushes
on book against
rope

Floor

Becky

16. What magnitude force does Jonathan need to exert on the physics book to keep the rope from slipping?
(A) M g
(B) k M g
(C) k M g/s
(D) (s + k )M g
(E) M g/s
The correct answer is E
17. Jonathan applies a normal force that is just enough to keep the rope from slipping. Becky makes a small jump,
barely leaving contact with the floor of the box. Upon landing on the box, the force of the impact causes the rope
to start slipping from Jonathans hand. At what speed does the box smash into the ground? Assume Jonathans
normal force does not change.

(A) 2gH(k /s )

(B) 2gH(1 k /s )
p

(C) 2gH k /s
p

(D) 2gH (1 k /s )

(E) 2gH(s k )
The correct answer is D
18. A block of mass m = 3.0 kg slides down one ramp, and then up a second ramp. The coefficient of kinetic friction
between the block and each ramp is k = 0.40. The block begins at a height h1 = 1.0 m above the horizontal.
Both ramps are at a 30 incline above the horizontal. To what height above the horizontal does the block rise on
the second ramp?
(A) 0.18 m
(B) 0.52 m
(C) 0.59 m
(D) 0.69 m
(E) 0.71 m
The correct answer is A

c
Copyright 2011
American Association of Physics Teachers

2011 F = ma Exam

Questions 19 and 20 refer to the following information


A particle of mass 2.00 kg moves under a force given by
~ = (8.00 N/m)(xi + yj)
F
where i and j are unit vectors in the x and y directions. The particle is placed at the origin with an initial velocity
~v = (3.00 m/s)i + (4.00 m/s)j.
19. After how much time will the particle first return to the origin?
(A) 0.785 s
(B) 1.26 s
(C) 1.57 s
(D) 2.00 s
(E) 3.14 s
The correct answer is C
20. What is the maximum distance between the particle and the origin?
(A) 2.00 m
(B) 2.50 m
(C) 3.50 m
(D) 5.00 m
(E) 7.00 m
The correct answer is B
21. An engineer is given a fixed volume Vm of metal with which to construct a spherical pressure vessel. Interestingly,
assuming the vessel has thin walls and is always pressurized to near its bursting point, the amount of gas the vessel
can contain, n (measured in moles), does not depend on the radius r of the vessel; instead it depends only on Vm
(measured in m3 ), the temperature T (measured in K), the ideal gas constant R (measured in J/(K mol)), and the
tensile strength of the metal (measured in N/m2 ). Which of the following gives n in terms of these parameters?

(C) n =

2 Vm
3 RT

2 3 Vm
3 RT

2 3 Vm 2
3 RT

(D) n =

2
3

(E) n =

2
3

(A) n =
(B) n =

Vm 2
RT

q
3

Vm 2
RT

The correct answer is A

c
Copyright 2011
American Association of Physics Teachers

2011 F = ma Exam

Output Torque (Nm)

22. This graph depicts the torque output of a hypothetical gasoline engine as a function of rotation frequency. The
engine is incapable of running outside of the graphed range.

30

20

10

0
1,000

2,000

II

III

Engine Revolutions per Minute


At what engine RPM (revolutions per minute) does the engine produce maximum power?
(A) I
(B) At some point between I and II
(C) II
(D) At some point between II and III
(E) III
The correct answer is D
23. A particle is launched from the surface of a uniform, stationary spherical planet at an angle to the vertical. The
particle travels in the absence of air resistance and eventually falls back onto the planet. Spaceman Fred describes
the path of the particle as a parabola using the laws of projectile motion. Spacewoman Kate recalls from Keplers
laws that every bound orbit around a point mass is an ellipse (or circle), and that the gravitation due to a uniform
sphere is identical to that of a point mass. Which of the following best explains the discrepancy?
(A) Because the experiment takes place very close to the surface of the sphere, it is no longer valid to replace
the sphere with a point mass.
(B) Because the particle strikes the ground, it is not in orbit of the planet and therefore can follow a nonelliptical path.
(C) Kate disregarded the fact that motions around a point mass may also be parabolas or hyperbolas.
(D) Keplers laws only hold in the limit of large orbits.
(E) The path is an ellipse, but is very close to a parabola due to the short length of the flight relative to the
distance from the center of the planet.
The correct answer is E

c
Copyright 2011
American Association of Physics Teachers

2011 F = ma Exam

10

24. A turntable is supported on a Teflon ring of inner radius R and outer radius R+ (  R), as shown in the diagram.
To rotate the turntable at a constant rate, power must be supplied to overcome friction. The manufacturer of the
turntable wishes to reduce the power required without changing the rotation rate, the weight of the turntable, or
the coefficient of friction of the Teflon surface. Engineers propose two solutions: increasing the width of the bearing
(increasing ), or increasing the radius (increasing R). What are the effects of these proposed changes?

(A) Increasing has no significant effect on the required power; increasing R increases the required power.
(B) Increasing has no significant effect on the required power; increasing R decreases the required power.
(C) Increasing increases the required power; increasing R has no significant effect on the required power.
(D) Increasing decreases the required power; increasing R has no significant effect on the required power.
(E) Neither change has a significant effect on the required power.
The correct answer is A
25. A hollow cylinder with a very thin wall (like a toilet paper tube) and a block are placed at rest at the top of a
plane with inclination above the horizontal. The cylinder rolls down the plane without slipping and the block
slides down the plane; it is found that both objects reach the bottom of the plane simultaneously. What is the
coefficient of kinetic friction between the block and the plane?
(A) 0
(B)
(C)
(D)

1
3
1
2
2
3

tan
tan
tan

(E) tan
The correct answer is C

c
Copyright 2011
American Association of Physics Teachers

United States Physics Team


Semi Final Contest Papers
2011

2011 Semifinal Exam

AAPT
AIP

UNITED STATES PHYSICS TEAM


2011

Semifinal Exam

DO NOT DISTRIBUTE THIS PAGE


Important Instructions for the Exam Supervisor
This examination consists of two parts.
Part A has four questions and is allowed 90 minutes.
Part B has two questions and is allowed 90 minutes.
The first page that follows is a cover sheet. Examinees may keep the cover sheet for both
parts of the exam.
The parts are then identified by the center header on each page. Examinees are only allowed
to do one part at a time, and may not work on other parts, even if they have time remaining.
Allow 90 minutes to complete Part A. Do not let students look at Part B. Collect the answers
to Part A before allowing the examinee to begin Part B. Examinees are allowed a 10 to 15
minutes break between parts A and B.
Allow 90 minutes to complete Part B. Do not let students go back to Part A.
Ideally the test supervisor will divide the question paper into 3 parts: the cover sheet (page
2), Part A (pages 3-4), and Part B (pages 6-7). Examinees should be provided parts A and
B individually, although they may keep the cover sheet.
The supervisor must collect all examination questions, including the cover sheet, at the end
of the exam, as well as any scratch paper used by the examinees. Examinees may not take
the exam questions. The examination questions may be returned to the students after March
31, 2011.
Examinees are allowed calculators, but they may not use symbolic math, programming, or
graphic features of these calculators. Calculators may not be shared and their memory must
be cleared of data and programs. Cell phones, PDAs or cameras may not be used during
the exam or while the exam papers are present. Examinees may not use any tables, books,
or collections of formulas.
Please provide the examinees with graph paper for Part A.

c
Copyright 2011
American Association of Physics Teachers

2011 Semifinal Exam

AAPT
AIP

Cover Sheet

UNITED STATES PHYSICS TEAM


2011

Semifinal Exam
INSTRUCTIONS
DO NOT OPEN THIS TEST UNTIL YOU ARE TOLD TO BEGIN
Work Part A first. You have 90 minutes to complete all four problems. Each question is
worth 25 points. Do not look at Part B during this time.
After you have completed Part A you may take a break.
Then work Part B. You have 90 minutes to complete both problems. Each question is worth
50 points. Do not look at Part A during this time.
Show all your work. Partial credit will be given. Do not write on the back of any page. Do
not write anything that you wish graded on the question sheets.
Start each question on a new sheet of paper. Put your AAPT ID number, your name, the
question number and the page number/total pages for this problem, in the upper right hand
corner of each page. For example,
AAPT ID #
Doe, Jamie
A1 - 1/3
A hand-held calculator may be used. Its memory must be cleared of data and programs. You
may use only the basic functions found on a simple scientific calculator. Calculators may not
be shared. Cell phones, PDAs or cameras may not be used during the exam or while the
exam papers are present. You may not use any tables, books, or collections of formulas.
Questions with the same point value are not necessarily of the same difficulty.
In order to maintain exam security, do not communicate any information about
the questions (or their answers/solutions) on this contest until after April 1, 2011.
Possibly Useful Information. You may
g = 9.8 N/kg
k = 1/40 = 8.99 109 N m2 /C2
c = 3.00 108 m/s
NA = 6.02 1023 (mol)1
= 5.67 108 J/(s m2 K4 )
1eV = 1.602 1019 J
me = 9.109 1031 kg = 0.511 MeV/c2
sin 16 3 for ||  1

use this sheet for both parts of the exam.


G = 6.67 1011 N m2 /kg2
km = 0 /4 = 107 T m/A
kB = 1.38 1023 J/K
R = NA kB = 8.31 J/(mol K)
e = 1.602 1019 C
h = 6.63 1034 J s = 4.14 1015 eV s
(1 + x)n 1 + nx for |x|  1
cos 1 12 2 for ||  1

c
Copyright 2011
American Association of Physics Teachers

2011 Semifinal Exam

Part A

Part A
Question A1
Single bubble sonoluminescence occurs when sound waves cause a bubble suspended in a fluid
to collapse so that the gas trapped inside increases in temperature enough to emit light. The
bubble actually undergoes a series of expansions and collapses caused by the sound wave pressure
variations.
We now consider a simplified model of a bubble undergoing sonoluminescence. Assume the bubble is originally at atmospheric pressure P0 = 101 kPa. When the pressure in the fluid surrounding
the bubble is decreased, the bubble expands isothermally to a radius of 36.0 m. When the pressure
increases again, the bubble collapses to a radius of 4.50 m so quickly that no heat can escape.
Between the collapse and subsequent expansion, the bubble undergoes isochoric (constant volume)
cooling back to its original pressure and temperature. For a bubble containing a monatomic gas,
suspended in water of T = 293 K, find
a. the number of moles of gas in the bubble,
b. the pressure after the expansion,
c. the pressure after collapse,
d. the temperature after the collapse, and
e. the total work done on the bubble during the whole process.
You may find the following useful: the specific heat capacity at constant volume is CV = 3R/2
and the ratio of specific heat at constant pressure to constant volume is = 5/3 for a monatomic
gas.

Solution
We consider the bubble to be filled with an ideal monatomic gas, so originally: P0 V0 = nRT0 .
The bubble undergoes 3 processes: 1) isothermal expansion, 2) adiabatic collapse (no heat
escapes), and 3) isochoric (constant volume) cooling. The final process is isochoric, so we know
that the bubbles collapsed volume is equal to its original volume, so
V2 = V0 ,
and
P0 V0 = P0 V2 = nRT0 .
Rearranging,
n=
n=

P0 V 2
RT0

P0 43 r23
RT0

101, 000 N2 34 (4.50 106 m)3


3.86 1011
m
n=
=
moles
2430
8.31 J 293 K
molK
c
Copyright 2011
American Association of Physics Teachers

2011 Semifinal Exam

a)

Part A

n = 1.58 1014 moles.

Process 1: Isothermal expansion


This process is isothermal, so T1 = T0 and
P1 V1 = nRT1 = nRT0
P1 =
b)

1.58 1014 moles 8.31 J 293 K


nRT0
molK
=
4
5 m)3
V1

(3.60

10
3

N = 197 Pa.
P1 = 197 m
2
The work done by the bubble is:
W1 = nRT0 ln
W1 = 1.58 1014 moles 8.31

V1
V0

J
293 K ln
molK

(3.60 105 )3
(4.50 106 )3

W1 = 2.40 1010 J
So, the work done on the bubble during the expansion is:
W1 = 2.40 1010 J.

Process 2: Adiabatic collapse


For an adiabatic process
P1 V1 = P2 V2
P2 =

P1 V1
V2

For a monatomic gas = 5/3 so,


N (3.60 105 m)5
197 m
2
P2 =
(4.50 106 m)5
c)

P2 = 6.46 106 Pa.


And
T2 =

P2 V 2
nR

6.46 106 Pa 34 (4.50 106 m)3


1.58 1014 moles 8.31 J
molK
T2 = 18800 K. Lord, have mercy! Thats hot!
T2 =

d)

The work done by the bubble during an adiabatic process is


W2 = Einternal = nCv T
c
Copyright 2011
American Association of Physics Teachers

2011 Semifinal Exam

Part A

where Cv = 3R/2.
14

W2 = 1.58 10

 
3
J
moles
8.31
(18800 293) K
2
molK
W2 = 3.64 109 J

The work done on the bubble is then


W2 = 3.64 109 J

Process 3: Isochoric cooling


The work done on the bubble during an isochoric process is zero, so W3 = 0 J.
The total work is then the sum of the work on the bubble
Wtotal = W1 + W2 + W3
Wtotal = 2.40 1010 J + 3.64 109 J + 0 J
e)

Wtotal = 3.4 10 9 J.

Question A2
A thin, uniform rod of length L and mass M = 0.258 kg is suspended from a point a distance R
away from its center of mass. When the end of the rod is displaced slightly and released it executes
simple harmonic oscillation. The period, T , of the oscillation is timed using an electronic timer.
The following data is recorded for the period as a function of R. What is the local value of g? Do
not assume it is the canonical value of 9.8 m/s2 . What is the length, L, of the rod? No estimation
of error in either value is required. The moment of inertia of a rod about its center of mass is
(1/12)M L2 .
R
(m)
0.050
0.075
0.102
0.156
0.198

T
(s)
3.842
3.164
2.747
2.301
2.115

R
(m)
0.211
0.302
0.387
0.451
0.588

T
(s)
2.074
1.905
1.855
1.853
1.900

You must show your work to obtain full credit. If you use graphical techniques then you must
plot the graph; if you use linear regression techniques then you must show all of the formulae and
associated workings used to obtain your result.

Solution
The period of a physical pendulum is given by
s
s
1 2
L + R2
I
T = 2
= 2 12
mgR
gR
c
Copyright 2011
American Association of Physics Teachers

2011 Semifinal Exam

Part A

A little math, and


g

T 2R
1
L2 = R2 .
2
4
12

This is of the form


mx + b = y
if we let
y = R2
and
x=

T 2R
4 2

R
T
T 2 R/4 2
R2
0.050 3.842
0.0187
0.0025
0.075 3.164
0.0190
0.0056
0.102 2.747
0.0195
0.0104
0.156 2.301
0.0209
0.0243
Filling out a table of data, we get 0.198 2.115
0.0224
0.0392 The corresponding graph of
0.211 2.074
0.0230
0.0445
0.302 1.905
0.0278
0.0912
0.387 1.855
0.0337
0.1498
0.451 1.853
0.0392
0.2034
0.588 1.900
0.0538
0.3457
1 2
2
2
2
L .
T /R/4 versus R ought yield a straight line such that the slope is g and the intercept is 12

g = 9.7923 m/s2
and
L = 1.470 m

c
Copyright 2011
American Association of Physics Teachers

2011 Semifinal Exam

Part A

Question A3
A light bulb has a solid cylindrical filament of length L and radius a, and consumes power P .
You are to design a new light bulb, using a cylindrical filament of the same material, operating at
the same voltage, and emitting the same spectrum of light, which will consume power nP . What
are the length and radius of the new filament? Assume that the temperature of the filament is
approximately uniform across its cross-section; the filament doesnt emit light from the ends; and
energy loss due to convection is minimal.

Solution
Since the new bulb emits the same spectrum of light, the emitted power is simply proportional
to the area:
P 2aL
P aL
If the resistivity of the filament is , the resistance is
R=

L
L
=
A
a2

and therefore the power is also given by


P =

V2
V 2 a2
=
R
L
P

a2
L

Combining our conditions,


a P 2/3
L P 1/3
So the new filament must have length n2/3 a and length n1/3 L.

Question A4
In this problem we consider a simplified model of the electromagnetic radiation inside a cubical
box of side length L. In this model, the electric field has spatial dependence
E(x, y, z) = E0 sin(kx x) sin(ky y) sin(kz z)
where one corner of the box lies at the origin and the box is aligned with the x, y, and z axes. Let
h be Plancks constant, kB be Boltzmanns constant, and c be the speed of light.
a. The electric field must be zero everywhere at the sides of the box. What condition does this
impose on kx , ky , and kz ? (Assume that any of these may be negative, and include cases
where one or more of the ki is zero, even though this causes E to be zero.)
b. In the model, each permitted value of the triple (kx , ky , kz ) corresponds to a quantum state.
These states can be visualized in a state space, which is a notional three-dimensional space
c
Copyright 2011
American Association of Physics Teachers

2011 Semifinal Exam

Part A

with axes corresponding to kx , ky , and kz . How many states occupy a volume s of state space,
if s is large enough that the discreteness of the states can be ignored?
c. Each quantum state, in turn, may be occupied by photons with frequency =
where
q
|k| = kx 2 + ky 2 + kz 2

f
2

= c|k|,

In the model, if the temperature inside the box is T , no photon may have energy greater than
kB T . What is the shape of the region in state space corresponding to occupied states?
d. As a final approximation, assume that each occupied state contains exactly one photon. What
is the total energy of the photons in the box, in terms of h, kB , c, T , and the volume of the box
V ? Again, assume that the temperature is high enough that there are a very large number of
occupied states. (Hint: divide state space into thin regions corresponding to photons of the
same energy.)
Note that while many details of this model are extremely inaccurate, the final result is correct
except for a numerical factor.

Solution
a. We require that sin(kx L) = 0, so that
kx L = nx
for any integer nx , and similarly for ky and kz .

b. The occupied states are equally spaced a distance L


apart. Each can therefore be thought of
3
as taking up volume L3 , and the number of states in the volume s is

L3
s
3
c. A photons energy is E = h
= h
c|k|, where h
=

h
2 .

Thus the occupied states obey

hc|k| kB T
|k|
This corresponds to a ball of radius

kB T
hc

kB T
hc

in state space.

d. As we have seen, the energy of a photon is proportional to its distance |k| from the origin
in state space. Thus consider the spherical shell in state space between radius k and radius
k + dk. The volume of this region is
ds = 4k 2 dk
Each photon in the region has energy h
ck, and from above there are
region. Therefore the photons in the region have total energy
dE = h
ck

L3
ds
3

L3
4k 2 dk
3

c
Copyright 2011
American Association of Physics Teachers

photons in the

2011 Semifinal Exam

Part A

dE =
From above, k ranges from zero to kmax =
Z

4
hcL3 k 3 dk
2
kB T
hc ,

kmax

E=
0

so the total energy is

4
hcL3 k 3 dk
2

1
4
E = 2 hcL3

kB T
hc

4

Since the volume of the box is V = L3 , and h = 2h, this cleans up to


E=

8kB 4 4
T V
h3 c3

c
Copyright 2011
American Association of Physics Teachers

2011 Semifinal Exam

Part A

10

STOP: Do Not Continue to Part B

If there is still time remaining for Part A, you should review your work for
Part A, but do not continue to Part B until instructed by your exam
supervisor.

c
Copyright 2011
American Association of Physics Teachers

2011 Semifinal Exam

Part B

11

Part B
Question B1
An AC power line cable transmits electrical power using a sinusoidal waveform with frequency
60 Hz. The load receives an RMS voltage of 500 kV and requires 1000 MW of average power.
For this problem, consider only the cable carrying current in one of the two directions, and ignore
effects due to capacitance or inductance between the cable and with the ground.
a. Suppose that the load on the power line cable is a residential area that behaves like a pure
resistor.
i. What is the RMS current carried in the cable?
ii. The cable has diameter 3 cm, is 500 km long, and is made of aluminum with resistivity
2.8 108 m. How much power is lost in the wire?
b. A local rancher thinks he might be able to extract electrical power from the cable using
electromagnetic induction. The rancher constructs a rectangular loop of length a and width
b < a, consisting of N turns of wire. One edge of the loop is to be placed on the ground; the
wire is straight and runs parallel to the ground at a height h much less than the length of the
wire. Write the current in the wire as I = I0 sin t, and assume the return wire is far away.
i. Determine an expression for the magnitude of the magnetic field at a distance r from
the power line cable in terms of I, r, and fundamental constants.
ii. Where should the loop be placed, and how should it be oriented, to maximize the induced
emf in the loop?
iii. Assuming the loop is placed in this way, determine an expression for the emf induced
in the loop (as a function of time) in terms of any or all of I0 , h, a, b, N , , t, and
fundamental constants.
iv. Suppose that a = 5 m, b = 2 m, and h = 100 m. How many turns of wire N does the
rancher need to generate an RMS emf of 120 V?
c. The load at the end of the power line cable changes to include a manufacturing plant with
a large number of electric motors. While the average power consumed remains the same, it
now behaves like a resistor in parallel with a 0.25 H inductor.
i. Does the power lost in the power line cable increase, decrease, or stay the same? (You
need not calculate the new value explicitly, but you should show some work to defend
your answer.)
ii. The power company wishes to make the load behave as it originally did by installing a
capacitor in parallel with the load. What should be its capacitance?

Solution
a.

i. Because the load is purely resistive, the average power is simply


Pav = Vrms Irms
c
Copyright 2011
American Association of Physics Teachers

2011 Semifinal Exam

Part B

so

12

Pav
= 2000 A
Vrms

Irms =

ii. The cross-sectional area of the wire is A = r2 = 7.07 104 m2 , so its resistance is
L
= 19.8
A

R=
The power loss is then

P = I 2 R = 79.2 MW
b.

i. The field is perpendicular to the wire and to the radius, and from Amperes Law
I
B ds = 0 Iencl
B 2r = 0 I
B=

0 I
2r

ii. The induced emf is proportional to the rate of change of the flux through the loop. Since
the time dependence of the magnetic field is uniform across space, the rate of change of
flux is maximized by maximizing the flux itself. This in turn can be accomplished by
maximizing the field in the loop and ensuring that it is normal to the loop. Because
the field gets stronger closer to the wire, the loop should be directly below the wire,
and since the field is horizontal and perpendicular to the wire at this location, the loop
should be vertical and parallel to the wire. Finally, again because the field gets stronger
closer to the wire, the long edge of the loop should be vertical.
In summary, the loop should be placed vertically, parallel to the wire and directly beneath
it, with the long edge vertical.
iii. From Faradays law,
d
B
dt
where we have dropped the sign and B is the magnetic flux through a single loop. The
flux, in turn, is defined as
Z
E =N

B dA

B =

Dividing the loop into strips of radial width dr and length b,


Z h
B =
B(r)b dr
ha

B =
ha

B =

0 I
b dr
2r

h
0 Ib
ln
2
ha

So,
E =N

d 0 Ib
h
ln
dt 2
ha

c
Copyright 2011
American Association of Physics Teachers

2011 Semifinal Exam

Part B

13

Since the loop is stationary, only I depends on t, and


E =N
E =N

h dI
0 b
ln
2
h a dt

h
0 b
ln
I0 cos t
2
ha

iv. Note that the RMS value of I0 cos t is the same as the RMS value of I0 sin t, i.e. Irms .
So, taking the RMS value of both sides of our previous result,
Erms = N
And (conveniently) the frequency f =

h
0 b
ln
Irms
2
ha

2 ,

so

Erms = N 0 bf ln

h
Irms
ha

With the given numbers,


0 bf ln

h
Irms = 0.0155 V
ha

so that the required number of turns is


N = 7757
c.

i. The inductor adds a new component of the current in the wire out of phase with the
voltage; this component does not transmit power, so the in-phase component must remain unchanged. The total current is thus increased, and with it the power lost in the
wire increases as well.
ii. The resonant frequency of an LC circuit is given by
=
so that
C=

1
LC

1
2L

Here = 2f = 377 s1 , so
C = 28.1 F

c
Copyright 2011
American Association of Physics Teachers

2011 Semifinal Exam

Part B

14

Question B2
A particle is constrained to move on the inner surface of a frictionless parabolic bowl whose crosssection has equation z = kr2 . The particle begins at a height z0 above the bottom of the bowl with
a horizontal velocity v0 along the surface of the bowl. The acceleration due to gravity is g.

z
z = kr2

a. For a particular value of horizontal velocity v0 , which we will name vh , the particle moves in
a horizontal circle. What is vh in terms of g, z0 , and/or k?
b. Suppose that the initial horizontal velocity is now v0 > vh . What is the maximum height
reached by the particle, in terms of v0 , z0 , g and/or k?
c. Suppose that the particle now begins at a height z0 above the bottom of the bowl with an
initial velocity v0 = 0.
i. Assuming that z0 is small enough so that the motion can be approximated as simple
harmonic, find the period of the motion in terms any or all of the mass of the particle
m, g, z0 , and/or k.
ii. Assuming that z0 is not small, will the actual period of motion be greater than, less
than, or equal to your simple harmonic approximation above? (You need not calculate
the new value explicitly, but you should show some work to defend your answer.)

Solution
a. Let the particle have mass m, let the radius of the bowl at height z0 be r0 , and let the angle
made by the bowls surface to the horizontal at that height be .
Two forces act on the particle: the normal force and gravity. If the particle moves in a
horizontal circle the horizontal component of the net force must equal the centripetal force
2
Fc = mvr0h , whereas the vertical component must be zero. From the free body diagram
[Diagram], these conditions are
mvh 2
FN sin =
r0
FN cos mg = 0
Combining these,
tan =

vh 2
gr0

c
American Association of Physics Teachers
Copyright 2011

2011 Semifinal Exam

Part B

15

dz
dr

However, tan is simply the slope of the bowl

= 2kr0 , so that

2ar0 =

vh 2
gr0

2gz0

Using the fact that z0 = kr0 2 ,


vh =

b. Let the maximum height be z, let the radius of the bowl at this point be r, and let the speed
of the particle at this point be v. From conservation of energy,
1
1
mv0 2 + mgz0 = mv 2 + mgz
2
2
Meanwhile, the two forces acting on the particle never exert a torque in the direction of the
bowls axis, and so angular momentum about this axis is conserved. Furthermore, at the
point of maximum height the velocity of the particle is entirely tangential to the axis, so the
conservation condition is simply
mv0 r0 = mvr
r0
v = v0
r
or, since z = kr2 and z0 = kr0 2 ,
r
v = v0

z0
z

Combining our results,


1
z0
1
mv0 2 + mgz0 = mv0 2 + mgz
2
2
z

 2
2
v0
v0
+ z0 z +
z0 = 0
z2
2g
2g


v0 2
(z z0 ) z
=0
2g
The root z = z0 corresponds to our starting condition, so the desired root is
z=

v0 2
2g

Note that we recover z = z0 if v0 = vh as we would expect; indeed, the analysis in this section
is an alternative path to the previous result.
c.

i. We present a force-based approach and an energy-based approach. In each case, let r


be the radial position of the particle, so that z = kr2 is the height of the particle above
the bottom of the bowl.
The force-based approach begins with the free body diagram. [Diagram] Again, let the
angle of the bowls surface to the horizontal be . Because z0 is small,
sin tan =

dz
= 2kr
dr

and cos 1.
c
Copyright 2011
American Association of Physics Teachers

2011 Semifinal Exam

Part B

16

We can consider the component of force tangential to the bowl, which is mg sin ; Newtons third law then gives for the magnitude of the acceleration a
ma = mg sin
noting that the acceleration is entirely tangential because the particle is constrained
to the surface of the bowl (and there is no centripetal force anymore). The radial
acceleration ar is given by
ar = a cos
where we have introduced the appropriate sign to match the sign of r. Thus
ar = g cos sin
So in the small-z approximation,
ar g tan
ar =

d2 r
= 2krg
dt2

The energy-based approach begins with the total energy


1
E = mv 2 + mgz
2
The velocity v is given by


v =
Because z is small,

dz
dt

dr
dt ,

dr
dt

2


+

dz
dt

2

and we conclude that


1
E= m
2

From conservation of energy,

dE
dt

dr
dt

2

+ mgkr2

= 0:

0=m

dr d2 r
dr
+ 2mgkr
2
dt dt
dt

0=

d2 r
+ 2krg
dt2

Both approaches lead to the standard SHM differential equation


d2 x
+ 2x = 0
dt2
with angular frequency =

2kg; since the period T =

2
,

2
T =
2kg
c
Copyright 2011
American Association of Physics Teachers

2011 Semifinal Exam

Part B

17

ii. The period is greater than the simple harmonic period. We can see this using both
approaches:
In the force-based approach, we obtained the exact equation
ar = g cos sin
and approximated it as
ar = g tan
Since cos sin < tan , the exact radial acceleration is smaller than the approximate
one, so that the particle takes longer to reach the origin in reality than it does in the
approximation, meaning that the period is larger.
In the energy-based approach, we dropped a (positive) term in the formula for the speed
dr
v as expressed in terms of dr
dt . Therefore we overestimated dt , and again the particle
takes longer to reach the origin in reality than it does in the approximation.

c
Copyright 2011
American Association of Physics Teachers

United States Physics Team


F = ma Contest Papers
2012

2012 F = ma Exam

AAPT
AIP

UNITED STATES PHYSICS TEAM


2012
2012 F = ma Contest
25 QUESTIONS - 75 MINUTES
INSTRUCTIONS

DO NOT OPEN THIS TEST UNTIL YOU ARE TOLD TO BEGIN


Use g = 10 N/kg throughout this contest.
You may write in this booklet of questions. However, you will not receive any credit for anything written in
this booklet.
Your answer to each question must be marked on the optical mark answer sheet.
Select the single answer that provides the best response to each question. Please be sure to use a No. 2 pencil
and completely fill the box corresponding to your choice. If you change an answer, the previous mark must
be completely erased.
Correct answers will be awarded one point; incorrect answers will result in a deduction of
no penalty for leaving an answer blank.

1
4

point. There is

A hand-held calculator may be used. Its memory must be cleared of data and programs. You may use only
the basic functions found on a simple scientific calculator. Calculators may not be shared. Cell phones may
not be used during the exam or while the exam papers are present. You may not use any tables, books, or
collections of formulas.
This test contains 25 multiple choice questions. Your answer to each question must be marked on the optical
mark answer sheet that accompanies the test. Only the boxes preceded by numbers 1 through 25 are to be
used on the answer sheet.
All questions are equally weighted, but are not necessarily the same level of difficulty.
In order to maintain exam security, do not communicate any information about the questions
(or their answers or solutions) on this contest until after February 20, 2012.
The question booklet and answer sheet will be collected at the end of this exam. You may not use scratch
paper.

DO NOT OPEN THIS TEST UNTIL YOU ARE TOLD TO BEGIN


Contributors to this years exam include Jiajia Dong, Qiuzi Li, Paul Stanley, Warren Turner, former US Team members Marianna Mao, Andrew Lin, Steve Byrnes,
Adam Jermyn, Ante Qu, Alok Saxena, Tucker Chan, Kenan Diab, Jason LaRue.

c
Copyright 2012
American Association of Physics Teachers

2012 F = ma Exam

1. Consider a dripping faucet, where the faucet is 10 cm above the sink. The time between drops is such that when
one drop hits the sink, one is in the air and another is about to drop. At what height above the sink will the drop
in the air be right as a drop hits the sink?
(A) Between 0 and 2 cm,
(B) Between 2 and 4 cm,
(C) Between 4 and 6 cm,
(D) Between 6 and 8 cm, CORRECT
(E) Between 8 and 10 cm,
2. A cannonball is launched with initial velocity of magnitude v0 over a horizontal surface. At what minimum angle
min above the horizontal should the cannonball be launched so that it rises to a height H which is larger than the
horizontal distance R that it will travel when it returns to the ground?
(A) min = 76 CORRECT
(B) min = 72
(C) min = 60
(D) min = 45
(E) There is no such angle, as R > H for all range problems.
3. An equilateral triangle is sitting on an inclined plane. Friction is too high for it to slide under any circumstance,
but if the plane is sloped enough it can topple down the hill. What angle incline is necessary for it to start
toppling?
(A) 30 degrees
(B) 45 degrees
(C) 60 degrees CORRECT
(D) It will topple at any angle more than zero
(E) It can never topple if it cannot slide
4. A particle at rest explodes into three particles of equal mass in the absence of external forces. Two particles emerge
at a right angle to each other with equal speed v. What is the speed of the third particle?
(A) v

(B) 2v CORRECT
(C) 2v

(D) 2 2v
(E) The third particle can have a range of different speeds.
5. A 12 kg block moving east at 4 m/s collides head on with a 6 kg block that is moving west at 2 m/s. The two
blocks move together after the collision. What is the loss in kinetic energy in this collision?
(A) 36 J
(B) 48 J
(C) 60 J
(D) 72 J CORRECT
(E) 96 J

c
Copyright 2012
American Association of Physics Teachers

2012 F = ma Exam

The following information applies to questions 6 and 7


Two cannons are arranged vertically, with the lower cannon pointing upward (towards the upper cannon) and the
upper cannon pointing downward (towards the lower cannon), 200m above the lower cannon. Simultaneously, they
both fire. The muzzle velocity of the lower cannon is 25m/s and the muzzle velocity of the upper cannon is 55m/s.

200 meters

6. How long after the cannons fire do the projectiles collide?


(A) 2.2 s
(B) 2.5 s CORRECT
(C) 3.6 s
(D) 6.7 s
(E) 8.0 s
7. How far beneath the top cannon do the projectiles collide?
(A) 31 m
(B) 67 m
(C) 110 m
(D) 140 m
(E) 170 m CORRECT
8. A block of mass m = 3.0 kg is moving on a horizontal surface towards a massless spring with spring constant
k = 80.0 N/m. The coefficient of kinetic friction between the block and the surface is k = 0.50. The block has a
speed of 2.0 m/s when it first comes in contact with the spring. How far will the spring be compressed?
(A) 0.19 m
(B) 0.24 m CORRECT
(C) 0.39 m
(D) 0.40 m
(E) 0.61 m

c
Copyright 2012
American Association of Physics Teachers

2012 F = ma Exam

9. A uniform spherical planet has radius R and the acceleration due to gravity at its surface is g. What is the escape
velocity of a particle from the planets surface?

(A) 12 gR

(B) gR

(C) 2gR CORRECT

(D) 2 gR
(E) The escape velocity cannot be expressed in terms of g and R alone.
10. Four objects are placed at rest at the top of an inclined plane and allowed to roll without slipping to the bottom
in the absence of rolling resistance and air resistance.
Object A is a solid brass ball of diameter d.
Object B is a solid brass ball of diameter 2d.
Object C is a hollow brass sphere of diameter d.
Object D is a solid aluminum ball of diameter d. (Aluminum is less dense than brass.)
The balls are placed so that their centers of mass all travel the same distance. In each case, the time of motion T
is measured. Which of the following statements is correct?
(A) TB > TC > TA = TD
(B) TA = TB = TC > TD
(C) TB > TA = TC = TD
(D) TC > TA = TB = TD CORRECT
(E) TA = TB = TC = TD
11. As shown below, Lily is using the rope through a fixed pulley to move a box with constant speed v. The kinetic
friction coefficient between the box and the ground is < 1; assume that the fixed pulley is massless and there is
no friction between the rope and the fixed pulley. Then, while the box is moving, which of the following statements
is correct?

(A) The magnitude of the force on the rope is constant.


(B) The magnitude of friction between the ground and the box is decreasing. CORRECT
(C) The magnitude of the normal force of the ground on the box is increasing.
(D) The pressure of the box on the ground is increasing.
(E) The pressure of the box on the ground is constant.

c
Copyright 2012
American Association of Physics Teachers

2012 F = ma Exam

12. A rigid hoop can rotate about the center. Two massless strings are attached to the hoop, one at A, the other at B.
These strings are tied together at the center of the hoop at O, and a weight G is suspended from that point. The
strings have a fixed length, regardless of the tension, and the weight G is only supported by the strings. Originally
OA is horizontal.

B
T2

T1
A

O
G

Now, the outer hoop will start to slowly rotate 90 clockwise until OA will become vertical, while keeping the angle
between the strings constant and keeping the object static. Which of the following statements about the tensions
T1 and T2 in the two strings is correct?
(A) T1 always decreases.
(B) T1 always increases.
(C) T2 always increases.
(D) T2 will become zero at the end of the rotation. CORRECT
(E) T2 first increases and then decreases.
13. Shown below is a graph of the x component of force versus position for a 4.0 kg cart constrained to move in one
dimension on the x axis. At x = 0 the cart has a velocity of 3.0 m/s (in the negative direction). Which of the
following is closest to the maximum speed of the cart?

F (Newtons)

4
2

4
x (meters)

(A) 1.6 m/s


(B) 2.5 m/s
(C) 3.0 m/s
(D) 4.0 m/s
(E) 4.2 m/s CORRECT

c
Copyright 2012
American Association of Physics Teachers

2012 F = ma Exam

14. A uniform cylinder of radius a originally has a weight of 80 N. After an off-axis cylinder hole at 2a/5 was drilled
through it, it weighs 65 N. The axes of the two cylinders are parallel and their centers are at the same height.
a

2a/5

A force T is applied to the top of the cylinder horizontally. In order to keep the cylinder is at rest, the magnitude
of the force is closest to:
(A) 6 N CORRECT
(B) 10 N
(C) 15 N
(D) 30 N
(E) 38 N
15. A car of mass m has an engine that provides a constant power output P . Assuming no friction, what is the
maximum constant speed vmax that this car can drive up a long incline that makes an angle with the horizontal?
(A) vmax = P/(mg sin ) CORRECT
(B) vmax = P 2 sin /mg
p
(C) vmax = 2P/mg/ sin
(D) There is no maximum constant speed.
(E) The maximum constant speed depends on the length of the incline.
16. Inside a cart that is accelerating horizontally at acceleration ~a, there is a block of mass M connected to two light
springs of force constants k1 and k2 . The block can move without friction horizontally. Find the vibration frequency
of the block.

1
2

k1 +k2
M

(B)

1
2

k1 k2
(k1 +k2 )M

(C)

1
2

k1 k2
(k1 +k2 )M

|k1 k2 |
M

k1 +k2
M

(A)

(D)

1
2

(E)

1
2

+a

+a

CORRECT

c
Copyright 2012
American Association of Physics Teachers

2012 F = ma Exam

17. Shown below is a log/log plot for the data collected of amplitude and period of oscillation for certain non-linear
oscillator.

log (T)

2
0

log (A)

According to the data, the relationship between period T and amplitude A is best given by
(A) T = 1000A2 CORRECT
(B) T = 100A3
(C) T = 2A + 3

(D) T = 3 A
(E) Period is independent of amplitude for oscillating systems
18. A mass hangs from the ceiling of a box by an ideal spring. With the box held fixed, the mass is given an initial
velocity and oscillates with purely vertical motion. When the mass reaches the lowest point of its motion, the box
is released and allowed to fall. To an observer inside the box, which of the following quantities does not change
when the box is released? Ignore air resistance.
(A) The amplitude of the oscillation
(B) The period of the oscillation CORRECT
(C) The maximum speed reached by the mass
(D) The height at which the mass reaches its maximum speed
(E) The maximum height reached by the mass
19. A 1,500 Watt motor is used to pump water a vertical height of 2.0 meters out of a flooded basement through a
cylindrical pipe. The water is ejected though the end of the pipe at a speed of 2.5 m/s. Ignoring friction and
assuming that all of the energy of the motor goes to the water, which of the following is the closest to the radius
of the pipe? The density of water is = 1000 kg/m3 .
(A) 1/3 cm
(B) 1 cm
(C) 3 cm
(D) 10 cm CORRECT
(E) 30 cm

c
Copyright 2012
American Association of Physics Teachers

2012 F = ma Exam

45.0 kg

??? kg

12.0 kg

20. A container of water is sitting on a scale. Originally, the scale reads M1 = 45 kg. A block of wood is suspended
from a second scale; originally the scale read M2 = 12 kg. The density of wood is 0.60 g/cm3 ; the density of the
water is 1.00 g/cm3 . The block of wood is lowered into the water until half of the block is beneath the surface.
What is the resulting reading on the scales?

??? kg

(A) M1 = 45 kg and M2 = 2 kg.


(B) M1 = 45 kg and M2 = 6 kg.
(C) M1 = 45 kg and M2 = 10 kg.
(D) M1 = 55 kg and M2 = 6 kg.
(E) M1 = 55 kg and M2 = 2 kg. CORRECT
21. A spring system is set up as follows: a platform with a weight of 10 N is on top of two springs, each with spring
constant 75N/m. On top of the platform is a third spring with spring constant 75N/m. If a ball with a weight of
5.0 N is then fastened to the top of the third spring and then slowly lowered, by how much does the height of the
spring system change?
The ball is placed here

Original height

(A) 0.033 m
(B) 0.067 m
(C) 0.100 m CORRECT
(D) 0.133 m
(E) 0.600 m
22. The softest audible sound has an intensity of I0 = 1012 W/m2 . In terms of the fundamental units of kilograms,
meters, and seconds, this is equivalent to
(A) I0 = 1012 kg/s3 CORRECT
(B) I0 = 1012 kg/s
(C) I0 = 1012 kg2 m/s
(D) I0 = 1012 kg2 m/s2
(E) I0 = 1012 kg/m s3

c
Copyright 2012
American Association of Physics Teachers

2012 F = ma Exam

23. Which of the following sets of equipment cannot be used to measure the local value of the acceleration due to
gravity (g)?
(A) A spring scale (which reads in force units) and a known mass.
(B) A rod of known length, an unknown mass, and a stopwatch.
(C) An inclined plane of known inclination, several carts of different known masses, and a stopwatch.
CORRECT
(D) A launcher which launches projectiles at a known speed, a projectile of known mass, and a meter stick.
(E) A motor with a known output power, a known mass, a piece of string of unknown length, and a stopwatch.
24. Three point masses m are attached together by identical springs. When placed at rest on a horizontal surface the
masses form a triangle with side length l. When the assembly is rotated about its center at angular velocity , the
masses form a triangle with side length 2l. What is the spring constant k of the springs?
(A) 2m 2
(B)
(C)
(D)
(E)

2 m 2
3
2
2
3 m
1 m 2
3
1
2
m
3

CORRECT

25. Consider the two orbits around the sun shown below. Orbit P is circular with radius R, orbit Q is elliptical such
that the farthest point b is between 2R and 3R, and the nearest point a is between R/3 and R/2. Consider the
magnitudes of the velocity of the circular orbit vc , the velocity of the comet in the elliptical orbit at the farthest
point vb , and the velocity of the comet in the elliptical orbit at the nearest point va . Which of the following rankings
is correct?

vc

orbit Q
va

b
vb

orbit P

(A) vb > vc > 2va


(B) 2vc > vb > va
(C) 10vb > va > vc CORRECT
(D) vc > va > 4vb

(E) 2va > 2vb > vc

c
Copyright 2012
American Association of Physics Teachers

2012 F = ma Exam

1
1d
2a
3c
4b
5d

10

Solutions
6b
7e
8b
9c
10d

11b
12d
13e
14a
15a

16e
17a
18b
19d
20e

21c
22a
23c
24c
25c

c
Copyright 2012
American Association of Physics Teachers

United States Physics Team


Semi Final Contest Papers
2012

2012 Semifinal Exam

AAPT
AIP

UNITED STATES PHYSICS TEAM


2012

Semifinal Exam

DO NOT DISTRIBUTE THIS PAGE


Important Instructions for the Exam Supervisor
This examination consists of two parts.
Part A has four questions and is allowed 90 minutes.
Part B has two questions and is allowed 90 minutes.
The first page that follows is a cover sheet. Examinees may keep the cover sheet for both
parts of the exam.
The parts are then identified by the center header on each page. Examinees are only allowed
to do one part at a time, and may not work on other parts, even if they have time remaining.
Allow 90 minutes to complete Part A. Do not let students look at Part B. Collect the answers
to Part A before allowing the examinee to begin Part B. Examinees are allowed a 10 to 15
minute break between parts A and B.
Allow 90 minutes to complete Part B. Do not let students go back to Part A.
Ideally the test supervisor will divide the question paper into 3 parts: the cover sheet (page
2), Part A (pages 3-4), and Part B (pages 6-7). Examinees should be provided parts A and
B individually, although they may keep the cover sheet.
The supervisor must collect all examination questions, including the cover sheet, at the end
of the exam, as well as any scratch paper used by the examinees. Examinees may not take
the exam questions. The examination questions may be returned to the students after April
1, 2012.
Examinees are allowed calculators, but they may not use symbolic math, programming, or
graphic features of these calculators. Calculators may not be shared and their memory must
be cleared of data and programs. Cell phones, PDAs or cameras may not be used during
the exam or while the exam papers are present. Examinees may not use any tables, books,
or collections of formulas.
Please provide the examinees with graph paper for Part A.

c
Copyright 2012
American Association of Physics Teachers

2012 Semifinal Exam

AAPT
AIP

Cover Sheet

UNITED STATES PHYSICS TEAM


2012

Semifinal Exam
INSTRUCTIONS
DO NOT OPEN THIS TEST UNTIL YOU ARE TOLD TO BEGIN
Work Part A first. You have 90 minutes to complete all four problems. Each question is
worth 25 points. Do not look at Part B during this time.
After you have completed Part A you may take a break.
Then work Part B. You have 90 minutes to complete both problems. Each question is worth
50 points. Do not look at Part A during this time.
Show all your work. Partial credit will be given. Do not write on the back of any page. Do
not write anything that you wish graded on the question sheets.
Start each question on a new sheet of paper. Put your AAPT ID number, your name, the
question number and the page number/total pages for this problem, in the upper right hand
corner of each page. For example,
AAPT ID #
Doe, Jamie
A1 - 1/3
A hand-held calculator may be used. Its memory must be cleared of data and programs. You
may use only the basic functions found on a simple scientific calculator. Calculators may not
be shared. Cell phones, PDAs or cameras may not be used during the exam or while the
exam papers are present. You may not use any tables, books, or collections of formulas.
Questions with the same point value are not necessarily of the same difficulty.
In order to maintain exam security, do not communicate any information about
the questions (or their answers/solutions) on this contest until after April 1, 2012.
Possibly Useful Information. You may
g = 9.8 N/kg
k = 1/40 = 8.99 109 N m2 /C2
c = 3.00 108 m/s
NA = 6.02 1023 (mol)1
= 5.67 108 J/(s m2 K4 )
1eV = 1.602 1019 J
me = 9.109 1031 kg = 0.511 MeV/c2
sin 16 3 for ||  1

use this sheet for both parts of the exam.


G = 6.67 1011 N m2 /kg2
km = 0 /4 = 107 T m/A
kB = 1.38 1023 J/K
R = NA kB = 8.31 J/(mol K)
e = 1.602 1019 C
h = 6.63 1034 J s = 4.14 1015 eV s
(1 + x)n 1 + nx for |x|  1
cos 1 12 2 for ||  1

c
Copyright 2012
American Association of Physics Teachers

2012 Semifinal Exam

Part A

Part A
Question A1
A newly discovered subatomic particle, the S meson, has a mass M . When at rest, it lives for
exactly = 3 108 seconds before decaying into two identical particles called P mesons (peons?)
that each have a mass of M .
a. In a reference frame where the S meson is at rest, determine
i. the kinetic energy,
ii. the momentum, and
iii. the velocity
of each P meson particle in terms of M , , the speed of light c, and any numerical constants.
b. In a reference frame where the S meson travels 9 meters between creation and decay, determine
i. the velocity and
ii. kinetic energy of the S meson.
Write the answers in terms of M , the speed of light c, and any numerical constants.

Solution
a. Apply conservation of four momentum. For the S meson, we have
pS c = (Es , 0)
and for the two P mesons we have
pP c = (Ep , p),
where p is the magnitude of the (relativistic) three momentum of the P mesons.
This yields
ES = 2EP
We must also satisfy the relation
E 2 = p2 c2 + m2 c4
for each particle, so
ES 2 = M 2 c4
and
EP 2 = p2 c2 + 2 M 2 c4 .
Therefore, the kinetic energy of each P meson is
1
KP = EP M c = M c2 M c2 =
2
2


1
M c2 .
2

c
Copyright 2012
American Association of Physics Teachers

2012 Semifinal Exam

Part A

Square the energy conservation expression, and combine with the momentum/energy/mass
relations:

1 2 4
M c = p2 c2 + 2 M 2 c4 ,
4


1
2 M 2 c4 = p2 c2
4
r
1
2 M c2 = pc.
4

The velocity of each P meson will be found from the relativistic three momentum,
p = mv
and the relativistic energy,
E = mc2
so
pc
E

Putting in the values for the P meson,


q
v = c

1
4

mcv
mc2
= .
=

2 M c2
1
2
2Mc

p
= c 1 42

b. From relativistic kinematics,


d = vt = v,
so

v
d
=
c
c

Call this k for now. Then


k = ,
k2 = 2 2 ,
2
,
k2 =
1 2
k 2 (1 2 ) = 2 ,
k 2 = (1 + k 2 ) 2 ,
k2
1 + k2

= 2.

Combine,
r
v=c

d2
c2 2 + d2

c
Copyright 2012
American Association of Physics Teachers

2012 Semifinal Exam

Part A

so

r
v=c

Then

c
92
=
92 + 92
2

= 1/ 1 1/2 = 2

It isnt much work to find the kinetic energy,

K = ( 1)M c2 = ( 2 1)M c2 .
c. This is a velocity addition problem, so
v=

vS + vS
1 + vS 2 /c2

or, using the numbers from the first part of the problem,

1 42
v=c
1 22

c
Copyright 2012
American Association of Physics Teachers

2012 Semifinal Exam

Part A

Question A2
An ideal (but not necessarily perfect monatomic) gas undergoes the following cycle.
The gas starts at pressure P0 , volume V0 and temperature T0 .
The gas is heated at constant volume to a pressure P0 , where > 1.
The gas is then allowed to expand adiabatically (no heat is transferred to or from the gas) to
pressure P0
The gas is cooled at constant pressure back to the original state.
The adiabatic constant is defined in terms of the specific heat at constant pressure Cp and the
specific heat at constant volume Cv by the ratio = Cp /Cv .
a. Determine the efficiency of this cycle in terms of and the adiabatic constant . As a
reminder, efficiency is defined as the ratio of work out divided by heat in.
b. A lab worker makes measurements of the temperature and pressure of the gas during the
adiabatic process. The results, in terms of T0 and P0 are
Pressure
Temperature

units of P0
units of T0

1.21
2.11

1.41
2.21

1.59
2.28

1.73
2.34

2.14
2.49

Plot an appropriate graph from this data that can be used to determine the adiabatic constant.
c. What is for this gas?

Solution
a. Label the end points as 0, 1, and 2. A quick application of P V = nRT requires that T1 = T0 .
It takes more work to do the process 1 2; it is acceptable to simply state the adiabatic law
of P V = constant; if you dont know this, you will need to derive it.
In the case that you know the adiabatic process law,
P1 V1 = P2 V2 = P1 V2 ,
so that

V2 = V1 () .
1

Another quick application of P V = nRT requires that T2 = () T0 .


Heat enters the gas during isochoric process 0 1, so
Qin = nCv T = nCv ( 1)T0
Heat exits the system during process 2 0, so
Qout = nCp T = nCp (1/ 1)T0
c
Copyright 2012
American Association of Physics Teachers

2012 Semifinal Exam

Part A

We only consider absolute values, and insert negative signs later as needed.
The work done is the difference, so
W = Qin Qout = nCv ( 1)T0 nCp (1/ 1)T0
and the efficiency is then
e=

Cv ( 1) Cp (1/ 1)
Cv ( 1)

This can be greatly simplified to


e=1

1/ 1
1

b. Along an adiabatic path, the relationship between pressure and temperature is given by
 
T

P V = constant P
P
so

P T 1 = constant
As such,

P T 1
Note that, for an ideal gas,
Cp /Cv
Cp

=
=
1
Cp /Cv 1
R
This means that we want to plot a log-log plot with log T horizontal and log P vertical. The
slope of the graph will be Cp /R.
For the data given, Cp = (7/2)R, so = 7/5.

c
Copyright 2012
American Association of Physics Teachers

2012 Semifinal Exam

Part A

Question A3
This problem inspired by the 2008 Guangdong Province Physics Olympiad
Two infinitely long concentric hollow cylinders have radii a and 4a. Both cylinders are insulators;
the inner cylinder has a uniformly distributed charge per length of +; the outer cylinder has a
uniformly distributed charge per length of .
An infinitely long dielectric cylinder with permittivity  = 0 , where is the dielectric constant,
has a inner radius 2a and outer radius 3a is also concentric with the insulating cylinders. The
dielectric cylinder is rotating about its axis with an angular velocity  c/a, where c is the speed
of light. Assume that the permeability of the dielectric cylinder and the space between the cylinders
is that of free space, 0 .

a. Determine the electric field for all regions.


b. Determine the magnetic field for all regions.

Solution
a. Consider a Gaussian cylinder of radius r and length l centered on the cylinder axis. Gausss
Law states that
Z
qencl
E dA =
0
encl l
2rEl =
0
encl
E=
2r0
where encl is the enclosed linear charge density.
The field due to the hollow cylinders alone is therefore

r<a
0

Eapplied = 2r0 a < r < 4a

0
r > 4a
The field within the dielectric is reduced by a

2r0

E = 2r
0

2r0

factor , so that in total


r<a
a < r < 2a
2a < r < 3a
3 < r < 4a
r > 4a

c
Copyright 2012
American Association of Physics Teachers

2012 Semifinal Exam

Part A

b. We can apply the results of the previous section to obtain the enclosed charge density encl
as a function of radius:

0 r < a

a < r < 2a
encl = 2a < r < 3a

3 < r < 4a

0 r > 4a
Defining

i =

1
1

we conclude that a charge density i exists on the inner surface of the dielectric, a charge
density i on the outer surface, and no charge on the interior.
As with the case of a very long solenoid, we expect the magnetic field to be entirely parallel
to the cylinder axis, and to go to zero for large r. Consider an Amperian loop of length l
extending along a radius, the inner side of which is at radius r and the outer side of which is
at a very large radius. We have on this loop
I
B dl = 0 Iencl
Letting B now be the magnetic field at radius r,
lB = 0 Iencl
0 Iencl
l
For r > 3a, Iencl = 0, since the charge on the hollow cylinders is not moving. For 2a < r < 3a,
the loop now encloses the outer surface of the dielectric. In time 2
a charge i l passes through
the loop, so the current due to the outer surface is
B=

Iout =

i l
2

and thus this is Iencl for 2a < r < 3a. For r < 2a, the loop now encloses both surfaces of the
dielectric; the inner surface contributes a current that exactly cancels the outer one, so again
Iencl = 0. Putting this together,

r < 2a
0
0
B=
2 i 2a < r < 3a

0
r > 3a
or, using our expression for i ,

0

B=
1 1 02

r < 2a
2a < r < 3a
r > 3a

c
Copyright 2012
American Association of Physics Teachers

2012 Semifinal Exam

Part A

10

Question A4
Two masses m separated by a distance l are given initial velocities v0 as shown in the diagram.
The masses interact only through universal gravitation.

v0
l
v0

a. Under what conditions will the masses eventually collide?


b. Under what conditions will the masses follow circular orbits of diameter l?
c. Under what conditions will the masses follow closed orbits?
d. What is the minimum distance achieved between the masses along their path?

Solution
a. In order for the masses to collide, the total angular momentum of the system must be zero,
which only occurs if v0 = 0.
b. In this case, the masses undergo uniform circular motion with radius

l
2

and speed v0 , so that

Gm2
mv0 2
= l
2
l
2
Gm
=2
v0 2 l
c. The masses follow closed orbits if they do not have enough energy to escape, i.e. if the total
energy of the system is negative. The total energy of the system is
Gm2
1
2 mv0 2
2
l
so that the condition required is
Gm2
<0
l
Gm
>1
v0 2 l

mv0 2

c
Copyright 2012
American Association of Physics Teachers

2012 Semifinal Exam

Part A

11

d. Note that the masses will always move symmetrically about the center of mass. Thus, in
order to be at minimum separation, their velocities must be perpendicular to the line joining
them (and will be oppositely directed). Let the minimum separation be d, and let the speed
of each mass at minimum separation be v.
L = 2mv

d
= mvd
2

The initial angular momentum is likewise mv0 l, and so by conservation of angular momentum
mvd = mv0 l
v = v0

l
d

By conservation of energy
1
Gm2
1
Gm2
2 mv0 2
= 2 mv 2
2
l
2
d
v0 2

Gm
Gm
= v2
l
d

Combining these,
l2
Gm
Gm
= v0 2 2
l
d
d

  2
 
Gm
d
Gm d
1 2
+ 2
1=0
v0 l
l
v0 l l

 


Gm d
d
1
1 2
+1 =0
l
v0 l l
v0 2

so that
d=l

or d =

l
Gm
v0 2 l

Gm
The second root is only sensible if vGm
2 > 1, and is only smaller than the first if v 2 l > 2.
0 l
0
(Note that both of these results make sense in light of the previous ones.) Thus the minimum
l
otherwise.
separation is l if vGm
2 2 and Gm
0 l
1
v0 2 l

c
Copyright 2012
American Association of Physics Teachers

2012 Semifinal Exam

Part A

12

STOP: Do Not Continue to Part B

If there is still time remaining for Part A, you should review your work for
Part A, but do not continue to Part B until instructed by your exam
supervisor.

c
Copyright 2012
American Association of Physics Teachers

2012 Semifinal Exam

Part B

13

Part B
Question B1
A particle of mass m moves under a force similar to that of an ideal spring, except that the force
repels the particle from the origin:
F = +m2 x
In simple harmonic motion, the position of the particle as a function of time can be written
x(t) = A cos t + B sin t
Likewise, in the present case we have
x(t) = A f1 (t) + B f2 (t)
for some appropriate functions f1 and f2 .
a. f1 (t) and f2 (t) can be chosen to have the form ert . What are the two appropriate values of
r?
b. Suppose that the particle begins at position x(0) = x0 and with velocity v(0) = 0. What is
x(t)?
c. A second, identical particle begins at position x(0) = 0 with velocity v(0) = v0 . The second
particle becomes closer and closer to the first particle as time goes on. What is v0 ?

Solution
a. We have
ma = m2 x
d2 x
2 x = 0
dt2
As with the case of simple harmonic motion, we insert a trial function, in this case x(t) = Aert :
d2
Aert 2 Aert = 0
dt2
r2 Aert 2 Aert = 0
r 2 2 = 0
r =
b. We have
x(t) = Aet + Bet
and therefore
v(t) = Aet Bet
Inserting our initial values,
x(0) = A + B = x0
c
Copyright 2012
American Association of Physics Teachers

2012 Semifinal Exam

Part B

14

v(0) = A B = 0
These equations have solution
A=B=
and therefore
x(t) =

x0
2

x0 t x0 t
e + e
2
2

c. This time our initial values are


x(0) = A + B = 0
v(0) = A B = v0
with solution

v0
2
v0
B=
2
A=

Therefore
x(t) =

v0 t x0 t
e
e
2
2

After a long time, the second (et ) term will become negligible. Thus, the second particle
will approach the first particle if the first term matches:
x0 t
v0 t
e =
e
2
2
v0 = x0

c
Copyright 2012
American Association of Physics Teachers

2012 Semifinal Exam

Part B

15

Question B2
For this problem, assume the existence of a hypothetical particle known as a magnetic monopole.
Such a particle would have a magnetic charge qm , and in analogy to an electrically charged
particle would produce a radially directed magnetic field of magnitude
B=

0 qm
4 r2

and be subject to a force (in the absence of electric fields)


F = qm B
A magnetic monopole of mass m and magnetic charge qm is constrained to move on a vertical,
nonmagnetic, insulating, frictionless U-shaped track. At the bottom of the track is a wire loop
whose radius b is much smaller than the width of the U of the track. The section of track near
the loop can thus be approximated as a long straight line. The wire that makes up the loop has
radius a  b and resistivity . The monopole is released from rest a height H above the bottom of
the track.
Ignore the self-inductance of the loop, and assume that the monopole passes through the loop
many times before coming to a rest.
a. Suppose the monopole is a distance x from the center of the loop. What is the magnetic flux
B through the loop?
b. Suppose in addition that the monopole is traveling at a velocity v. What is the emf E in the
loop?
c. Find the change in speed v of the monopole on one trip through the loop.
d. How many times does the monopole pass through the loop before coming to a rest?
e. Alternate Approach: You may, instead, opt to find the above answers to within a dimensionless multiplicative constant (like 23 or 2 ). If you only do this approach, you will be able
to earn up to 60% of the possible score for each part of this question.
You might want to make use of the integral
Z
1
3
du =
2
3
8
(1 + u )
or the integral
Z

sin4 d =

3
8

Solution
Version 1
The magnetic field around a monopole is given by
B=

0 qm
4 r2

c
Copyright 2012
American Association of Physics Teachers

2012 Semifinal Exam

Part B

16

The flux through the loop will then be


Z
Z
~ dA
~ = 1 0 qm sin d
B = B
2
where is the angle between a line along the axis of the loop and a line drawn between the monopole
and any point on the rim of the loop.
It is easy to see that
1
dB = 0 qm sin
2
From Faradays law, we have that a changing flux will induce a current I in the loop.
dB
= IR,
dt
where R is the resistance of the loop. Well figure R out later.
The induced current will create a magnetic field that will oppose the monopole motion. We
need to use the law of Biot & Savart to find that field. Along the axis of the loop, we have
0 I d~l ~r
,
4 r3

~ =
dB

where ~r is a vector connecting the monopole with some point on the rim of the loop. Only compo~ parallel to the axis of the loop will survive, so we can concern ourselves with
nents of B
0 I dl
sin
4 r2

dB =

The integral is trivial; dl is around the circumference; nothing else changes, so


B=

0 Ib
sin
2r2

It is better to think in terms of b, the radius of the loop, than it is to deal with r, the distance from
the rim of the loop to the monopole. In that case,
b
sin = ,
r
so
B=

0 I
(sin )3
2b

The force on the monopole is then


F = qm B = qm

0
1 dB
0 I
(sin )3 = qm (sin )3
2b
2b
R dt

Note than multiplying through by dt gives an expression that is related to the change in momentum,
dp = qm

0
1
0 2
(sin )3 dB = qm 2
(sin )4 d
2b
R
4bR

Using the provided integral,


p =

qm 2 0 2 3
4bR 8

c
Copyright 2012
American Association of Physics Teachers

2012 Semifinal Exam

Part B

17

as the monopole moves from one side to the other.


If the monopole started from rest a distance H above the loop, then the initial energy of the
system is mgH, and the initial momentum when passing through the loop (assuming there is no
loop) is then
p
p0 = m 2gH
The monopole will lose p from the momentum on each pass through the loop, so the number of
times it passes through the loop N is
N=

p
p0
= m 2gH
p

or

qm 2 0 2 3
4bR 8

1

32bRm 2gH
N=
30 2 qm 2

Oh, we still need to do R. Since a  b, we can treat it as a long thin cylindrical wire, and
R=
so we finally get

2b
a2

64b2 m 2gH
N=
30 2 qm 2 a2

Version 2
Instead of force, focus on the power dissipated in the loop, which is
P =


1
2 0 qm

P =

P =

E2
R
b2
3

(b2 +x2 ) 2
2b
a2

dx
dt

1
0 2 qm 2 a2 b3
8
(b2 + x2 )3

2

dx
dt

2

The energy is lost from the particle:


d 1
mv 2
dt 2
dv
P = mv
dt

P =

Combining these, and since v =

dx
dt ,

0 2 qm 2 a2 b3
dx
1
dv
=
2
2
3
dt
8m
(b + x ) dt
Use the provided integral (or do a trig subsitution that gives the other provided integral), and
continue.

c
Copyright 2012
American Association of Physics Teachers

United States Physics Team


F = ma Contest Papers
2013

2013 F = ma Exam

AAPT
AIP

UNITED STATES PHYSICS TEAM


CEE
2013
2013 F = ma Contest
25 QUESTIONS - 75 MINUTES
INSTRUCTIONS

DO NOT OPEN THIS TEST UNTIL YOU ARE TOLD TO BEGIN


Use g = 10 N/kg throughout this contest.
You may write in this booklet of questions. However, you will not receive any credit for anything written in
this booklet.
Your answer to each question must be marked on the optical mark answer sheet.
Select the single answer that provides the best response to each question. Please be sure to use a No. 2 pencil
and completely fill the box corresponding to your choice. If you change an answer, the previous mark must
be completely erased.
Correct answers will be awarded one point; incorrect answers will result in a deduction of
no penalty for leaving an answer blank.

1
4

point. There is

A hand-held calculator may be used. Its memory must be cleared of data and programs. You may use only
the basic functions found on a simple scientific calculator. Calculators may not be shared. Cell phones may
not be used during the exam or while the exam papers are present. You may not use any tables, books, or
collections of formulas.
This test contains 25 multiple choice questions. Your answer to each question must be marked on the optical
mark answer sheet that accompanies the test. Only the boxes preceded by numbers 1 through 25 are to be
used on the answer sheet.
All questions are equally weighted, but are not necessarily the same level of difficulty.
In order to maintain exam security, do not communicate any information about the questions
(or their answers or solutions) on this contest until after February 20, 2013.
The question booklet and answer sheet will be collected at the end of this exam. You may not use scratch
paper.

DO NOT OPEN THIS TEST UNTIL YOU ARE TOLD TO BEGIN


Contributors to this years exam include David Fallest, Jiajia Dong, Paul Stanley, Warren Turner, Qiuzi Li, and former US Team members Marianna Mao, Andrew
Lin, Steve Byrnes.

c
Copyright 2013
American Association of Physics Teachers

2013 F = ma Exam

1. An observer stands on the side of the front of a stationary train. When the train starts moving with constant
acceleration, it takes 5 seconds for the first car to pass the observer. How long will it take for the 10th car to pass?
(A) 1.07s
(B) 0.98s
(C) 0.91s
(D) 0.86s
(E) 0.81s

Solution
Start with

1 2
at + vi t
2
We have four times. t0 = 0 is when the train starts, and when the first car is aligned with the observer. t1 is
when the end of the first car is aligned with the observer. Then
x =

L=

1 2
at1
2

We are assuming the car has a length L. t2 is when the tenth car is first aligned with the observer, so
9L =

1 2
at2
2

10L =

1 2
at3
2

and finally, t3 is when that car has passed,

From the equation for t1 we find


2L/a = 25 s2
so

t2 =

and

t3 =

9 25 s2 = 15 s

10 25 s2 = 15.81 s

c
Copyright 2013
American Association of Physics Teachers

2013 F = ma Exam

2. Jordi stands 20 m from a wall and Diego stands 10 m from the same wall. Jordi throws a ball at an angle of 30
above the horizontal, and it collides elastically with the wall. How fast does Jordi need to throw the ball so that
Diego will catch it? Consider Jordi and Diego to be the same height, and both are on the same perpendicular line
from the wall.
(A) 11 m/s
(B) 15 m/s
(C) 19 m/s
(D) 30 m/s
(E) 35 m/s

Solution
The wall acts like a mirror for a perfectly elastic collision. In that case, Jordi and Diego are effectively 30
meters apart. Using the range formula,
v2
sin 2,
R=
g
we get
v2 =

gR
300 m2 /s2

sin 2
3/2
v 19 m/s

c
Copyright 2013
American Association of Physics Teachers

2013 F = ma Exam

3. Tom throws a football to Wes, who is a distance l away. Tom can control the time of flight t of the ball by choosing
any speed up to vmax and any launch angle between 0 and 90 . Ignore air resistance and assume Tom and Wes
are at the same height. Which of the following statements is incorrect?

(A) If vmax < gl, the ball cannot reach Wes at all.
(B) Assuming the ball can reach Wes, as vmax increases with l held fixed, the minimum value of t decreases.
(C) Assuming the ball can reach Wes, as vmax increases with l held fixed, the maximum value of t increases.
(D) Assuming the ball can reach Wes, as l increases with vmax held fixed, the minimum value of t increases.
(E) Assuming the ball can reach Wes, as l increases with vmax held fixed, the maximum value of t increases.

Solution
The greater the initial vertical velocity of the football, the longer it will take to fall back to the ground.
Meanwhile, the initial horizontal velocity of the football increases with l and decreases with the time of flight.
Thus the minimum time of flight is obtained by the minimum possible initial vertical velocity; it is limited
by the increasing required horizontal velocity, and this limitation becomes more severe as l increases. The
maximum time of flight is obtained by the maximum possible initial vertical velocity; it is limited by the
required horizontal velocity component, and this limitation likewise becomes more severe as l increases. Thus
increasing vmax expands the range of available time of flight in both directions; increasing l contracts it in
both directions.
(A) follows from the standard result for the maximum range of a projectile.
For those who prefer the mathematical approach, we have for a launch angle and initial speed v0
2v0 sin = gt
(v0 cos )t = l
2

Since sin + cos2 = 1,


v0 2 =

gt
2

2
+

 2
l
t

g2 4
t v0 2 t2 + l2 = 0
4
p
v0 2 v0 4 g 2 l 2
2
t =2
g2
(A) follows immediately from the requirement that the result be real. It is also clear that increasing l contracts
the range of available times in both directions, and that increasing vmax increases the maximum available
time. As for the minimum available time, we have
p
v0 2 v0 4 g 2 l 2
2
t = 2
g2
!

t 2
2
2v0 2
= 2 1 p
(v0 2 )
g
2 v0 4 g 2 l 2


t 2
2
1

= 2 1 q
2 2
(v0 2 )
g
1 g l
v0 4

The right hand side is always negative, so t is always minimized by choosing the largest possible v0 .

c
Copyright 2013
American Association of Physics Teachers

2013 F = ma Exam

4. The sign shown below consists of two uniform legs attached by a frictionless hinge. The coefficient of friction
between the ground and the legs is . Which of the following gives the maximum value of such that the sign will
not collapse?

(A) sin = 2
(B) sin /2 = /2
(C) tan /2 =
(D) tan = 2
(E) tan /2 = 2

Solution
For each leg, friction must balance the horizontal force from the other leg at the top. Balancing torques,
L
(M/2)g sin = LFf cos
2
where Ff is the force of friction on the leg, and = 21 .
The upward normal force on the leg is equal to the weight of the leg, so
FN = (M/2)g
Equating,
FN = Ff
or

L
(M/2)g sin = L(M/2)g cos
2

or
tan /2 = 2

c
Copyright 2013
American Association of Physics Teachers

2013 F = ma Exam

The following information applies to questions 5 and 6


A student steps onto a stationary elevator and stands on a bathroom scale. The elevator then travels from the top
of the building to the bottom. The student records the reading on the scale as a function of time.
120

Scale reading (kg)

100
80
60
40
20
0

10

12

14

16

18

20

22

24

26

Time (s)
5. At what time(s) does the student have maximum downward velocity?
(A) At all times between 2 s and 4 s
(B) At 4 s only
(C) At all times between 4 s and 22 s
(D) At 22 s only
(E) At all times between 22 s and 24 s
6. How tall is the building?
(A) 50 m
(B) 80 m
(C) 100 m
(D) 150 m
(E) 400 m

Solution
The bathroom scale does not directly measure the weight of the student; instead it measures the normal force
FN supporting her feet, scaled by g.
mscale g = FN
The normal force acts upward on the student. Gravity, of course, always exerts a force mg downward. Thus
her downward acceleration a is given by
ma = mg FN
Combining these and rearranging,


mscale 
a=g 1
m

Since the elevator begins at rest, the students acceleration is initially zero, so her mass must be 80 kg. From

c
Copyright 2013
American Association of Physics Teachers

2013 F = ma Exam

the graph we see that there are two periods of uniform acceleration; the first is downward with magnitude


60 kg
2
a = (10 m/s ) 1
= 2.5 m/s2
80 kg
and the second is upward with the same magnitude. The maximum downward velocity occurs between the
periods of uniform acceleration, when the car travels at a constant speed; this occurs between 4 s and 22 s.
The downward acceleration lasts for 2 s, so the speed of the elevator after the acceleration is 5 m/s. The
period between accelerations is approximately 20 s, and so the distance traveled is approximately 100 m. In
fact, this calculation yields the exact answer, as can be seen by considering the exact graph of velocity vs.
time:
0

Velocity (m/s)

1
2
3
4
5

10

12

14

16

18

20

22

24

26

Time (s)
The displacement is the area between the curve and the horizontal axis, which is exactly 100 m.

7. A light car and a heavy truck have the same momentum. The truck weighs ten times as much as the car. How do
their kinetic energies compare?
(A) The trucks kinetic energy is larger by a factor of 100
(B) They trucks kinetic energy is larger by a factor of 10
(C) They have the same kinetic energy
(D) The cars kinetic energy is larger by a factor of 10
(E) The cars kinetic energy is larger by a factor of 100

Solution
Express kinetic energy in terms of momentum,
K=
Then the ratio is

p2
2m

K1
m2
=
K2
m1

c
Copyright 2013
American Association of Physics Teachers

2013 F = ma Exam

The following information applies to questions 8 and 9


A truck is initially moving at velocity v. The driver presses the brake in order to slow the truck to a stop. The
brake applies a constant force F to the truck. The truck rolls a distance x before coming to a stop, and the time
it takes to stop is t.
8. Which of the following expressions is equal the initial kinetic energy of the truck (i.e. the kinetic energy before the
driver starts braking)?
(A) F x
(B) F vt
(C) F xt
(D) F t
(E) Both (a) and (b) are correct

Solution
Considering magnitudes only,
Ki = K = W = F x

9. Which of the following expressions is equal the initial momentum of the truck (i.e. the momentum before the driver
starts braking)?
(A) F x
(B) F t/2
(C) F xt
(D) 2F t
(E) 2F x/v

Solution
Considering magnitudes only,
pi = p = F t
but
x=

1
1
1 2
at = (at)t = vt
2
2
2

so
pi = 2F x/v

c
Copyright 2013
American Association of Physics Teachers

2013 F = ma Exam

10. Which of the following can be used to distinguish a solid ball from a hollow sphere of the same radius and mass?
(A) Measurements of the orbit of a test mass around the object.
(B) Measurements of the time it takes the object to roll down an inclined plane.
(C) Measurements of the tidal forces applied by the object to a liquid body.
(D) Measurements of the behavior of the object as it floats in water.
(E) Measurements of the force applied to the object by a uniform gravitational field.

Solution
The measurement described in (B) is certainly appropriate; the solid ball has a smaller moment of inertia
than the hollow sphere and will accelerate down the inclined plane faster.
The measurements in (A) and (C) are unhelpful because they only probe the gravitational field outside the
object; for a spherically symmetric object this depends only on the mass. The force measured in (E) depends
only on the objects total mass; the buoyant force applied in (D) depends only on the external shape of the
object.

c
Copyright 2013
American Association of Physics Teachers

2013 F = ma Exam

10

11. A right-triangular wooden block of mass M is at rest on a table, as shown in figure. Two smaller wooden cubes,
both with mass m, initially rest on the two sides of the larger block. As all contact surfaces are frictionless, the
smaller cubes start sliding down the larger block while the block remains at rest. What is the normal force from
the system to the table?

90

(A) 2mg
(B) 2mg + M g
(C) mg + M g
(D) M g + mg(sin + sin )
(E) M g + mg(cos + cos )

Solution
Two forces act on each cube: the normal force from the triangular block, and gravity. The normal force must
balance the normal component of gravity, which in the case of the left cube is
FN = mg cos
The vertical component of this normal force is transmitted through the triangular block to the ground, and
is
FN y = mg cos2
A similar result holds for the other cube, and in addition the ground must support the weight of the triangular
block; thus in total
Ftot = mg cos2 + mg cos2 + M g
However, because the block is a right triangle, cos2 + cos2 = 1, so that
Ftot = mg + M g
Note that the horizontal component of the normal force due to the left cube is
FN x = mg cos sin
The right cube likewise applies a normal force with horizontal component mg cos sin in the other direction.
But, once again, because the block is a right triangle, cos sin = cos sin , so the net horizontal force is
zero! This justifies the given assumption that the triangular block does not slide on the table.

c
Copyright 2013
American Association of Physics Teachers

2013 F = ma Exam

11

12. A spherical shell of mass M and radius R is completely filled with a frictionless fluid, also of mass M . It is released
from rest, and then it rolls without slipping down an incline that makes an angle with the horizontal. What will
be the acceleration of the shell down the incline just after it is released? Assume the acceleration of free fall is g.
The moment of inertia of a thin shell of radius r and mass m about the center of mass is I = 32 mr2 ; the moment
of inertia of a solid sphere of radius r and mass m about the center of mass is I = 25 mr2 .
(A) a = g sin
(B) a = 43 g sin
(C) a = 12 g sin
(D) a = 38 g sin
(E) a = 35 g sin

Solution
One can use torque or energy to solve this problem.
The torque about an axis through the point of contact is
= RF sin = 2M gR sin
The angular acceleration is given by
= I
where the moment of inertia is
I=

2
8
M R2 + M R2 + M R2 = M R2
3
3

The acceleration is then


a = R =

2M gR sin
3
R = g sin
8
2
4
M
R
3

Alternatively, the kinetic energy of the object is


T =

1
1 2
(2M )v 2 + M R2 2
2
2 3

4
M v2
3
The potential energy is related to the vertical position y by
T =

U = (2M )gy
and so by conservation of energy
d
(T + U ) = 0
dt
8
dv
dy
Mv
= (2M )g
3
dt
dt
But

and so

dy
= v sin
dt
3
dv
= g sin
dt
4

c
Copyright 2013
American Association of Physics Teachers

2013 F = ma Exam

12

13. There is a ring outside of Saturn. In order to distinguish if the ring is actually a part of Saturn or is instead part
of the satellites of Saturn, we need to know the relation between the velocity v of each layer in the ring and the
distance R of the layer to the center of Saturn. Which of the following statements is correct?
(A) If v R, then the layer is part of Saturn.
(B) If v 2 R, then the layer is part of the satellites of Saturn.
(C) If v 1/R, then the layer is part of Saturn.
(D) If v 2 1/R, then the layer is part of Saturn.
(E) If v R2 , then the layer is part of the satellites of Saturn.

Solution
If attached to Saturn, then = v/R is a constant, so
vR
If in orbit, then
GM
v2
= 2 ,
R
r
or
v 2 1/R

14. A cart of mass m moving at 12 m/s to the right collides elastically with a cart of mass 4.0 kg that is originally
at rest. After the collision, the cart of mass m moves to the left with a velocity of 6.0 m/s. Assuming an elastic
collision in one dimension only, what is the velocity of the center of mass (vcm ) of the two carts before the collision?
(A) vcm = 2.0 m/s
(B) vcm = 3.0 m/s
(C) vcm = 6.0 m/s
(D) vcm = 9.0 m/s
(E) vcm = 18 m/s

Solution
In the center of mass frame, the carts initially have equal and opposite momenta, so that the total momentum
is zero. After the elastic collision in this frame the momenta simply reverse direction; this clearly conserves
total energy and momentum. The frame of reference in which the velocity of the cart of mass m is reversed
travels to the right at 3 m/s, so that the initial velocity is 9 m/s to the right and the final velocity is 9 m/s
to the left.
Note that the mass and initial velocity of the other cart are not required at all!

c
Copyright 2013
American Association of Physics Teachers

2013 F = ma Exam

13

15. A uniform rod is partially in water with one end suspended, as shown in figure. The density of the rod is 5/9 that
of water. At equilibrium, what portion of the rod is above water?

11111111111111111
00000000000000000
00000000000000000
11111111111111111
00000000000000000
11111111111111111
00000000000000000
11111111111111111
00000000000000000
11111111111111111
00000000000000000
11111111111111111
00000000000000000
11111111111111111
00000000000000000
11111111111111111
00000000000000000
11111111111111111
00000000000000000
11111111111111111
(A) 0.25
(B) 0.33
(C) 0.5
(D) 0.67
(E) 0.75

Solution
Suppose a fraction of the rod is above water. Let the length of the rod be l, the volume of the rod be V ,
the density of water be w , and the density of the rod be r . Consider torques about the pivot point. Gravity
applies a torque
1
g = r V g l
2
A fraction 1 of the rod is submerged, and the center of the submerged portion is a distance ( + 12 (1 ))l
from the pivot. So the buoyant force applies a torque


1
b = w (1 )V g + (1 ) l
2
b =

1
w (1 2 )V gl
2

These torques must balance:


g = b
1
1
r = w (1 2 )
2
2
r
= 1 2
w
We are given

r
w

= 95 , so that
5
= 1 2
9
2
=
3

c
Copyright 2013
American Association of Physics Teachers

2013 F = ma Exam

14

16. Inspired by a problem from the 2012 International Physics Olympiad, Estonia.
A very large number of small particles forms a spherical cloud. Initially they are at rest, have uniform mass density
per unit volume 0 , and occupy a region of radius r0 . The cloud collapses due to gravitation; the particles do not
interact with each other in any other way.
q
How much time passes until the cloud collapses fully? (The constant 0.5427 is actually 3
32 .)
(A)

0.5427

r0 2 G0

(B)

0.5427

r0 G0

(C)

0.5427
r0 G0

(D)

0.5427

G0

(E)

0.5427

r
G0 0

Solution
This problem is a matter of dimensional analysis. All of the answers have the form
1/2

t = 0.527G1/2 0

r0 n

The dimensions of t are [T ], the dimensions of G are [L]3 [M ]1 [T ]2 , the dimensions of 0 are [L]3 [M ], and
the dimensions of r0 are [L]. So
[T ] = ([L]3 [M ]1 [T ]2 )1/2 ([L]3 [M ])1/2 [L]n
[T ] = [T ][L]n
n=0
The time to collapse does not depend on the size of the cloud at all!

c
Copyright 2013
American Association of Physics Teachers

2013 F = ma Exam

15

17. Two small, equal masses are attached by a lightweight rod. This object orbits a planet; the length of the rod is
smaller than the radius of the orbit, but not negligible. The rod rotates about its axis in such a way that it remains
vertical with respect to the planet.
Is there a force in the rod? If so, is it tension or compression?
Is the equilibrium stable, unstable, or neutral with respect to a small perturbation in the angle of the
rod? (Assume this perturbation maintains the rate of rotation, so that in the co-rotating frame the rod
is still stationary but at an angle to the vertical.)

(A) There is no force in the rod; the equilibrium is neutral.


(B) The rod is in tension; the equilibrium is stable.
(C) The rod is in compression; the equilibrium is stable.
(D) The rod is in tension; the equilibrium is unstable.
(E) The rod is in compression; the equilibrium is unstable.

Solution
Because the gravitational force goes as 1/r2 , the inward force on the inner mass is greater than the inward
force on the outer mass. On the other hand, both masses have the same angular velocity, so the centripetal
acceleration goes as r, and the outer mass must be subject to a greater centripetal force than the inward one.
So the rod must exert an inward force on the outer mass and an outward force on the inner mass. Thus the
rod is in tension.
The stability of the position is best understood in the corotating frame. In this frame the outer mass
experiences an outward radial force and the inward tension of the rod, and the inner mass experiences an
inward radial force and the outward tension of the rod. When the rod is rotated slightly these forces create
a restoring torque, so the equilibrium is stable.

c
Copyright 2013
American Association of Physics Teachers

2013 F = ma Exam

16

18. Two point particles, each of mass 1 kg, begin in the state shown below.
3
2
1.0 m/s
y (m)

1
0
1.0 m/s

1
2
3
3 2 1

x (m)

The system evolves through internal forces only. Which of the following could be the state after some time has
passed?
3

1
1.0 m/s

0
1.0 m/s

(B)

y (m)

(A)

y (m)

2.0 m/s
0
1

3
3 2 1

3
3 2 1

x (m)

x (m)

1.0 m/s
1

1.0 m/s

0
1

(D)

1.0 m/s

y (m)

(C)

y (m)

0
1
1.0 m/s

3
3 2 1

3
3 2 1

x (m)

x (m)

3
2
1.0 m/s

(E)

y (m)

1
0
1
1.0 m/s
2
3
3 2 1

x (m)

Solution
First, linear momentum is conserved; as it happens, the system begins with zero linear momentum. This
eliminates choice (B).
c
Copyright 2013
American Association of Physics Teachers

2013 F = ma Exam

17

Angular momentum about any point is also conserved. (It is specified that the particles are point particles
to rule out their having spin angular momentum.) As it happens, the system has zero linear momentum, so
the angular momentum is the same measured about any point; otherwise it would suffice to consider any one
point. The total angular momentum must be 2 kg m2 /s counterclockwise. This eliminates choices (C) and
(D).
Finally, because the linear momentum of the system is zero, the center of mass does not move. This eliminates
choice (A).
The following information applies to questions 19, 20, and 21.
A simple pendulum experiment is constructed from a point mass m attached to a pivot by a massless rod of length
L in a constant gravitational field. The rod is released from an angle 0 < /2 at rest and the period of motion is
found to be T0 . Ignore air resistance and friction.
19. At what angle g during the swing is the tension in the rod the greatest?
(A) The tension is the greatest at the point g = 0 .
(B) The tension is the greatest at the point g = 0.
(C) The tension is the greatest at an angle g with 0 < g < 0 .
(D) The tension is constant.
(E) None of the above is true for all values of 0 with 0 < 0 < /2.
20. What is the maximum value of the tension in the rod?
(A) mg
(B) 2mg
(C) mL0 /T0 2
(D) mg sin 0
(E) mg(3 2 cos 0 )
21. The experiment is repeated with a new pendulum with a rod of length 4L, using the same angle 0 , and the period
of motion is found to be T . Which of the following statements is correct?
(A) T = 2T0 regardless of the value of 0 .
(B) T > 2T0 with T 2T0 if 0  1.
(C) T < 2T0 with T 2T0 if 0  1.
(D) T > 2T0 for some values of 0 and T < 2T0 for other values of 0 .
(E) T0 and T are undefined because the motion is not periodic unless 0  1.

Solution
The mass accelerates towards the pivot due to centripetal acceleration; if its speed is v, this acceleration is
ac =

v2
L

Two forces act in the radial direction; the tension in the rod, inward, and the radial component of gravity,
outward. At an angle to the vertical the radial component of gravity is
Fg,rad = mg cos
From Newtons second law we thus have for the tension F
F Fg,rad = mac
c
Copyright 2013
American Association of Physics Teachers

2013 F = ma Exam

18

v2
L
Both terms increase as the mass approaches the bottom of the swing, so the maximum tension certainly
occurs there. At that point the mass has traveled a vertical distance L(1 cos 0 ), so from conservation of
energy
1
mv 2 = mgL(1 cos 0 )
2
and so at the bottom
F = mg + 2mg(1 cos 0 )
F = mg cos + m

F = mg(3 2 cos 0 )
The motion is certainly periodic (one argument considers the fact that energy is conserved and the particle
eventually returns to rest). From dimensional analysis the period must take the form
s
l0
T = f (0 )
g
Therefore, for any fixed 0 , the period goes as

l0 .

22. A simplified model on the foot is shown. When a student of mass m = 60 kg stands on a single toe, the tension T
in the Achilles Tendon is closest to
Leg Bone
Achilles
Tendon

Ankle Joint
Approximation of Foot Bone
Outline of Foot
Toe

12

16

20

24

28

32

36

40

Centimeter Rule

(A) T = 600 N
(B) T = 1200 N
(C) T = 1800 N
(D) T = 2400 N
(E) T = 3000 N

Solution
The entire weight of the student, W = mg = 600 N, is supported by the toe. Balancing torques about the
ankle,
T 5 cm = 600 N 20 cm
T = 2400 N

c
Copyright 2013
American Association of Physics Teachers

2013 F = ma Exam

19

The following information applies to questions 23 and 24


A man with mass m jumps off of a high bridge with a bungee cord attached to his ankles. The man falls through
a maximum distance H at which point the bungee cord brings him to a momentary rest before he bounces back
up. The bungee cord is perfectly elastic, obeying Hookes force law with a spring constant k, and stretches from
an original length of L0 to a final length L = L0 + h. The maximum tension in the Bungee cord is four times the
weight of the man.
23. Determine the spring constant k.
(A) k =
(B) k =
(C) k =
(D) k =
(E) k =

mg
h
2mg
h
mg
H
4mg
H
8mg
H

24. Find the maximum extension of the bungee cord h.


(A) h = 12 H
(B) h = 41 H
(C) h = 51 H
(D) h = 52 H
(E) h = 81 H

Solution
At the moment of maximum extension, all of the gravitational potential energy has been converted to spring
potential energy.
1
mgH = kh2
2
We are given that the maximum tension in the cord is four times the weight of the man; this occurs at the
moment of maximum extension.
kh = 4mg
These can be solved to yield
8mg
H
1
h= H
2

k=

as well as
k=

4mg
h

although the latter is not an available answer choice.

c
Copyright 2013
American Association of Physics Teachers

2013 F = ma Exam

20

25. A box with weight W will slide down a 30 incline at constant speed under the influence of gravity and friction
alone. If instead a horizontal force P is applied to the box, the box can be made to move up the ramp at constant
speed. What is the magnitude of P ?
(A) P = W/2

(B) P = 2W/ 3
(C) P = W

(D) P = 3W
(E) P = 2W

Solution
If block slides down at constant speed, then
tan =
If block is pushed by horizontal force P up ramp at constant speed, then
FN = mg cos + P sin
so friction is
Ff = (mg cos + P sin )
This is balanced by gravity and the push up the ramp, so
P cos = (mg cos + P sin ) + mg sin
Divide through by cos , and use first equation
P = (W + P ) + W
or
P =
or
P =

2
W
1 2
2 13
1

1
3

c
Copyright 2013
American Association of Physics Teachers

United States Physics Team


Semi Final Contest Papers
2013

2013 Semifinal Exam

AAPT
AIP

UNITED STATES PHYSICS TEAM


CEE
2013

Semifinal Exam

DO NOT DISTRIBUTE THIS PAGE


Important Instructions for the Exam Supervisor
This examination consists of two parts.
Part A has four questions and is allowed 90 minutes.
Part B has two questions and is allowed 90 minutes.
The first page that follows is a cover sheet. Examinees may keep the cover sheet for both
parts of the exam.
The parts are then identified by the center header on each page. Examinees are only allowed
to do one part at a time, and may not work on other parts, even if they have time remaining.
Allow 90 minutes to complete Part A. Do not let students look at Part B. Collect the answers
to Part A before allowing the examinee to begin Part B. Examinees are allowed a 10 to 15
minutes break between parts A and B.
Allow 90 minutes to complete Part B. Do not let students go back to Part A.
Ideally the test supervisor will divide the question paper into 3 parts: the cover sheet (page 2),
Part A (pages 3-9), and Part B (pages 11-17). Examinees should be provided parts A and B
individually, although they may keep the cover sheet.
The supervisor must collect all examination questions, including the cover sheet, at the end
of the exam, as well as any scratch paper used by the examinees. Examinees may not take
the exam questions. The examination questions may be returned to the students after April
1, 2013.
Examinees are allowed calculators, but they may not use symbolic math, programming, or
graphic features of these calculators. Calculators may not be shared and their memory must
be cleared of data and programs. Cell phones, PDAs or cameras may not be used during
the exam or while the exam papers are present. Examinees may not use any tables, books,
or collections of formulas.
Please provide the examinees with graph paper for Part A. A straight edge or
ruler could also be useful.

c
Copyright 2013
American Association of Physics Teachers

2013 Semifinal Exam

AAPT
AIP

Cover Sheet

UNITED STATES PHYSICS TEAM


CEE
2013

Semifinal Exam
INSTRUCTIONS
DO NOT OPEN THIS TEST UNTIL YOU ARE TOLD TO BEGIN
Work Part A first. You have 90 minutes to complete all four problems. Each question is
worth 25 points. Do not look at Part B during this time.
After you have completed Part A you may take a break.
Then work Part B. You have 90 minutes to complete both problems. Each question is worth
50 points. Do not look at Part A during this time.
Show all your work. Partial credit will be given. Do not write on the back of any page. Do
not write anything that you wish graded on the question sheets.
Start each question on a new sheet of paper. Put your AAPT ID number, your name, the
question number and the page number/total pages for this problem, in the upper right hand
corner of each page. For example,
AAPT ID #
Doe, Jamie
A1 - 1/3
A hand-held calculator may be used. Its memory must be cleared of data and programs. You
may use only the basic functions found on a simple scientific calculator. Calculators may not
be shared. Cell phones, PDAs or cameras may not be used during the exam or while the
exam papers are present. You may not use any tables, books, or collections of formulas.
Questions with the same point value are not necessarily of the same difficulty.
In order to maintain exam security, do not communicate any information about
the questions (or their answers/solutions) on this contest until after April 1, 2013.
Possibly Useful Information. You may
g = 9.8 N/kg
k = 1/40 = 8.99 109 N m2 /C2
c = 3.00 108 m/s
NA = 6.02 1023 (mol)1
= 5.67 108 J/(s m2 K4 )
1eV = 1.602 1019 J
me = 9.109 1031 kg = 0.511 MeV/c2
sin 16 3 for ||  1

use this sheet for both parts of the exam.


G = 6.67 1011 N m2 /kg2
km = 0 /4 = 107 T m/A
kB = 1.38 1023 J/K
R = NA kB = 8.31 J/(mol K)
e = 1.602 1019 C
h = 6.63 1034 J s = 4.14 1015 eV s
(1 + x)n 1 + nx for |x|  1
cos 1 21 2 for ||  1

c
Copyright 2013
American Association of Physics Teachers

2013 Semifinal Exam

Part A

Part A
Question A1
The flow of heat through a material can be described via the thermal conductivity . If the two faces
of a slab of material with thermal conductivity , area A, and thickness d are held at temperatures
differing by T , the thermal power P transferred through the slab is
AT
d
A heat exchanger is a device which transfers heat between a hot fluid and a cold fluid; they are
common in industrial applications such as power plants and heating systems. The heat exchanger
shown below consists of two rectangular tubes of length l, width w, and height h. The tubes are
separated by a metal wall of thickness d and thermal conductivity . Originally hot fluid flows
through the lower tube at a speed v from right to left, and originally cold fluid flows through the
upper tube in the opposite direction (left to right) at the same speed. The heat capacity per unit
volume of both fluids is c.
The hot fluid enters the heat exchanger at a higher temperature than the cold fluid; the difference
between the temperatures of the entering fluids is Ti . When the fluids exit the heat exchanger
the difference has been reduced to Tf . (It is possible for the exiting originally cold fluid to have
a higher temperature than the exiting originally hot fluid, in which case Tf < 0.)
P =

h
d
h

v
l

v
w

Assume that the temperature in each pipe depends only on the lengthwise position, and consider
transfer of heat only due to conduction in the metal and due to the bulk movement of fluid. Under
the assumptions in this problem, while the temperature of each fluid varies along the length of the
exchanger, the temperature difference across the wall is the same everywhere. You need not prove
this.
Find Tf in terms of the other given parameters.

Solution
Suppose the temperature difference across the wall is Tw . Since the total area of the wall is
simply lw, the power transferred across the wall is
P =

lw
Tw
d

c
Copyright 2013
American Association of Physics Teachers

2013 Semifinal Exam

Part A

In a time dt, the energy transferred is therefore


dE = P dt =

lw
Tw dt
d

Meanwhile, suppose the red fluid enters at temperature Tr and the blue fluid at temperature
Tb . The red fluid then exits at temperature Tb + Tw , so the overall temperature change of the red
fluid is
Tr = Tr (Tb + Tw ) = Ti Tw
In a time dt, a volume of red fluid vwh dt flows through the pipe; the heat capacity of this amount
of fluid is vwhc dt, so the energy transferred out of it is therefore
dE = vwhc dt Tr = vwhc(Ti Tw ) dt
(Note that the same result is obtained for the heat transferred into the blue fluid; if the flow rates or
heat capacities were not the same, this would not hold, exposing the fact that Tw is not constant
in that case.)
Equating,
lw
Tw = vwhc(Ti Tw )
d
Ti
Tw =
l
1 + dvhc
Because the red fluid exits at Tb + Tw , and the blue fluid at Tr Tw ,
Tf = (Tb + Tw ) (Tr Tw ) = Ti + 2Tw
!
2
1
Tf = Ti
l
1 + dvhc
l
; as
The performance of the heat exchanger is controlled entirely by the dimensionless parameter dvhc
might be intuitive, a long tube and high conductivity are beneficial, whereas a thick wall, high flow
rate, and high heat capacity is not. The poorest performance, unsurprisingly, reflects essentially
no heat transfer, with the red fluid and blue fluid exiting with the same temperatures they started
with. Interestingly, the limit of performance is a complete reversal in the temperatures of the two
fluids, with Tf Ti .

c
Copyright 2013
American Association of Physics Teachers

2013 Semifinal Exam

Part A

Question A2
A solid round object of radius R can roll down an incline that makes an angle with the horizontal.
Assume that the rotational inertia about an axis through the center of mass is given by I = mR2 .
The coefficient of kinetic and static friction between the object and the incline is . The object
moves from rest through a vertical distance h.
a. If the angle of the incline is sufficiently large, then the object will slip and roll; if the angle
of the incline is sufficiently small, then the object with roll without slipping. Determine the
angle c that separates the two types of motion.
b. Derive expressions for the linear acceleration of the object down the ramp in the case of
i. Rolling without slipping, and
ii. Rolling and slipping.

Solution
As the object rolls down the incline, there is a torque about the center of mass given by
= Rf
where f is the force of friction. The angular acceleration is then
=

f
=
I
mR

or, as will be more useful,


f = mR
The object experience a linear acceleration down the incline given by
ma = mg sin f
We have to cases to consider. Either the object rolls without slipping so that f mg cos
and a = R, or the object rolls while slipping so that f = mg cos and a > R.
Rolling without slipping
Combining the equalities, we get
ma = mg sin ma
or
a=g

sin
1+

Rolling while slipping


Combining the equalities, we get
ma = mg sin mg cos
or
a = g (sin cos )
c
Copyright 2013
American Association of Physics Teachers

2013 Semifinal Exam

Part A

The motion is changes at an angle where the static friction is greatest, or when both conditions
are equalities:
f = mg cos
and
a = R
In that case

sin c
= sin c cos c
1+


1
+1
tan c =

or

Question A3
A beam of muons is maintained in a circular orbit by a uniform magnetic field. Neglect energy loss
due to electromagnetic radiation.
The mass of the muon is 1.88 1028 kg, its charge is 1.602 1019 C, and its half-life is
1.523 s.
a. The speed of the muons is much less than the speed of light. It is found that half of the
muons decay during each full orbit. What is the magnitude of the magnetic field?
b. The experiment is repeated with the same magnetic field, but the speed of the muons is
increased; it is no longer much less than the speed of light. Does the fraction of muons which
decay during each full orbit increase, decrease, or stay the same?
The following facts about special relativity may be useful:
The Lorentz factor for a particle moving at speed v is
1
=p
1 v 2 /c2
The Lorentz factor gives the magnitude of time dilation; that is, a clock moving at speed
v in a given reference frame runs slow by a factor in that frame.
The momentum of a particle is given by
p~ = m~v
where m does not depend on v.
The Lorentz force law in the form
d~
p
~ + ~v B)
~
= q(E
dt
continues to hold.

c
Copyright 2013
American Association of Physics Teachers

2013 Semifinal Exam

Part A

Solution
For brevity we simply present the full relativistic solution.
The relationships for circular motion

d~v |~v |2
=
dt
r
2r = |~v | T
are purely a matter of mathematics, and thus continue to hold under special relativity. Meanwhile,
since |~v | is constant for circular motion, is constant as well. Thus we can take magnitudes in the
~ = 0) to find
Lorentz force law (and set E

d~v
m = q |~v | B
dt
Combining these relationships,
2
m = qB
T
If half of the muons decay during each orbit, in the muons frame of reference each orbit takes one
half-life T1/2 . In the lab frame, then,
T = T1/2
and so

2
m = qB
T1/2
B=

2m
qT1/2

Numerically,
B = 4.85 mT
The speed of the muons is irrelevant to the fraction which decay per orbit, even in the relativistic
case. (The orbits take longer, but the muons live longer, both by the same factor .)

Question A4
A graduated cylinder is partially filled with water; a rubber duck floats at the surface. Oil is poured
into the graduated cylinder at a slow, constant rate, and the volume marks corresponding to the
surface of the water and the surface of the oil are recorded as a function of time.

c
Copyright 2013
American Association of Physics Teachers

2013 Semifinal Exam

Part A

500
Volume mark at water surface
Volume mark at oil surface

450
400

Volume (mL)

350
300
250
200
150
100
50
0

4
5
6
Time (min)

10

Water has a density of 1.00 g/mL; the density of air is negligible, as are surface effects. Find
the density of the oil.

Solution
As the oil is poured in, more and more of the weight of the duck is supported by oil, and it rises
out of the water, reducing the water level. Eventually this stops, either because the duck is fully
submerged in oil or because it is floating entirely above the water. At all times, the weight of the
water that is no longer displaced equals the weight of the newly displaced oil:
o gVo = w gVw
With this understanding many approaches are possible; we illustrate one. The change in the
volume of displaced water is easily read off the graph as the distance between the dotted and
dashed lines; it is 143 mL. Finding the volume of displaced oil requires us to take into account the
increasing amount of oil in the cylinder. We know there is no oil at t = 0, because the oil level
and water level coincide, and we know that the rate of change of the oil level for t > 6 min is the
pour rate, because the water level is not changing. Extrapolating to t = 0 we conclude that the
volume of oil in the container at any time is given by the height of the shaded region. The volume
of displaced oil can then be read as the distance between the solid and dashed lines; it is 186 mL.
The density of the oil is
Vw
143 mL
o = w
= (1.00 g/mL)
Vo
186 mL
c
Copyright 2013
American Association of Physics Teachers

2013 Semifinal Exam

Part A

o = 0.77 g/mL
500
Volume mark at water surface
Volume mark at oil surface

450
400

Volume (mL)

350
300
250
200
150
100
50
0

4
5
6
Time (min)

c
Copyright 2013
American Association of Physics Teachers

10

2013 Semifinal Exam

Part A

10

STOP: Do Not Continue to Part B

If there is still time remaining for Part A, you should review your work for
Part A, but do not continue to Part B until instructed by your exam
supervisor.

c
Copyright 2013
American Association of Physics Teachers

2013 Semifinal Exam

Part B

11

Part B
Question B1
Shown below is the Blackbird, a vehicle built in 2009.
There is no source of stored energy such as a battery or gasoline engine; all of the power used
to move the car comes from the wind. The only important mechanism in the car is a gearbox that
can transfer power between the wheels and the propeller.
The Blackbird was driven both directly downwind and directly upwind, as shown below. In
each case the car remained exactly parallel (or anti-parallel) to the wind without turning. The
tests were conducted on level ground, in steady, uniform wind, and continued long enough to reach
the steady state.
~vw

~v

Source: fasterthanthewind.org

Downwind

~vw

~v

Upwind

When driving downwind, the builders claim that they were able to drive faster than the wind:
that is, with |~v | > |~vw |, so that the car experienced a relative headwind while traveling. Commenters
on the Internet claimed, often angrily, that this was physically impossible and that the Blackbird
was a hoax. Some commenters also claimed that the upwind case was physically impossible.
a. Consider first the downwind faster than the wind case.
Is the motion actually possible as claimed? If not, offer a brief explanation!
If the motion is possible, is power transferred from the propeller to the wheels or vice
versa?
If the motion is possible, what ground speed is attained? For this question, suppose
that when transferring power in either direction between the propeller and the wheels,
a fraction of the useful work is lost; let the wind speed be vw . Neglect all other losses
of energy.
b. Answer the previous questions for the upwind case.

Solution

c
Copyright 2013
American Association of Physics Teachers

2013 Semifinal Exam

Part B

12

Both modes are possible as claimed.


When a force exists between two bodies in relative motion, the net power produced or consumed
is proportional to the relative velocity between them:
P = F vr
This value is independent of the reference frame in which the power is measured, even though the
power delivered to each body separately is not. This is a consequence of Newtons third law; if the
velocities of the bodies in a particular reference frame are v1 and v2 , the net power is
P = F v1 + (F )v2
P = F (v1 v2 )
The relative speed of the Blackbird and the air is different from the relative speed of the
Blackbird and the ground; if the ground speed is v, the airspeed is v vw in the downwind mode
and v + vw in the upwind mode. Thus, even though the force due to the air and the force due to
the ground must balance, one force can produce more power than the other one absorbs.
Power should always be produced by the force corresponding to the larger relative velocity.
Thus in the downwind case, power is transferred from the wheels to the propeller,
whereas in the upwind case, power is transferred from the propeller to the wheels. (In
practice this reversal required a lengthy reconfiguration of the Blackbird between the two trials.)
In the downwind case, the wheel power is
Pw = F v
and the propeller power is
Pp = F (v vw )
Power is transferred to the propeller, so
Pp = (1 )Pw
F (v vw ) = (1 )F v
vw
v=

At high energy loss one moves close to the wind speed (as expected); with sufficiently low energy
loss, any speed is possible.
In the upwind case, the wheel power is still
Pw = F v
but the propeller power is
Pp = F (v + vw )
Power is transferred to the wheels, so
Pw = (1 )Pp
F v = (1 )F (v + vw )


1
v = vw
1

At high energy loss one cannot make any progress at all (as expected); again, with sufficiently low
energy loss, any speed is possible.
c
Copyright 2013
American Association of Physics Teachers

2013 Semifinal Exam

Part B

13

Question B2
This problem concerns three situations involving the transfer of energy into a region of space by
electromagnetic fields. In the first case, that energy is stored in the kinetic energy of a charged
object; in the second and third cases, the energy is stored in an electric or magnetic field.
In general, whenever an electric and a magnetic field are at an angle to each other, energy
is transferred; for example, this principle is the reason electromagnetic radiation transfers energy.
The power transferred per unit area is given by the Poynting vector :
~= 1E
~ B
~
S
0
In each part of this problem, the last subpart asks you to verify that the rate of energy transfer
agrees with the formula for the Poynting vector. Therefore, you should not use the formula for the
Poynting vector before the last subpart!
a. A long, insulating cylindrical rod has radius R and carries a uniform volume charge density
. A uniform external electric field E exists in the direction of its axis. The rod moves in the
direction of its axis at speed v.
i. What is the power per unit length P delivered to the rod?
ii. What is the magnetic field B at the surface of the rod? Draw the direction on a diagram.
iii. Compute the Poynting vector, draw its direction on a diagram, and verify that it agrees
with the rate of energy transfer.
b. A parallel plate capacitor consists of two discs of radius R separated by a distance d  R.
The capacitor carries charge Q, and is being charged by a small, constant current I.
i. What is the power P delivered to the capacitor?
ii. What is the magnetic field B just inside the edge of the capacitor? Draw the direction
on a diagram. (Ignore fringing effects in the electric field for this calculation.)
iii. Compute the Poynting vector, draw its direction on a diagram, and verify that it agrees
with the rate of energy transfer.
c. A long solenoid of radius R has N turns of wire per unit length. The solenoid carries current
I, and this current is increased at a small, constant rate dI
dt .
i. What is the power per unit length P delivered to the solenoid?
ii. What is the electric field E just inside the surface of the solenoid? Draw its direction
on a diagram.
iii. Compute the Poynting vector, draw its direction on a diagram, and verify that it agrees
with the rate of energy transfer.

Solution

c
Copyright 2013
American Association of Physics Teachers

2013 Semifinal Exam

a.

Part B

14

i. A length l of the rod has charge q = R2 l; the force on it is F = qE and the power
delivered is P = F v. Combining these,
P = R2 lEv
P = R2 Ev
ii. The length l of the rod moves past a point in a time t = vl , so the current carried by the
rod is
q
I = = R2 v
t
Applying Amperes law to a loop of radius R,
I
B dl = 0 Ienc
2RB = 0 R2 v
1
B = 0 Rv
2
The field is circumferential as given by the right-hand rule.
iii. The electric and magnetic fields are perpendicular, so the Poynting vector has magnitude
S=

1
EB
0

1
S = RvE
2
A quick application of the right hand rule indicates that it points inward along the
surface of the cylinder, as it ought. The cylinder has area per unit length 2r, so the
rate of energy transfer per unit length is
P = 2rS = R2 vE
in agreement with the previous result.
b.

i. The capacitance is given by the standard parallel-plate capacitor formula:


C=

0 R2
d

The voltage on the capacitor is thus


V =

Q
Qd
=
C
0 R2

and the power is


P = IV
IQd
P =
0 R2
Students may choose instead to apply the formula for the volume energy density,
1
U = 0 E 2
2
c
Copyright 2013
American Association of Physics Teachers

2013 Semifinal Exam

Part B

15

ii. Consider an Amperian loop encircling the edge of the capacitor, and use a flat Gaussian
surface through the center of the capacitor. The electric field here is perpendicular to
the surface and has magnitude
E=

V
Q
=
d
0 R2

The electric flux through the surface is thus


E = R2 E =

Q
0

This can also be determined directly using Gausss law and appropriate symmetries.
There is no current through the surface, so from Amperes Law
I
dE
B dl = 0 0
dt
2RB = 0

dQ
dt

0 I
2R
The field is circumferential as given by the right-hand rule.
Note that we could instead use a curved Gaussian surface that avoids the center of the
capacitor and intersects one of the charging wires! In this case we have directly
I
B dl = 0 I
B=

and the calculation proceeds as before.


iii. The electric and magnetic fields are perpendicular, so again
S=

1
EB
0

IQ
20 2 R3
A quick application of the right hand rule indicates that it points inward along the edge
of the capacitor, as it ought. The area of this region is 2Rd, so the power delivered is
S=

P = 2RdS =

IQd
0 R2

in agreement with the previous result.


c.

i. Suppose that the solenoid has length l. The inductance is


L = 0 N 2 R2 l
Students may quote this formula directly, or derive it as follows. Consider an Amperian
loop of length d intersecting the solenoid. This loop encloses N d turns of wire, so from
Amperes law (remembering that the magnetic field exists entirely within the solenoid)
I
B dl = 0 Ienc

c
Copyright 2013
American Association of Physics Teachers

2013 Semifinal Exam

Part B

16

Bd = 0 N dI
B = 0 N I
There are N l loops, so the total flux is
= N lBR2
= 0 N 2 IR2 l
and since = LI,
L = 0 N 2 R2 l
as quoted above.
The voltage across the inductor is thus
V =L

dI
dt

V = 0 N 2 R2 l

dI
dt

and the power delivered is


P = IV
P = 0 N 2 R2 lI

dI
dt

P = 0 N 2 R2 I

dI
dt

or, dividing by l,

Students may choose instead to apply the formula for the volume energy density,
U=

1 2
B
20

ii. Consider an Amperian loop just inside the surface of the solenoid. From above, the
magnetic field through this loop is B = 0 N I; thus (working in magnitudes)
I
dB
E dl =
dt
2RE = 0 N R2

dI
dt

1
dI
E = 0 N R
2
dt
The field is circumferential as given by Lenzs law and the right-hand rule.
iii. The electric and magnetic fields are perpendicular, so again
S=

1
EB
0

1
dI
S = 0 N 2 RI
2
dt
c
Copyright 2013
American Association of Physics Teachers

2013 Semifinal Exam

Part B

17

A quick application of the right hand rule indicates that it points inward towards the
axis of the solenoid, as it ought. The area per unit length is just 2R, so the power per
unit length is
P = 2RS
dI
P = 0 N 2 R2 I
dt
in agreement with the previous result.

c
Copyright 2013
American Association of Physics Teachers

United States Physics Team


F = ma Contest Papers
2014

2014 F = ma Exam

AAPT
AIP

UNITED STATES PHYSICS TEAM


2014

2014 F = ma Contest
25 QUESTIONS - 75 MINUTES
INSTRUCTIONS
DO NOT OPEN THIS TEST UNTIL YOU ARE TOLD TO BEGIN
Use g = 10 N/kg throughout this contest.
You may write in this booklet of questions. However, you will not receive any credit for
anything written in this booklet.
Your answer to each question must be marked on the optical mark answer sheet.
Select the single answer that provides the best response to each question. Please be sure to
use a No. 2 pencil and completely fill the box corresponding to your choice. If you change
an answer, the previous mark must be completely erased.
Correct answers will be awarded one point; incorrect answers will result in a deduction of
point. There is no penalty for leaving an answer blank.

1
4

A hand-held calculator may be used. Its memory must be cleared of data and programs.
You may use only the basic functions found on a simple scientific calculator. Calculators
may not be shared. Cell phones may not be used during the exam or while the exam papers
are present. You may not use any tables, books, or collections of formulas.
This test contains 25 multiple choice questions. Your answer to each question must be marked
on the optical mark answer sheet that accompanies the test. Only the boxes preceded by
numbers 1 through 25 are to be used on the answer sheet.
All questions are equally weighted, but are not necessarily the same level of difficulty.
In order to maintain exam security, do not communicate any information about
the questions (or their answers or solutions) on this contest until after February
20, 2014.
The question booklet and answer sheet will be collected at the end of this exam. You may
not use scratch paper.

DO NOT OPEN THIS TEST UNTIL YOU ARE TOLD TO BEGIN


Contributors to this years exam include David Fallest, David Jones, Jiajia Dong, Paul Stanley, Warren Turner, Qiuzi Li, and former US Team members
Andrew Lin, Matthew Huang, Samuel Zbarsky.

c
Copyright 2014
American Association of Physics Teachers

2014 F = ma Exam

1. A car turning to the right is traveling at constant speed in a circle. From the drivers perspective,
the angular momentum vector about the center of the circle points X and the acceleration vector
of the car points Y where
(A) X is left, Y is left.
(B) X is forward, Y is right.
(C) X is down, Y is forward.
(D) X is left, Y is right.
(E) X is down, Y is right. CORRECT

Solution
Use right hand rule to find angular momentum, and acceleration is toward center in uniform
circular motion.

c
Copyright 2014
American Association of Physics Teachers

2014 F = ma Exam

2. A ball rolls without slipping down an inclined plane as shown in the diagram.

Which of the following vectors best represents the direction of the total force that the ball exerts
on the plane?
(A)

(B)

(C)

(D)

(E)

CORRECT answer is (E)

Solution
Normal force into plane. Friction must be less than the parallel force, or it wouldnt slide
down incline! Correct answer is into plane and down plane; note that if the ball is rolling
downward and accelerating, friction must be less than the parallel component of gravity.

c
Copyright 2014
American Association of Physics Teachers

2014 F = ma Exam

3. An object of uniform density floats partially submerged so that 20% of the object is above the
water. A 3 N force presses down on the top of the object so that the object becomes fully
submerged. What is the volume of the object? The density of water is H2 O = 1000 kg/m3 .
(A) Vobject = 0.3 L
(B) Vobject = 0.67 L
(C) Vobject = 1.2 L
(D) Vobject = 1.5 L CORRECT
(E) Vobject = 3.0 L

Solution
Submerging object requires pushing against the additional buoyant force of 0.2w V g, so
V = 1.5 L.

c
Copyright 2014
American Association of Physics Teachers

2014 F = ma Exam

4. What are the correct values of the numbers in the following statements? Assume there are no
external forces, and take N = 1 to mean that the statement cannot be made for any meaningful
number of particles.
If a particle at rest explodes into N1 or fewer particles with known masses, and the
total kinetic energy of the new particles is known, the kinetic energy of each of the new
particles is completely determined.
If a particle at rest explodes into N2 or fewer particles, the velocities of the new particles
must lie in a line.
If a particle at rest explodes into N3 or fewer particles, the velocities of the new particles
must lie in a plane.
(A) N1 = 2, N2 = 1, N3 = 1
(B) N1 = 1, N2 = 2, N3 = 3
(C) N1 = 2, N2 = 2, N3 = 3 CORRECT
(D) N1 = 3, N2 = 2, N3 = 3
(E) N1 = 2, N2 = 3, N3 = 4

Solution
Certainly N2 is at most 2 and N3 is at most 3; it is certainly possible for three particles to
emerge with non-collinear velocities, or four to emerge with non-planar ones. (Consider for
example the case where all of the particles have equal mass and they emerge at the corners
of a triangle or tetrahedron.) However, note that the total momentum of the daughter
particles must be zero; it is impossible for two non-collinear vectors to sum to zero, nor
three non-coplanar vectors. Thus N2 = 2 and N3 = 3.
Meanwhile, N1 is at most 2; consider three identical particles, where (among many other
possibilities) any one could emerge at rest with the energy split equally among the other
two. In the case of two particles, the velocities must be in opposite direction and (as argued)
in a straight line; thus, up to overall direction, they are entirely determined by the kinetic
energies. There are two independent equations governing the kinetic energies; the total
momentum must be zero, and the total kinetic energy is fixed. Thus the kinetic energies are
fully determined in this case, and N1 = 2.

c
Copyright 2014
American Association of Physics Teachers

2014 F = ma Exam

5. A unicyclist goes around a circular track of radius 30 m at a (amazingly fast!) constant speed of
10 m/s. At what angle to the left (or right) of vertical must the unicyclist lean to avoid falling?
Assume that the height of the unicyclist is much smaller than the radius of the track.
(A) 9.46
(B) 9.59
(C) 18.4 CORRECT
(D) 19.5
(E) 70.5

Solution
Force toward center is mv 2 /r. Force toward ground is mg. The force toward the center is
from friction and acts at the point in contact with the ground. The normal force is equal
Im magnitude to the force of gravity and acts upward at the point of contact. We are then
interested in the tangent angle as defined by
tan =

v2
gr

c
Copyright 2014
American Association of Physics Teachers

2014 F = ma Exam

6. A cubical box of mass 10 kg with edge length 5 m is free to move on a frictionless horizontal
surface. Inside is a small block of mass 2 kg, which moves without friction inside the box. At
time t = 0, the block is moving with velocity 5 m/s directly towards one of the faces of the box,
while the box is initially at rest. The coefficient of restitution for any collision between the block
and box is 90%, meaning that the relative speed between the box and block immediately after a
collision is 90% of the relative speed between the box and block immediately before the collision.
10 kg cube

2 kg block
frictionless surface
After 1 minute, the block is a displacement x from the original position. Which of the following
is closest to x?
(A) 0 m
(B) 50 m CORRECT
(C) 100 m
(D) 200 m
(E) 300 m

Solution
The easiest way to consider this is the motion of the center of mass of the system, which,
by conservation of momentum, must move with a speed of
vcm =

(2 kg)(5 m/s)
5
= m/s.
(2 kg) + (10 kg)
6

After one minute the center of mass has moved ( 65 m/s)(60 s) = 50 m.


Now the center of mass must be bounded by the cubical box, so the position of the inside
block must be somewhere near the 50 meter mark.

c
Copyright 2014
American Association of Physics Teachers

2014 F = ma Exam

7. A 1.00 m long stick with uniform density is allowed to rotate about a point 30.0 cm from its end.
The stick is perfectly balanced when a 50.0 g mass is placed on the stick 20.0 cm from the same
end. What is the mass of the stick?
(A) 35.7 g
(B) 33.3 g
(C) 25.0 g CORRECT
(D) 17.5 g
(E) 14.3 g
8. An object of mass M is hung on a vertical spring of spring constant k and is set into vertical
oscillations. The period of this oscillation is T0 . The spring is then cut in half and the same mass
is attached and the system is set up to oscillate on a frictionless inclined plane making an angle
to the horizontal. Determine the period of the oscillations on the inclined plane in terms of T0 .
(A) T0
(B) T0 /2
(C) 2T0 sin

(D) T0 / 2 CORRECT

(E) T0 sin / 2

Solution
The spring constant of a spring is inversely proportional to the length of the un-stretched
spring; cutting a spring in half will double the spring constant.
Placing the system on an incline will change the equilibrium position of the mass, but will
not affect the period of oscillation in any way.
As such, the new period of oscillation is given by
r
r
m
1
m
T0
T = 2
= 2
=
2k
k
2
2

c
Copyright 2014
American Association of Physics Teachers

2014 F = ma Exam

9. A 5.0 kg object undergoes a time-varying force as shown in the graph below. If the velocity at
t = 0.0 s is +1.0 m/s, what is the velocity of the object at t = 7 s?
4

Force (N)

3
2
1

4
5
time (s)

(A) 2.45 m/s


(B) 2.50 m/s
(C) 3.50 m/s CORRECT
(D) 12.5 m/s
(E) 15.0 m/s

Solution
The area under the graph is a measure of the change in momentum, so
1
1
p = (3 s)(3 N) + (4 s)(3 N + 1 N) = 12.5 N s.
2
2
The additional velocity is then v = 2.5 m/s.

c
Copyright 2014
American Association of Physics Teachers

2014 F = ma Exam

10

10. A radio controlled car is attached to a stake in the ground by a 3.00 m long piece of string, and
is forced to move in a circular path. The car has an initial angular velocity of 1.00 rad/s and
smoothly accelerates at a rate of 4.00 rad/s2 . The string will break if the centripetal acceleration
exceeds 2.43 102 m/s2 . How long can the car accelerate at this rate before the string breaks?
(A) 0.25 s
(B) 0.50 s
(C) 1.00 s
(D) 1.50 s
(E) 2.00 s CORRECT

Solution
The string breaks if ac exceeds the maximum tension, or
2

1 2
2
t + 0
ac = r = r
2
Solve for the time,
s r

2
ac
t=
0

r
or
t = 2s

c
Copyright 2014
American Association of Physics Teachers

2014 F = ma Exam

11

11. A point mass m is connected to an ideal spring on a horizontal frictionless surface. The mass is
pulled a short distance and then released.
Which of the following is the most correct plot of the kinetic energy as a function of potential
energy?
Ekinetic
Ekinetic

(A)

(B)

Epotential

Ekinetic

(C)

Epotential

Ekinetic

(D)

Epotential

Epotential

Ekinetic

(E)

Epotential

CORRECT

Solution
Energy is conserved, so the total must be constant. So it is the descending diagonal line of
slope magnitude one.

c
Copyright 2014
American Association of Physics Teachers

2014 F = ma Exam

12

The following information applies to questions 12 and 13


A paper helicopter with rotor radius r and weight W is dropped from a height h in air with a
density of .
r

Assuming that the helicopter quickly reaches terminal velocity, a function for the time of flight T
can be found in the form
T = kh r W .
where k is an unknown dimensionless constant (actually, 1.164). , , , and are constant
exponents to be determined.
12. Determine .
(A) = 1
(B) = 1/2
(C) = 0
(D) = 1/2
(E) = 1 CORRECT

Solution
If the helicopter is at terminal velocity, then it is falling at constant speed. As such, h = vT ,
where v is the terminal velocity. In that case, = 1.

c
Copyright 2014
American Association of Physics Teachers

2014 F = ma Exam

13

13. Determine .
(A) = 1/3
(B) = 1/2
(C) = 2/3
(D) = 1 CORRECT
(E) can not be uniquely determined without more information.

Solution
Dimensional analysis on mass requires
that = . SInce only W has units of time (inverse
p
squared), then = 1/2. But /W has units of length2 time1 , and we know = 1,
then = 1.

14. A disk of moment of inertia I, mass M , and radius R has a cord wrapped around it tightly as
shown in the diagram. The disk is free to slide on its side as shown in the top down view. A
constant force of T is applied to the end of the cord and accelerates the disk along a frictionless
surface.
rope
Force T

Solid disk

After the disk has accelerated some distance, determine the ratio of the translational KE to total
KE of the disk,
KEtranslational /KEtotal =
(A)

I
M R2

(B)
(D)

M R2
I
I
3M R2
I
M R2 +I

(E)

M R2
M R2 +I

(C)

CORRECT

Solution
c
Copyright 2014
American Association of Physics Teachers

2014 F = ma Exam

14

The applied force T accelerates the center of mass of the disk at a rate a = T /M . The
applied force produces a torque on the disk about the center of mass given by = RT , and
therefore an angular acceleration of = RT /I.
After a time t the velocity of the disk will be v = at and the angular velocity will be = t.
Then
KEtranslational

1
= M
2

T
M

and
KErotational

1
= I
2

RT
I

2

2

t2 ,

t2 ,

so the desired ratio is then


KEtranslational
=
KEtotal
=
=


1
T 2 2
M M
t
2

 ,
2
T
2 + 1 I RT 2 t2
t
M
2
I

1
M
2
1
M
2
1
+ RI
M

I
.
I + M R2

15. The maximum torque output from the engine of a new experimental car of mass m is . The
maximum rotational speed of the engine is . The engine is designed to provide a constant power
output P . The engine is connected to the wheels via a perfect transmission that can smoothly
trade torque for speed with no power loss. The wheels have a radius R, and the coefficient of
static friction between the wheels and the road is .
What is the maximum sustained speed v the car can drive up a 30 degree incline? Assume no
frictional losses and assume is large enough so that the tires do not slip.
(A) v = 2P/(mg) CORRECT

(B) v = 2P/( 3mg)


(C) v = 2P/(mg)
(D) v = /(mg)
(E) v = /(mg)

Solution
The fundamental idea is P = F v where F is the component of the weight parallel to the
incline. Then
v = P/mg sin
Since = 30 , the answer is v = 2P/mg
c
Copyright 2014
American Association of Physics Teachers

2014 F = ma Exam

15

16. An object of mass m1 initially moving at speed v0 collides with an object of mass m2 = m1 , where
< 1, that is initially at rest. The collision could be completely elastic, completely inelastic, or
partially inelastic. After the collision the two objects move at speeds v1 and v2 . Assume that the
collision is one dimensional, and that object one cannot pass through object two.
After the collision, the speed ratio r1 = v1 /v0 of object 1 is bounded by
(A) (1 )/(1 + ) r1 1
(B) (1 )/(1 + ) r1 1/(1 + ) CORRECT
(C) /(1 + ) r1 1
(D) 0 r1 2/(1 + )
(E) 1/(1 + ) r1 2/(1 + )

Solution
Conserving momentum and kinetic energy in the completely elastic collision yields the following quantities.
1
v1
=
,
r1 =
v0
(1 + )
r2 =

2
v2
=
,
v0
(1 + )

Since < 1, object 1 is tricking a less massive object, and therefore continues to move
forward.
Conserving momentum in a completely inelastic collision yields
r1 = r2 =

1
,
(1 + )

Note that object 2 will always be moving forward, and since object 1 cant pass through it,
object 2 must always move with a more positive (or equal) velocity than object 1. Consequently,
1/(1 + ) r2 2/(1 + )
Sorting out object 1 is a little harder, since under certain circumstances it can bounce
backward. But in this case, since < 1, it retains a forward velocity after the collision.
Then
(1 )/(1 + ) r1 1/(1 + )

c
Copyright 2014
American Association of Physics Teachers

2014 F = ma Exam

16

17. A spherical cloud of dust in space has a uniform density 0 and a radius R0 . The gravitational
acceleration of free fall at the surface of the cloud due to the mass of the cloud is g0 .
A process occurs (heat expansion) that causes the cloud to suddenly grow to a radius 2R0 , while
maintaining a uniform (but not constant) density. The gravitational acceleration of free fall at a
point R0 away from the center of the cloud due to the mass of the cloud is now
(A) g0 /32
(B) g0 /16
(C) g0 /8 CORRECT
(D) g0 /4
(E) g0 /2

Solution
Newtons Law of gravitation for a spherical object depends only on matter located closer to
the center than the point of reference.
g=

4
GM
=
Gr,
2
r
3

where the last equality is true for uniform density.


Doubling the size cuts the density to 1/8.

c
Copyright 2014
American Association of Physics Teachers

2014 F = ma Exam

17

18. Consider the following diagram of a box and two weight scales. Scale A supports the box via a
massless rope. A pulley is attached to the top of the box; a second massless rope passes over the
pulley, one end is attached to the box and the other end to scale B. The two scales read indicate
the tensions TA and TB in the ropes. Originally scale A reads 30 Newtons and scale B reads 20
Newtons.
TA

Scale A

Pulley

Box Frame

Scale B

TB
If an additional force pulls down on scale B so that the reading increases to 30 Newtons, what will
be the new reading on scale A?
(A) 35 Newtons
(B) 40 Newtons CORRECT
(C) 45 Newtons
(D) 50 Newtons
(E) 60 Newtons
Adapted from a demonstration by Richard Berg.
c
Copyright 2014
American Association of Physics Teachers

2014 F = ma Exam

18

Solution
As cute as the box is, the only thing that matter is that you pull down on scale B with 10
extra newtons, so scale A must increase by 10 to balance.
19. A helicopter is flying horizontally at constant speed. A perfectly flexible uniform cable is suspended
beneath the helicopter; air friction on the cable is not negligible.
Which of the following diagrams best shows the shape of the cable as the helicopter flies through
the air to the right?

(A)

(B)
CORRECT

(C)

(D)

(E)

Solution
Since there is air friction on the cable, then there must be a horizontal component to the
force where the cable attaches to the helicopter. Since the amount of air friction would be
proportional to the length of the cable hanging beneath at any point on the cable, then the
cable would be hanging in a straight diagonal line.
This question generated a great deal of controversy. At least two test takers challenged the
answer, one who even tried to do the experiment. Im told that several different Ph.D
physicists declared that the correct answer was X, but, interestingly enough, couldnt agree
on what X should be.
Originally the problem was worded as a cable with a hanging mass. As such, the correct
answer would have been D, as explained below.
c
Copyright 2014
American Association of Physics Teachers

2014 F = ma Exam

19

Since air friction on the hanging object is negligible, the only forces to consider are gravity
and the cable. So the cable must be pulling directly upward. Since there is air friction on
the cable, then there must be a horizontal component to the force where the cable attaches
to the helicopter. The correct answer would then be curving to the left then falling straight
down.
20. A crew of scientists has built a new space station. The space station is shaped like a wheel of
radius R, with essentially all its mass M at the rim. When the crew arrives, the station will be set
rotating at a rate that causes an object at the rim to have radial acceleration g, thereby simulating
Earths surface gravity. This is accomplished by two small rockets, each with thrust T newtons,
mounted on the stations rim. How long a time t does one need to fire the rockets to achieve the
desired condition?
p
(A) t = gR3 M/(2T )

(B) t = gR M/(2T ) CORRECT

(C) t = gR M/T
p
(D) t = gR/ M/T

(E) t = gR M/(T )
Adapted from a problem in Physics for Scientists and Engineers by Richard Wolfson

Solution
p
Desired acceleration: g = 2 R = g/R
The two rockets provide: 2T R = M R2 = 2T /M R
g 1/2 R1/2 M
time needed: t = / =
2T

c
Copyright 2014
American Association of Physics Teachers

2014 F = ma Exam

20

21. Two pulleys (shown in figure) are made of the same metal with density . Pulley A is a uniform
disk with radius R. Pulley B is identical except a circle of R/2 is removed from the center. When
two boxes M = m ( > 1) are connected over the pulleys through a massless rope and move
without slipping, what is the ratio between the accelerations in system A and B? The mass of
pulley A is M + m.
Pulley B

Pulley A

m
M

(A) aA /aB = 47/48 CORRECT


(B) aA /aB = 31/32
(C) aA /aB = 15/16
(D) aA /aB = 9/16
(E) aA /aB = 3/4

Solution
This is effectively a force balance problem with a generalized Atwood machine.
For mass M , the equations are
M a = M g TM
where TM is tension in rope on the M side.
For mass m, the equations are
ma = Tm mg
For the pulley, the equations are
I = R(TM tm )
with = a/R and I the moment of inertia.
Combining,
Ia/R2 = (M g M a) (ma + mg)
or
a=g

M m
I/R2 + M + m

Let I = (M + m)R2 for the two pulleys. Then the acceleration ratio is
B + 1
aA
=
aB
A + 1
c
Copyright 2014
American Association of Physics Teachers

2014 F = ma Exam

21

A is easy enough, it is just 1/2, since it is a uniform disk.


B requires a little more work, it is 1/2(1 (1/4)(1/4)) = 15/32 to account for the removal
of the interior disk. The ratio of accelerations is then
47
aA
=
aB
48

22. A body of mass M and a body of mass m  M are in circular orbits about their center of mass
under the influence of their mutual gravitational attraction to each other. The distance between
the bodies is R, which is much larger than the size of either body.
A small amount of matter m  m is removed from the body of mass m and transferred to the
body of mass M . The transfer is done in such a way so that the orbits of the two bodies remain
circular, and remain separated by a distance R. Which of the following statements is correct?
(A) The gravitational force between the two bodies increases.
(B) The gravitational force between the two bodies remains constant.
(C) The total angular momentum of the system increases.
(D) The total angular momentum of the system remains constant.
(E) The period of the orbit of two bodies remains constant. CORRECT

Solution
The force between two bodies is given by
F =G

Mm
R2

The new force, F 0 , would be given by


F0 = G

(M + m)(m m)
M m (M m)m (m)2
=
G
r2
R2

This can be approximated to first order as




m
0
F = 1
F
m
For centripetal motion,
mv 2 /r = F
where r is the distance from m to the center of mass, which is
r=

M
R
M +m

c
Copyright 2014
American Association of Physics Teachers

2014 F = ma Exam

22

In terms of angular momenta,


Lm = mvr =
or

s
Lm =

m(mv 2 /r)r3 =

mM 3 RGmM
= mM 2
(m + M )3

mr3 F

GR
(m + M )3

By symmetry,
s
LM = m2 M

GR
(m + M )3

and the sum is then

GR
m+M
and we already saw that the quantity M m decreases as m is transferred from m to M .
L = Lm + LM = mM

Finally, the period of motion can be found from v = 2r/T , so


r
2r
mr
T =
= 2
v
F
Combining the above,
r
T = 2

mM R R2
= 2
m + M GmM

R3
G(m + M )

And thats constant.


The following information applies to questions 23 and 24
A 100 kg astronaut carries a launcher loaded with a 10 kg bowling ball; the launcher and the
astronauts spacesuit have negligible mass. The astronaut discovers that firing the launcher results
in the ball moving away from her at a relative speed of 50 m/s.
23. What is the impulse delivered to the astronaut when firing the launcher?
(A) 455 N s CORRECT
(B) 500 N s
(C) 550 N s
(D) 5000 N s
(E) 5500 N s

Solution
Work in the center of mass frame, where the impulse delivered is simply the final momentum
c
Copyright 2014
American Association of Physics Teachers

2014 F = ma Exam

23

of the astronaut. Let the astronaut have mass m1 and final velocity v1 , and let the launcher
have mass m2 and final velocity v2 . Then from conservation of momentum
m1 v1 + m2 v2 = 0
and the relative velocity is (taking v1 > 0)
vr = v1 v2
Substituting,
m1 v1 + m2 (v1 vr ) = 0
m1 m2
m1 v1 =
vr
m1 + m2
so
J = m1 v1 = 455 N s

24. The astronaut in the previous situation is now moving at 10 m/s (as measured in a certain
frame of reference). She wishes to fire the launcher so that her velocity turns through as
large an angle as possible (in this frame of reference). What is this maximum angle? (Hint:
a diagram may be useful.)
(A) 24.4
(B) 26.6
(C) 27.0 CORRECT
(D) 30.0
(E) 180.0

Solution
The final momentum of the astronaut is the sum of her initial momentum and the impulse
delivered to her. Her initial momentum has magnitude
pi = (100 kg)(10 m/s) = 1000 kg m/s
and the impulse available has the fixed, smaller magnitude found in the previous problem.
(Note that the impulse is the same in all inertial reference frames.) Her final momentum is
thus confined to the circle shown below:

c
Copyright 2014
American Association of Physics Teachers

2014 F = ma Exam

24

p~i

J~

p~f

The maximum angle is obtained by the tangent to the circle.

p~i

p~f

J~

The maximum angle is thus


= arcsin

J
= 27.0
pi

25. A block with mass m is released from rest at the top of a frictionless ramp. The block starts at
a height h1 above the base of the ramp, slides down the ramp, and then up a second ramp. The
coefficient of kinetic friction between the block and the second ramp is k . If both ramps make
an angle of with the horizontal, to what height h2 above the base of the second ramp will the
block rise?
(A) h2 = (h1 sin )/(k cos + sin ) CORRECT
(B) h2 = (h1 sin )/(k + sin )
(C) h2 = (h1 sin )/(k cos2 + sin )
(D) h2 = (h1 sin )/(k cos2 + sin2 )
(E) h2 = (h1 cos )/(k sin + cos )

c
Copyright 2014
American Association of Physics Teachers

United States Physics Team


Semi Final Contest Papers
2014

2014 USA Physics Olympiad Exam

AAPT
AIP

UNITED STATES PHYSICS TEAM


2014
USA Physics Olympiad Exam

DO NOT DISTRIBUTE THIS PAGE


Important Instructions for the Exam Supervisor
This examination consists of two parts.
Part A has four questions and is allowed 90 minutes.
Part B has two questions and is allowed 90 minutes.
The first page that follows is a cover sheet. Examinees may keep the cover sheet for both
parts of the exam.
The parts are then identified by the center header on each page. Examinees are only allowed
to do one part at a time, and may not work on other parts, even if they have time remaining.
Allow 90 minutes to complete Part A. Do not let students look at Part B. Collect the answers
to Part A before allowing the examinee to begin Part B. Examinees are allowed a 10 to 15
minutes break between parts A and B.
Allow 90 minutes to complete Part B. Do not let students go back to Part A.
Ideally the test supervisor will divide the question paper into 4 parts: the cover sheet (page 2),
Part A (pages 3-16), Part B (pages 17-23), and several answer sheets for two of the questions
in part A (pages 25-28). Examinees should be provided parts A and B individually, although
they may keep the cover sheet. The answer sheets should be printed single sided!
The supervisor must collect all examination questions, including the cover sheet, at the end
of the exam, as well as any scratch paper used by the examinees. Examinees may not take
the exam questions. The examination questions may be returned to the students after April
15, 2014.
Examinees are allowed calculators, but they may not use symbolic math, programming, or
graphic features of these calculators. Calculators may not be shared and their memory must
be cleared of data and programs. Cell phones, PDAs or cameras may not be used during
the exam or while the exam papers are present. Examinees may not use any tables, books,
or collections of formulas.

c
Copyright 2014
American Association of Physics Teachers

2014 USA Physics Olympiad Exam

AAPT
AIP

Cover Sheet

UNITED STATES PHYSICS TEAM


2014

USA Physics Olympiad Exam


INSTRUCTIONS
DO NOT OPEN THIS TEST UNTIL YOU ARE TOLD TO BEGIN
Work Part A first. You have 90 minutes to complete all four problems. Each question is
worth 25 points. Do not look at Part B during this time.
After you have completed Part A you may take a break.
Then work Part B. You have 90 minutes to complete both problems. Each question is worth
50 points. Do not look at Part A during this time.
Show all your work. Partial credit will be given. Do not write on the back of any page. Do
not write anything that you wish graded on the question sheets.
Start each question on a new sheet of paper. Put your AAPT ID number, your name, the
question number and the page number/total pages for this problem, in the upper right hand
corner of each page. For example,
AAPT ID #
Doe, Jamie
A1 - 1/3
A hand-held calculator may be used. Its memory must be cleared of data and programs. You
may use only the basic functions found on a simple scientific calculator. Calculators may not
be shared. Cell phones, PDAs or cameras may not be used during the exam or while the
exam papers are present. You may not use any tables, books, or collections of formulas.
Questions with the same point value are not necessarily of the same difficulty.
In order to maintain exam security, do not communicate any information about
the questions (or their answers/solutions) on this contest until after April 15,
2014.
Possibly Useful Information. You may
g = 9.8 N/kg
k = 1/40 = 8.99 109 N m2 /C2
c = 3.00 108 m/s
NA = 6.02 1023 (mol)1
= 5.67 108 J/(s m2 K4 )
1eV = 1.602 1019 J
me = 9.109 1031 kg = 0.511 MeV/c2
sin 16 3 for ||  1

use this sheet for both parts of the exam.


G = 6.67 1011 N m2 /kg2
km = 0 /4 = 107 T m/A
kB = 1.38 1023 J/K
R = NA kB = 8.31 J/(mol K)
e = 1.602 1019 C
h = 6.63 1034 J s = 4.14 1015 eV s
(1 + x)n 1 + nx for |x|  1
cos 1 21 2 for ||  1

c
Copyright 2014
American Association of Physics Teachers

2014 USA Physics Olympiad Exam

Part A

Part A
Question A1
Inspired by: http://www.wired.com/wiredscience/2012/04/a-leaning-motorcycle-on-a-vertical-wall/
A unicyclist of total height h goes around a circular track of radius R while leaning inward at
an angle to the vertical. The acceleration due to gravity is g.
a. Suppose h  R. What angular velocity must the unicyclist sustain?

Solution
Work in the rotating frame, where four forces act on the unicyclist: a normal and frictional
force at the point of contact, gravity downwards at the center of mass, and a (fictitious)
centrifugal force.
If h  R, all parts of the unicyclist are at a distance of approximately R from the center of the
circle, so the centripetal acceleration of every part of the unicyclist is 2 R. The centrifugal
force can then be taken to act at the center of mass for purposes of computing the torque.
If the center of mass is a distance l from the point of contact, the torque about the point of
contact is
= m 2 Rl cos mgl sin
Since the unicyclist is stationary in this frame, = 0:
m 2 Rl cos mgl sin = 0
r
g
tan
=
R

b. Now model the unicyclist as a uniform rod of length h, where h is less than R but not
negligible. This refined model introduces a correction to the previous result. What is the new
expression for the angular velocity ? Assume that the rod remains in the plane formed by
the vertical and radial directions, and that R is measured from the center of the circle to the
point of contact at the ground.

Solution
The centripetal acceleration now varies meaningfully along the length of the unicyclist. In
the rotating frame, the torque about the point of contact is given by
Z
c = 2 rz dm
where r is the distance from the center of the circle, z is the height above the ground, and
dm is a mass element. Because the mass of the unicyclist is uniformly distributed along a
length h, the mass element dm can be written as m
h ds for a length element ds, and we have
Z h
m
c =
2 (R s sin )(s cos )
ds
h
0
c
Copyright 2014
American Association of Physics Teachers

2014 USA Physics Olympiad Exam

Part A

c = m h cos

R h
sin
2
3

Gravity continues to act at the center of mass, a distance h2 from the point of contact, and
in the opposite direction:
h
g = mg sin
2
Again, the total torque is zero, so


R h
h
2
m h cos
sin mg sin = 0
2
3
2
s
1
g

2h
1
tan
sin
=
R
3R

Question A2
A room air conditioner is modeled as a heat engine run in reverse: an amount of heat QL is
absorbed from the room at a temperature TL into cooling coils containing a working gas; this gas is
compressed adiabatically to a temperature TH ; the gas is compressed isothermally in a coil outside
the house, giving off an amount of heat QH ; the gas expands adiabatically back to a temperature
TL ; and the cycle repeats. An amount of energy W is input into the system every cycle through
an electric pump. This model describes the air conditioner with the best possible efficiency.

heating coil

pump

room

valve

cooling coil

Assume that the outside air temperature is TH and the inside air temperature is TL . The
air-conditioner unit consumes electric power P . Assume that the air is sufficiently dry so that no
condensation of water occurs in the cooling coils of the air conditioner. Water boils at 373 K and
freezes at 273 K at normal atmospheric pressure.
a. Derive an expression for the maximum rate at which heat is removed from the room in terms
of the air temperatures TH , TL , and the power consumed by the air conditioner P . Your
derivation must refer to the entropy changes that occur in a Carnot cycle in order to receive
full marks for this part.

c
Copyright 2014
American Association of Physics Teachers

2014 USA Physics Olympiad Exam

Part A

Solution
From Carnot cycles, and by entropy conservation, we have
QH
TH
=
QL
TL
Also, by energy conservation,
QH = QL + W
So heat is removed at a rate QL /t. But
QL = QH W = QL
or


W = QL

TH
W
TL


TH
1
TL

Rearrange and divide by time,


QL
=P
t

TL
TH TL

b. The room is insulated, but heat still passes into the room at a rate R = kT , where T is
the temperature difference between the inside and the outside of the room and k is a constant.
Find the coldest possible temperature of the room in terms of TH , k, and P .

Solution
Equate.
kT = P

TL
TH T
=P
T
T

or
k(T )2 = P TH P T,
which is a quadratic that can be solved as
T =

p
P 2 + 4P kTH
,
2k

but only the positive root has physical significance. Writing x = P/k,

x p
T =
1 + 4TH /x 1
2
Thats the amount the room is colder than the outside, so

x p
TL = TH
1 + 4TH /x 1
2

c
Copyright 2014
American Association of Physics Teachers

2014 USA Physics Olympiad Exam

Part A

c. A typical room has a value of k = 173 W/ C. If the outside temperature is 40 C, what


minimum power should the air conditioner have to get the inside temperature down to 25 C?

Solution
Dont forget to convert to Kelvin!
From above,
P =

k(T )2
,
TL

so
P = 130 W

c
Copyright 2014
American Association of Physics Teachers

2014 USA Physics Olympiad Exam

Part A

Question A3
When studying problems in special relativity it is often the invariant distance s between two
events that is most important, where s is defined by


(s)2 = (ct)2 (x)2 + (y)2 + (z)2
where c = 3 108 m/s is the speed of light.1
a. Consider the motion of a projectile launched with initial speed v0 at angle of 0 above the
horizontal. Assume that g, the acceleration of free fall, is constant for the motion of the
projectile.
i. Derive an expression for the invariant distance of the projectile as a function of time t as
measured from the launch, assuming that it is launched at t = 0. Express your answer
as a function of any or all of 0 , v0 , c, g, and t.
ii. The radius of curvature of an objects trajectory can be estimated by assuming that
the trajectory is part of a circle, determining the distance between the end points, and
measuring the maximum height above the straight line that connects the endpoints. Assuming that we mean invariant distance as defined above, find the radius of curvature
of the projectiles trajectory as a function of any or all of 0 , v0 , c, and g. Assume that
the projectile lands at the same level from which it was launched, and assume that the
motion is not relativistic, so v0  c, and you can neglect terms with v/c compared to
terms without.

Solution
The particle begins at ct = x = y = 0 and takes a path satisfying
x = v0 t cos 0
1
z = v0 t sin 0 gt2
2
Thus

1
s2 = (ct)2 (v0 t cos 0 )2 (v0 t sin 0 gt2 )2
2
which can be simplified to
1
1
s2 = (c2 v02 )t2 + gv0 sin 0 t3 + g 2 t4
2
4

b. The particle reaches the ground again at


tf = 2

v0 sin
g

xf = v0 cos tf
zf = 0
1

We are using the convention used by Einstein

c
Copyright 2014
American Association of Physics Teachers

2014 USA Physics Olympiad Exam

Part A

and so the invariant distance between the endpoints is


s2 = (ctf )2 + (v0 cos tf )2
s 2c

v0 sin
g

The maximum height above the ground is


zmax =

(v0 sin )2
2g

Because zmax  s, we have from similar triangles


1
s
zmax
2
1
R
2s

1 s2
4 zmax

R2

c2
g

c. A rocket ship far from any gravitational mass is accelerating in the positive x direction at a
constant rate g, as measured by someone inside the ship. Spaceman Fred at the right end
of the rocket aims a laser pointer toward an alien at the left end of the rocket. The two are
separated by a distance d such that dg  c2 ; you can safely ignore terms of the form (dg/c2 )2 .
i. Sketch a graph of the motion of both Fred and the alien on the space-time diagram
provided in the answer sheet. The graph is not meant to be drawn to scale. Note that t
and x are reversed from a traditional graph. Assume that the rocket has velocity v = 0
at time t = 0 and is located at position x = 0. Clearly indicate any asymptotes, and the
slopes of these asymptotes.

Solution
Since the rocket-ship can never exceed the speed of light, yet it is always accelerating
(in the local frame), it must approach an asymptote that has a slope of one on the
space-time diagram shown. There is a slight challenge to consider, however. Since the
rocket ship is an extended object, do the two ends (represented by Fred and the Alien)
approach the same asymptote, or two different asymptotes?
At this point we must remember a consequence of special relativity for a ship moving
at relativistic speeds: the ship will contract in length as measured in the original frame.
As the speed of the ship approaches that of light, the length of the ship will approach
zero. The only way for that to happen is for the two ends of the ship to have slightly
different accelerations.

c
Copyright 2014
American Association of Physics Teachers

2014 USA Physics Olympiad Exam

Part A

If you had assumed (somewhat incorrectly) that the two ends of the ship have the same
acceleration, then the two trajectories would be approaching two different asymptotes
separated by a constant horizontal distance. But this would mean the apparent length
of the ship was constant, regardless of speed. In the instantaneous rest frame of the ship
we then require that Fred and the Alien be moving apart. This means that the ship
must be stretching, and eventually breaking.

c
Copyright 2014
American Association of Physics Teachers

2014 USA Physics Olympiad Exam

Part A

10

ii. If the frequency of the laser pointer as measured by Fred is f1 , determine the frequency
of the laser pointer as observed by the alien. It is reasonable to assume that f1  c/d.

Solution
If the spaceship is uniformly accelerating then we can choose a reference frame which is
instantaneously at rest with respect to the spaceship at t = 0.
Consider two instantaneous flashes from the astronaut. Flash 1 is emitted by Fred at
t = 0, flash 2 is emitted by the Fred at t = 1 .

c
Copyright 2014
American Association of Physics Teachers

2014 USA Physics Olympiad Exam

Part A

11

Flash 1 travels down towards the alien, who is accelerating upward. Let t1 be the time
at which the alien sees flash 1. Equate the distances,
1
ct1 = d gt1 2 .
2
Flash 2 is emitted at 1 . Flash 2 travels down towards the alien, who is still accelerating
upward. Let t2 be the time at which the alien sees flash 2. Equate the distances,
1
1
c(t2 1 ) = g1 2 + d gt2 2.
2
2
Notice that the pulse travel for a time t2 1 ! Defining t2 =1 +2 , and then expanding
to keep terms linear in  t1 ,
1
c(t1 + 2 1 ) = h gt1 2 gt1 2 .
2
Combining,
c(2 1 ) = gt1 2 .
but t1 d/c, so


gh
2 = 1 1 2
c
In terms of frequency this is

f2 = f1

gd
1+ 2
c

Alternatively, we can follow the motion of two wave crests from Fred to the alien. We
can work in the reference frame where Fred is stationary when the first crest is emitted.
Because f1  c/d, we can assume that Fred remains stationary during the period between the first and second crests; then, because the first crest moves towards the alien
at a speed c, they are separated by a distance c/f1 .
Because dg  c2 , the time taken for the crests to reach the alien is due almost entirely
to the motion of the crests, and is d/c. In this time, the spaceship accelerates to a speed
gd/c, and (in Freds frame of reference) the relative speed of the crests and the alien is
c + gd
c . The time between crests reaching the alien is thus
1
c/f1
=
f2
c + gd
c


gd
f2 = f1 1 + 2
c
While this time interval is measured in Freds reference frame, time dilation effects are
 2
and can thus be ignored.
of the order gd
c2

c
Copyright 2014
American Association of Physics Teachers

2014 USA Physics Olympiad Exam

Part A

12

Question A4
A positive point charge q is located inside a neutral hollow spherical conducting shell. The shell
has inner radius a and outer radius b; b a is not negligible. The shell is centered on the origin.
y

b
a
x

a. Assume that the point charge q is located at the origin in the very center of the shell.
i. Determine the magnitude of the electric field outside the conducting shell at x = b.

Solution
Conducting shell is neutral, so there is equal but opposite charge on surface r = a and
r = b. The electric field inside of a static conductor is zero, so the charge on inner surface
is equal but opposite to q by Gausss Law. Spherical symmetry requires a spherically
symmetric electric field, so by Gausss law, outside the shell, we have
E(r > b) =

q
40 r2

and then, at x = r = b, we have


E(b) =

q
40 b2

ii. Sketch a graph for the magnitude of the electric field along the x axis on the answer
sheet provided.

Solution

c
Copyright 2014
American Association of Physics Teachers

2014 USA Physics Olympiad Exam

Part A

13

iii. Determine the electric potential at x = a.

Solution
The shell is a conductor, so it is an equipotential surface. This means potential at r = a
is same as r = b. For points outside the shell, spherical symmetry (and Gausss Law)
makes the problem reducible to a point charge at the origin, so
V (r > b) =

q
40 r

But V (a) = V (b), so V (x = a) is


V (a) =

q
40 b

iv. Sketch a graph for the electric potential along the x axis on the answer sheet provided.

Solution

b. Assume that the point charge q is now located on the x axis at a point x = 2a/3.
c
Copyright 2014
American Association of Physics Teachers

2014 USA Physics Olympiad Exam

Part A

14

i. Determine the magnitude of the electric field outside the conducting shell at x = b.

Solution
The problem (for r > a) maintains the spherical symmetry of above, so the answer is
unchanged:
q
E(b) =
40 b2
ii. Sketch a graph for the magnitude of the electric field along the x axis on the answer
sheet provided.

Solution

iii. Determine the electric potential at x = a.

Solution
The problem (for r > a) maintains the spherical symmetry of above, so the answer is
unchanged:
q
V (a) =
40 b
iv. Sketch a graph for the electric potential along the x axis on the answer sheet provided.

Solution

c
Copyright 2014
American Association of Physics Teachers

2014 USA Physics Olympiad Exam

Part A

15

v. Sketch a figure showing the electric field lines (if any) inside, within, and outside the
conducting shell on the answer sheet provided. You should show at least eight field lines
in any distinct region that has a non-zero field.

Solution

c
Copyright 2014
American Association of Physics Teachers

2014 USA Physics Olympiad Exam

Part A

16

STOP: Do Not Continue to Part B

If there is still time remaining for Part A, you should review your work for
Part A, but do not continue to Part B until instructed by your exam
supervisor.

c
Copyright 2014
American Association of Physics Teachers

2014 USA Physics Olympiad Exam

Part B

17

Part B
Question B1
A block of mass M has a hole drilled through it so that a ball of mass m can enter horizontally
and then pass through the block and exit vertically upward. The ball and block are located on a
frictionless surface; the block is originally at rest.

M
m

frictionless horizontal surface


a. Consider the scenario where the ball is traveling horizontally with a speed v0 . The ball enters
the block and is ejected out the top of the block. Assume there are no frictional losses as the
ball passes through the block, and the ball rises to a height much higher than the dimensions
of the block. The ball then returns to the level of the block, where it enters the top hole
and then is ejected from the side hole. Determine the time t for the ball to return to the
position where the original collision occurs in terms of the mass ratio = M/m, speed v0 ,
and acceleration of free fall g.

Solution
The ball will be ejected vertically relative to the block, so the collision is effectively inelastic.
This means the horizontal velocity v1 of the block and ball after the collision will be given by
v1 =

m
v0
m+M

The ball will now have a vertical component to the velocity, v2 Since there are no frictional
losses, we can use conservation of energy to determine this vertical velocity. Prior to the
collision, the kinetic energy is
1
E0 = mv0 2
2
After the collision, the block carries away a kinetic energy of
1
E1 = M v1 2
2
so the ball must have kinetic energy

1
E2 = E0 E1 = m v1 2 + v2 2
2

c
Copyright 2014
American Association of Physics Teachers

2014 USA Physics Olympiad Exam

Part B

18

Equating,
mv0 2 M v1 2

2
m
2
mv0 (M + m)
v0 2
m+M


m
1
v0 2
m+M
r
M
v0
m+M

= m v1 2 + v2 2

= mv2 2
= v2 2
= v2

The time spent by the ball in the air is given by


t2 = 2v2 /g
The distance traveled horizontally by the ball in the air is given by
x = v1 t2 = 2v1 v2 /g
or
m
x=2
v0
m+M

M
1
2v0 2
v0 =
m+M g
g

m2 M
(m + M )3

When the ball returns to the block it then is projected horizontally back toward the starting
point (but it is now a distance x further away. It moves with a velocity v3 given by the
equation for an elastic collision
mM
v3 = v0
m+M
The time for the ball to return to where first collision occurs is then given by
t3 =
or
2v0
t3 =
g
or
2v0
t3 =
g

x
|v3 |

m2 M m + M
(m + M )3 M m

m2 M
(M + m)(M m)2

The total time is the sum of t2 and t3 , or


s
!
r
2v0
M
m2 M
+
g
m+M
(M + m)(M m)2
which can be simplified to give
2v0
g

M
m+M

s
1+

m2
(M m)2

c
Copyright 2014
American Association of Physics Teachers

2014 USA Physics Olympiad Exam

Part B

or
2v0
g

19

M
m+M

M
M m

Writing it in terms of = M/m,


2v0
g

1+

b. Now consider friction. The ball has moment of inertia I = 25 mr2 and is originally not rotating.
When it enters the hole in the block it rubs against one surface so that when it is ejected
upwards the ball is rolling without slipping. To what height does the ball rise above the
block?

Solution
Some of the previous work still holds true.
The ball will be ejected vertically relative to the block, so the collision is effectively inelastic.
This means the horizontal velocity v1 of the block and ball after the collision will be given by
m
v1 =
v0
m+M
The vertical velocity v4 of the ball, however, is now changed from v2 . Friction slows the ball
down with an impulse given by
p = f t
all the while causing the ball to rotate with a new angular momentum given by
L = T = rf t
but L = I = Iv4 /r and m(v2 v4 ) = p, so
Iv4 /r = mr(v2 v4 )
or, writing I = mr2 ,

v2
1+
The vertical velocity of the ball will take it to a height
v4 =

h=

v4 2
2g

so

1
v0 2
M
2g (1 + )2 m + M
Thankfully, the problem reduces to what we expect if M  m and = 0.
h=

Writing it in terms of = M/m and = 2/5,


h=

v0 2 25
2g 49 1 +

c
Copyright 2014
American Association of Physics Teachers

2014 USA Physics Olympiad Exam

Part B

20

Question B2
In parts a and b of this problem assume that velocities v are much less than the speed of light c,
and therefore ignore relativistic contraction of lengths or time dilation.
a. An infinite uniform sheet has a surface charge density and has an infinitesimal thickness.
The sheet lies in the xy plane.
~ (magnitude and direction)
i. Assuming the sheet is at rest, determine the electric field E
above and below the sheet.

Solution
From symmetry, the fields above and below the sheet are equal in magnitude and directed
away from the sheet. From Gausss Law, using a cylinder of end area A,
2EA =

A
0

E = /20
pointing directly away from the sheet in the z direction, or
~ =
z
E
20
above the sheet and

~ =
E
z
20

below the sheet


ii. Assuming the sheet is moving with velocity ~v = v
x (parallel to the sheet), determine
~ (magnitude and direction) above and below the sheet.
the electric field E

Solution
The motion does not affect the electric field, which is still directed away from the sheet
and still has magnitude

E=
20
~
iii. Assuming the sheet is moving with velocity ~v = v
x, determine the magnetic field B
(magnitude and direction) above and below the sheet.

Solution
Application of the right-hand rule indicates that (for v > 0) there is a magnetic field
in the y direction for z > 0 and in the +y direction for z < 0. From Amperes law
applied to a loop of length l normal to the x direction,
2Bl = 0 vl
c
Copyright 2014
American Association of Physics Teachers

2014 USA Physics Olympiad Exam

Part B

21

(In a time t, an area vtl moves through the loop, so the charge that moves is vtl and
the current is vl.) So
0 v
B=
2
This can also be written as
~ = 0 v y
B

2
above the sheet and
~ = 0 v y
B

2
below the sheet
iv. Assuming the sheet is moving with velocity ~v = v
z (perpendicular to the sheet), deter~
mine the electric field E (magnitude and direction) above and below the sheet.

Solution
Once again the motion does not affect the electric field, which is still directed away from
the sheet and still has magnitude

E=
20
~
v. Assuming the sheet is moving with velocity ~v = v
z, determine the magnetic field B
(magnitude and direction) above and below the sheet.

Solution
There is no magnetic field above and below the sheet. Interestingly, theres no magnetic
field at the sheet either. Consider an Amperian loop of area A in the x-y plane as the
sheet passes through. The loop experiences a current of the form
A(t).
But the loop also experiences an oppositely directed change in flux of the form
A

(t),
0

so the right-hand side of Amperes law remains zero.


~ = Ex x
b. In a certain region there exists only an electric field E
+ Ey y
+ Ez
z (and no magnetic
~ 0 and B
~ 0 as meafield) as measured by an observer at rest. The electric and magnetic fields E
~ regardless
sured by observers in motion can be determined entirely from the local value of E,
of the charge configuration that may have produced it.
~ 0 as measured by an observer moving with
i. What would be the observed electric field E
velocity ~v = v
z?

c
Copyright 2014
American Association of Physics Teachers

2014 USA Physics Olympiad Exam

Part B

22

Solution
The electric field was unaffected by the motion of the sheet of charge, so the electric
field in the frame of reference of the moving observer should be the same:
~0 = E
~
E

~ 0 as measured by an observer moving with


ii. What would be the observed magnetic field B
velocity ~v = v
z?

Solution
No magnetic field was created by the motion of the sheet of charge in the direction of
the electric field, so the magnetic field in the frame of reference of the moving observer
should likewise not depend on the component of the electric field in the direction of
motion. When the sheet of charge was moving in the +x direction, a magnetic field was
created in the y direction; the observer moving in the +x direction is equivalent to the
sheet of charge moving in the x direction, creating a magnetic field in the +y direction.
That is, an electric field in the z direction causes an observer moving in the x
direction
to observe a magnetic field in the y = z x
direction.
Previously, furthermore, the electric and magnetic fields satisfied
B=

0 0
E
v

Combining this with the previous equation,


~ 0 = 0 0 ~v E
~
B
or, using c2 = 1/0 0 ,
~ 0 = 1 ~v E
~
B
c2
or in our case, where ~v = v
z,
~ 0 = v (Ey x
B
Ex y
)
c2

c. An infinitely long wire wire on the z axis is composed of positive charges with linear charge
density which are at rest, and negative charges with linear charge density moving with
speed v in the z direction.
~ (magnitude and direction) at points outside the wire.
i. Determine the electric field E

Solution
The wire as a whole is neutral, so there is no electric field outside the wire.

c
Copyright 2014
American Association of Physics Teachers

2014 USA Physics Olympiad Exam

Part B

23

~ (magnitude and direction) at points outside the wire.


ii. Determine the magnetic field B

Solution
The current in the wire is v, so Amperes Law quickly yields
B = 0

v
2r

in the tangential direction to a circle of radius r centered on and perpendicular to the


wire.
The actual direction is given by the right hand rule; since the current is in the z
direction, the circular B field lines would point clockwise looking in that direction.
iii. Now consider an observer moving with speed v parallel to the z axis so that the negative
charges appear to be at rest. There is a symmetry between the electric and magnetic
fields such that a variation to your answer to part b can be applied to the magnetic
field in this part. You will need to change the multiplicative constant to something
dimensionally correct and reverse the sign. Use this fact to find and describe the electric
field measured by the moving observer, and comment on your result. (Some familiarity
with special relativity can help you verify the direction of your result, but is not necessary
to obtain the correct answer.)

Solution
Part b indicated that a velocity boost in an electric field resulted in a magnetic field
given by
~ 0 = 1 ~v E.
~
B
c2
By symmetry, and insisting on a dimensionally correct answer, we assume that
~ 0 = ~v B
~
E
is correct, give or take a sign. We were told to switch the sign, so make it positive.
Taking the cross product yields an electric field vector that points outward, with magnitude
v
v2
E 0 = v0
=
2r
20 r c2
This can be attributed to length contraction of the positive charges and inverse length
contraction of the (now-stationary) negative charges.

c
Copyright 2014
American Association of Physics Teachers

2014 USA Physics Olympiad Exam

Answer Sheets

24

Answer Sheets

Following are answer sheets for some of the graphical portions of the test.

c
Copyright 2014
American Association of Physics Teachers

2014 USA Physics Olympiad Exam

Answer Sheets

25

Answer for Part A, Question 3


Space-time graph for accelerated rocket. The positions of Fred and the Alien at t = 0 are shown.

ct

t=0
Fred

Alien

c
Copyright 2014
American Association of Physics Teachers

2014 USA Physics Olympiad Exam

Answer Sheets

26

Answer for Part A, Question 4


E

Answer for Part A, Question 4


V

c
Copyright 2014
American Association of Physics Teachers

2014 USA Physics Olympiad Exam

Answer Sheets

27

Answer for Part A, Question 4


E

Answer for Part A, Question 4


V

c
Copyright 2014
American Association of Physics Teachers

2014 USA Physics Olympiad Exam

Answer Sheets

Answer for Part A, Question 4

c
Copyright 2014
American Association of Physics Teachers

28

You might also like